Sei sulla pagina 1di 46

Answers with Solutions JEE Main & Advanced Solutions (2021-18) 1

æ æ 1 - x öö
Differentiation 1 1
3. (c) f (x) = cos çç2 tan-1 sin çcot-1 ÷÷
x ø ÷ø
- è è
1. (17) Given, y4 + y 4 = 2x
1- x
Þ (y 1/ 4 + y -1/ 4 ) 2 = (2x) 2 cot-1 = sin-1 x
x
Þ (y 1/ 4 + y -1/ 4 ) 2 = 4x 2
\ f (x) = cos (2 tan-1 sinsin-1 x )
Differentiating w.r.t. x, we get or f (x) = cos (2 tan-1 x )
æ 1 1
- ö
æ2 x ö
1 ç y 4 - y 4 ÷ dy = 2 = cos tan-1 ç ÷
4y ç ÷ dx è1- x ø
è ø 1- x ö
f (x) = cos cos -1 æç ÷
æ 1 1
- ö è1 + x ø
Þ ç y 4 - y 4 ÷ dy = 8y …(i)
ç ÷ dx 1- x
è ø f (x) =
1+ x
1 1 æ 1 1
2 - (1 + x) - (1 - x) -2
- - ö f ' (x) = =
Now, y 4 -y 4 = çy 4 + y 4 ÷ - 4 (1 + x) 2 (1 + x) 2
ç ÷
è ø 2
1- x ö
1 1 f ' (x)(1 - x) 2 = -2æç ÷
- è1 + x ø
Þ y4 -= 2 x2 - 1
y 4 …(ii) 2 2
æ1 - x ö æ1 - x ö
Þ ( x 2 - 1)
dy
= 4y [using Eqs. (i) and (ii)] (1 - x) 2 f ' (x) + 2[f (x)] 2 = - 2 ç ÷ + 2ç ÷ =0
dx è 1 + x ø è1 + x ø
Squaring on both sides, æ æ 1 - 2 2x ö ö
2 4. (481) f (x) = sinççcos - 1 ç ÷÷
dy 2x ÷
(x 2 - 1) æç ö÷ = 16y 2 è è1 + 2 øø
è dx ø
2x
Let 2 be tan2 q.
Again, differentiating w.r.t. x
2 é æ 1 - tan2 q ö ù
dy d 2 y dy dy \ f (x) = sinêcos - 1 çç ÷ú
( x 2 - 1) × 2
× + 2x æç ö÷ = 32y 2 ÷
dx dx 2 è dx ø dx êë è 1 + tan q ø úû
2dy = sin[cos - 1 (cos 2q)] = sin2q
On dividing by , we get
dx 2 tan q 2×2x
= =
d 2y dy 1 + tan q 1 + 22 x
2
(x 2 - 1) 2 + x = 16y
dx dx æ 2x ö
d 2y dy f (x) = 2× çç ÷
2x ÷
2
or (x - 1 ) 2 + x - 16y = 0 è1 + 2 ø
dx dx
é (1 + 22 x ) (2x log2) - 2x (22 x log2×2) ù
Comparing with f ¢ (x) = 2× ê ú
d 2y dy êë (1 + 22 x ) 2 úû
(x 2 - 1 ) 2 + ax + by = 0 æ 5×2log2 - 2×8log2 ö
dx dx f¢ (1) = 2ç ÷
è 52 ø
a =1, b = - 16
æ 12 ö
\ a - b = 1 + 16 = 17 = ç - ÷ log2
è 25 ø
2. (40) -b
= loge 2
We have, In (x + y) = 4xy a
Þ x + y = e4 xy Þ a = 25 and b = 12
dy æ dy \ | a2 - b2 | min = | 252 - 122 | = 481
Þ 1+ = ç4x + 4y ö÷ e4 xy
dx è dx ø
5. (91.00)
If x = 0, then y = 1 6
dy Given, y = ì3 cos (kx) - 4 sin(kx) ü
-1
At (0, 1), =3 åk cosí
î5 5
ý
þ
dx k =1
d 2y dy
2 2
ö + e4 xy æç4x d y + 4dy + 4 dy ö÷ 3 4
= e 4 xy æ
ç4x + 4y ÷ Let = cos q and = sinq
dx 2 è dx ø ç
è dx
2
dx dx ÷ø 5 5
3 4
At x = 0, So, cos(kx) - sin(kx) = cos(kx + q)
5 5
d 2y 4
= 16 + 24 = 40 where, tanq =
dx 2 3
2 Differential Calculus

6 6
-1 2 æ 2x 1 – x 2 ö
\y= åk cos cos (kx + q) = å (k x + kq)
And, let b = tan–1 ç ÷
k =1 k =1 ç 1 –2x 2 ÷
è ø
6 ´ 7 ´ 13 ö æ 6 ´ 7 ö
= x æç ÷+ ç ÷q Put x = sinf
è 6 ø è 2 ø
æ 2sin fcos f ö –1 æ sin2 f ö
Þ y = 91x + 21q \ b = tan–1 ç ÷ = tan ç ÷
dy è 1 –2sin2 f ø è cos 2 f ø
\ = 91
dx x =0
= tan–1 (tan2f)
Þ b = 2f = 2sin–1 x
Hence, answer is 91.00.
da 1 æ 1 ö
6. (c) Given equation, y 2 + loge (cos 2 x) = y, ç ÷
da dx 2 è 1 + x 2 ø 1–x2
\ = = =
p p db d b 1 4(1 + x 2 )
x Îæç - , ö÷ … (i) 2
è 2 2ø dx 1–x 2

On differentiating w.r.t. x, we get 1


1–
2yy ¢ - 2 tan x = y ¢ … (ii) da 4 3
\ = =
Again, on differentiating w.r.t x, we get db x = 1/ 2 4æ1 + 1 ö 10
ç ÷
2(y ¢) 2 + 2yy ¢ ¢ - 2sec2 x = y ¢ ¢ è 4ø
Þ 2(y ¢) 2 + 2yy ¢ ¢ = 2sec2 x + y ¢ ¢ … (iii) 9. (5.00)
From Eq. (i), at x = 0 Þ y = 0 or 1 ì 5 æ1ö 2
Now, from Eq. (ii) ï x sin çè x ÷ø + 5x , x < 0
at x = 0, y = 0 Þ y¢ = 0, ï
Given function, f (x) = í 0 , x =0
or at x = 0, y = 1 Þ y¢ = 0 ï 5
x cos æ 1 ö + lx 2 , x > 0
ï ç ÷
î èxø
Now, from Eq. (iii) ì 4 æ1ö 3 æ1ö x <0
ïï5 x sin çè x ÷ø - x cos çè x ÷ø + 10x,
at x = 0, y = 0 and y¢ = 0 y ¢ ¢ = -2 Þf ¢ ( x) = í
So, at x = 0, y = 1 and y¢ = 0 Þ y ¢ ¢ = 2 ï5 x 4 cos æç 1 ö÷ + x 3 sin æç 1 ö÷ + 2lx, x > 0
Þ y¢ ¢ (0) = 2 \ | y¢ ¢ (0)| = 2 ïî èxø èxø
Hence, option (c) is correct. ì 3 æ 1ö 2 æ1ö
ï 20x sin çè x ÷ø - 5 x cos çè x ÷ø
7. (b) It is given (a + 2b cos x)(a - 2b cos y) = a 2 - b 2 ï x <0
ï - 3 x 2 cos æ 1 ö + x sin 1 + 10,
where a > b > 0, ï ç
èxø
÷
x
Þf ¢ ¢ ( x) = í Q It is given that f¢ ¢ (0)
On differentiating w.r.t. ‘y’, we get 1 1
ï20x cos ç ö÷ + 5 x 2 sin æç ö÷
3 æ
æ dx ö ï èxø èxø
(a - 2bcos y) ç0 + 2b (- sin x) ÷ x >0
è dy ø ï 2 æ 1ö 1
ï + 3 x sin ç ÷ - x cos + 2l,
+ (a + 2bcos x)(0 - 2b (- sin y)) = 0 î èxø x
æ dx ö exists.
Þ (a - 2bcos y) ç - 2b (sin x) ÷ + 2b(sin y)(a + 2 bcos x) = 0
è dy ø So, f ¢ ¢ (0+ ) = f ¢ ¢ (0- )
æ p pö Þ 2l = 10 Þ l = 5
At ç , ÷, we get
è4 4ø 10. (c) It is given that x k + y k = a k ,(a, k > 0)
æ dx ö dy
(a - b) ç -b ÷ + b (a + b) = 0 On differentiating both sides w.r.t. ‘x’, we get kxk - 1 + kyk - 1 =0
è dy ø k -1 dx
dy æ x ö
dx a + b Þ + ç ÷ = 0, [as, k ¹ 0] …(i)
Þ = dx è y ø
dy a - b 1/ 3
dy æ y ö
Since, + ç ÷ = 0 (given) …(ii)
Hence, option (b) is correct. dx è x ø
æ 1 + x 2 –1 ö On comparing Eqs. (i) and (ii), we get
8. (d) Let a = tan–1 ç ÷. Put x = tan q 1 2
ç x ÷ k -1= - Þ k =
è ø 3 3
sec q –1 ö –1 æ 1 –cos q ö 11. (d) It is given that
\ a = tan–1 æç ÷ = tan ç ÷
è tan q ø è sin q ø
æ tan a + cot a ö 1 3p
æ 2sin q /2 2 ö y(a) = 2çç ÷÷ + , a Îæç , p ö÷
= tan–1 çç ÷÷ 2
è 1 + tan a ø sin a
2 è4 ø
è 2sin q / 2 cos q / 2 ø
q 1 æ tan2 a + 1 ö 1
= tan (tan q /2) = = tan–1 x
–1 Þ y(a) = 2cot açç 2 ÷
÷+
2
2 2 è 1 + tan a ø sin a
= 2cota + cosec2 a
JEE Main & Advanced Solutions (2021-18) 3

= 2cot a + 1 + cot2 a é 2 tan-1 x, | x| £1


-1 2xê -1
Q sin = p - 2 tan x, x >1
= (1 + cot a) 2 = |1 + cot a| 1 + x2 ê
ê
-1
- p - 2 tan x x < 1
3p ë
Qcot a Î(-¥, -1), for a Îæç , p ö÷
è4 ø p
Q at x = 3; y = (given)
\ y(a) = - (1 + cot a) [Q| x| = - x, for x < 0] 6
p
dy \ 2 = p - 2 tan-1 ( 3) + C Þ C = 0
\ = - (0 - cosec2 a) = cosec2 a 6
da
Then at x = - 3
dy 5p
So, = cosec2 æç ö÷ 2p p
da a = 5 p è6ø 2y = - p - 2 tan-1 (- 3) = - p + =-
6 3 3
p
p p Þ y=-
= cosec ç p - ö÷ = cosec2 æç ö÷ = (2) 2 = 4

6
è 6ø è6 ø
Hence, option (b) is correct.
12. (b) Given functional relation is 14. (*) It is given that x = 2sin q - sin2 q
2 2
y 1- x =k - x 1- y and y = 2cos q - cos 2 q, q Î[0, 2p]
On differentiating both sides w.r.t. x, we get dy
dy dy dq -2sin q + 2sin2 q sin2 q - sin q
2xy \ = = =
-2yx 2 dy dx - 1 - y 2 dx dx 2cos q - 2cos 2 q cos q - cos 2 q
+ 1- x =0+
dx dq
2 1 - x2 2 1 - y2
d 2y d æ dy ö d æ dy ö dq
dy ìï xy üï ìï yx ü
ï \ = ç ÷= ç ÷´
Þ 2
í 1- x - ý =í - 1 - y2 ý dx 2 dx è dx ø dq è dx ø dx
dx ï 2
1 - y ïþ ïî 1 - x 2
ïþ
î d æ sin2 q - sin q ö 1
= ç ÷´
2 2
dy ìï 1 - x 1 - y - xy üï xy - 1 - x 1 - y
2 2 dq è cos q - cos 2 q ø dx
Þ í ý= dq
dx ï 1- y 2
ïþ 1- x 2
î (cos q - cos 2 q)(2cos 2 q - cos q)
dy 1 - y2 =
- (sin2 q - sin q)(- sin q + 2sin2 q)
Þ =-
dx 1 - x2 (cos q - cos 2 q) 2
1
1 1 ´
Q y æç ö÷ = - (given) (2cos q - 2cos 2 q)
è2 ø 4
2 d 2y (-1 - 1)(2 + 1) - (0 - 0)(-0 + 0) -2 ´ 3 3
1 16 - 1 \ = = =
1 - æç - ö÷ dx 2 2(-1 - 1) 3 -2 ´ 8 8
dy è 4ø q =p
\ =- = - 16
dx x = 1 2 4-1 15. (b) Given expression is
1
2 1 - æç ö÷ 4 2
è2 ø æ æ 3 cos x + sin x ö ö
2y = ççcot-1 ç ÷÷
15 5 è è cos x - 3 sin x ø ÷ø
=- =-
2 3 2 2
æ æ 3 cot x + 1 ö ö
13. (b) It is given that f (x) = (sin(tan-1 x) + sin(cot-1 x)) 2 - 1 = ççcot-1 ç ÷÷
2 è è cot x - 3 ø ÷ø
é æ æ öö ù
êsinçsin-1 ç x ÷ ÷ ú [dividing each term of numerator and denominator by sin x]
2
ç ç 2 ÷ ÷ æ ö 2
ê è 1+ x øø ú x 1 æ æ cot p cot x + 1 ö ö
è
=ê ú - 1= ç + ÷ -1 ç ç ÷÷
ç 2
1 + x 2 ÷ø -1
= çcot ç 6 éQcot p = 3 ù
ê æ -1 æ 1 ö ö ú è 1+ x ÷÷ êë úû
ê + sinççsin ç ÷ ÷÷ ú ç p
ç cot x - cot ÷ ÷÷ 6
è 1 + x 2 ø ø úû ç
êë è è è 6 øø
(x + 1) 2 x 2 + 1 + 2x - 1 - x 2 2
cot A cotB + 1 ù
= -1 = æ æ p öö
= çcot-1 çcotæç - x ö÷ ÷ ÷
é
1+ x 2
1+ x 2 êQcot(A - B) = ú
è è è 6 ø øø ë cotB - cot A û
2x
Þ f (x) = …(i) for | x | > 1 ì æp 2
1 + x2 ö
ï ç - x÷ , 0< x <
p
dy 1 d ï è6 ø 6
and, also given that = (sin-1 (f (x))) =í 2
dx 2 dx æ
ï p+ æ p ö ö p p
ïîçè ç - x ÷÷ , <x<
Þ 2y = sin-1 (f (x)) + C (on integrating both è6 øø 6 2
sides)
é ìp + q, - p < q < 0ù
ê ï
æ 2x ö
Þ 2y = sin-1 çç ÷ + C, for | x | > 1 êQcot-1
(cot q) = í q, 0< q< p ú

ú
è1 + x ø ê ï
î q - p, p < q < 2p úû
ë
4 Differential Calculus

2 dy 1
ì æp ö p Now, on putting x = 0, y = 1 and = - in Eq. (iv), we get
ï ç - x÷ , 0 < x < dx e
ï è6 ø 6
Þ 2y = í d 2y 1
2
æd 2 y ö 1 1
+ e1 æç - ö÷ + 0çç 2 ÷÷ + æç - ö÷ + æç - ö÷ = 0
2
ïæ 7 p p p e1
- x ö÷ , <x< dx 2 è eø è dx ø è e ø è e ø
ïîçè 6 ø 6 2
ì æp p d 2y 1
2 - x ö÷ (-1), 0 < x < Þ =
dy ïï çè 6 ø 6 dx 2 e2
Þ 2 =í (0, 1)
dx ï2æ 7 p - x ö (-1), p < x < p æ dy d 2 y ö
ç ÷ 1 1
îï è 6 ø 6 2 So, çç , ÷ at (0, 1) is æç - , ö÷.
2÷ è e e2 ø
è dx dx ø
ì x - p, 0< x < p
dy ï æ sin x - cos x ö -1 æ tan x - 1 ö
Þ =í 6 6 19. (d) Let f (x) = tan-1 ç ÷ = tan ç ÷
dx ïx - 7 p , p < x < p è sin x + cos x ø è tan x + 1 ø
î 6 6 2
[dividing numerator and
16. (d) Let y = f (f (f (x))) + (f (x)) 2 p ù
denominator by cos x > 0, x Îæç0, ö÷ ú
On differentiating both sides w.r.t. x, we get è 2 øû
dy [by chain rule] æ p ö
= f ¢ (f (f (x))) ×f ¢ (f (x)) ×f ¢ (x) + 2f (x)f ¢ (x) ç tan x - tan ÷
dx -1 4 ÷
= tan ç
dy ç æ pö ÷
So, = f ¢ (f (f (1))) × f ¢ (f (1)) × f ¢ (1) + 2f (1)f ¢ (1)
dx ç 1 + çè tan ÷ø (tan x) ÷
at x = 1 è 4 ø
dy [Qf(1) = 1 and f¢ (1) = 3] é tan A - tanB
\ = f ¢ (f (1)) ×f ¢ (1) × (3) + 2(1)(3) é p ù
= tan-1 ê tanæç x - ö÷ ú
ù
= tan (A - B) ú
dx x = 1 êQ
ë è 4 øû ë 1 + tan A tanB û
= f ¢ (1) × (3) × (3) + 6 = (3 ´ 9) + 6 p
Since, it is given that x Îæç0, ö÷, so
= 27 + 6 = 33 è 2ø
17. (b) Given functions, f (x) = loge (sin x) , (0 < x < p) and g(x) = sin- 1 (e - x ) , p p p
x - Îæç - , ö÷
x ³ 0. 4 è 4 4ø
Now, fog(x) = f (g(x)) = f (sin- 1 (e- x )) p p p
Also, for æç x - ö÷ Îæç - , ö÷,
= loge (sin(sin- 1 (e- x ))) è 4ø è 4 4ø
= loge (e- x ) {Qsin(sin- 1 x) = x, if x Î[- 1, 1]} æ p ö p
=-x …(i) Then, f (x) = tan-1 ç tan æç x - ö÷ ÷ = x -
d è è 4 ø ø 4
and (fog) ¢ (x) = (- x) = - 1 …(ii)
dx é -1 æ p p öù
êQ tan tan q = q, for q Îçè - 2 , 2 ÷ø ú
According to the question, x
ë û
Q a = (fog) ¢ (a) = - 1 [from Eq. (ii)] Now, derivative of f (x) w.r.t. is
2
and b = (fog) (a) = - (a) [from Eq. (i)] d (f (x)) df (x) d æ pö
=2 =2´ çx - ÷ = 2
for a positive real value ‘a’. d (x /2) d (x) dx è 4ø
Since, the value of a = - 1 and b = - a, satisfy the quadratic 20. (b) We have, x = 3 tan t and y = 3 sec t
equation (from the given options)
dy d
aa2 - ba - a = 1. (3sect)
dy dt dt
Clearly, = =
18. (b) Key Idea Differentiating the given equation twice w.r.t. ‘x’. dx dx d
(3 tant)
Given equation is dt dt
ey + xy = e …(i) 3 sec t tant tant
= = = sint
On differentiating both sides w.r.t. x, we get 3 sec2 t sec t

ey
dy dy
+ x + y =0 …(ii) d 2y
d æ dy ö d æ dy ö dt
and =
ç ÷ = ç ÷×
dx dx dx 2
dx è dx ø dt è dx ø dx
dy æ y ö d æ dy ö d
Þ = - çç y ÷÷ …(iii) ç ÷ (sin t)
dx èe + x ø = dt è dx ø dt
=
dx d
Again differentiating Eq. (ii) w.r.t. ‘x’, we get (3 tan t)
2 dt dt
d 2y dy d 2 y dy dy
ey 2
+ ey æç ö÷ + x 2 + + =0 cos t cos 3 t
dx è dx ø dx dx dx = =
3sec2 t 3
…(iv) p
2 cos 3
Now, on putting x = 0 in Eq. (i), we get d y p 4 = 1 = 1
Now, 2 æçat t = ö÷ =
ey = e1 Þ y = 1 dx è 4ø 3 3(2 2) 6 2
On putting x = 0, y = 1 in Eq. (iii), we get 21. (b) We have, x loge (loge x) - x 2 + y 2 = 4, which can be written as
dy 1 1
=- =- y 2 = 4 + x 2 - x loge (loge x) … (i)
dx e+0 e
Now, differentiating Eq. (i) w.r.t. x, we get
JEE Main & Advanced Solutions (2021-18) 5

dy 1 1 f¢ (1) = - 5
2y = 2x - x . - 1×loge (loge x) Þ
dx loge x x Þ f¢ ¢ (2) = 12 + 2 (- 5) = 2 [using Eq. (v)]
[by using product rule of derivative] \ f (x) = x 3 + x 2f ¢ (1) + xf ¢ ¢ (2) + f ¢ ¢ ¢ (3)
1
2x - - loge (loge x) Þ f (x) = x 3 - 5x 2 + 2x + 6
dy log ex
Þ æç ö÷ = … (ii) Þ f(2) = 23 - 5(2) 2 + 2(2) + 6
è dx ø 2y
= 8 - 20 + 4 + 6 = - 2
Now, at x = e, y 2 = 4 + e2 - eloge (loge e) [using Eq. (i)]
23. (b) Given equation is
2 2 2
= 4 + e - eloge (1) = 4 + e - 0 = e + 4 (2x) 2 y = 4×e2 x - 2 y ...(i)
Þ y = e2 + 4 [Q y > 0] On applying ‘loge ’ both sides, we get
\ At x = e and y = e2 + 4,
loge (2x) 2 y = loge 4 + loge e2 x - 2 y
dy 2e - 1 - 0 2e - 1
= = [using Eq. (ii)] 2y loge (2x) = loge (2) 2 + (2x - 2y)
dx 2 e2 + 4 2 e2 + 4
[Qloge nm = mloge n and loge ef (x) = f (x)]
22. (c) We have, f (x) = x 3+ x 2f ¢ (1) + xf ¢ ¢ (2) + f ¢ ¢ ¢ (3)
Þ (2loge (2x) + 2) y = 2x + 2loge (2)
Þ f ¢ (x) = 3x 2 + 2xf ¢ (1) + f ¢ ¢ (2) … (i)
x + loge 2
Þ f ¢¢(x) = 6x + 2f ¢ (1) … (ii) Þ y=
1 + loge (2x)
Þ f ¢¢¢(x) = 6 … (iii)
Þ f ¢¢¢(3) = 6 On differentiating ‘y’ w.r.t. ‘x’, we get
2
Putting x = 1 in Eq. (i), we get (1 + loge (2x))1 - (x + loge 2)
dy 2x
f ¢ (1) = 3 + 2f ¢ (1) + f ¢ ¢ (2) … (iv) =
dx (1 + loge (2x)) 2
and putting x = 2 in Eq. (ii), we get
1
1 + loge (2x) - 1 - loge 2
f ¢ ¢ (2) = 12 + 2f ¢ (1) …(v) x
=
From Eqs. (iv) and (v), we get (1 + loge (2x)) 2
f ¢ (1) = 3 + 2f ¢ (1) + (12 + 2f ¢ (1)) dy æ x loge (2x) - loge 2 ö
So, (1 + loge (2x)) 2 =ç ÷
Þ 3f¢ (1) = - 15 dx è x ø

Functions
3p p 1
1. (d) f (x) = log 5 æç3 + cos æç + x ö÷ + cos æç + x ö÷ x³-
è è4 ø è4 ø 2
p 3p ö
+ cos æç - x ö÷ - cos æç - x ö÷ ÷ So,
1
x Î éê - , ¥ùú - {0}
è4 ø è4 øø
ë 2 û
= log 5 (3 - 2 sin x + 2 cos x) 4. (b) We have,
Q - 2 £ - 2 sin x + 2 cos x £ 2 100 é
(- 1)n n ù
åê ú (Q [x] is the greatest integer function)
Þ1 £ 3 - 2 sin x + 2 cos x £ 5 n = 8ë 2 û
Þlog 5 1 £ log 5 (3 - 2 sin x + 2 cos x) £ log 5 5 Substitute the values of n
Þ0 £ f (x) £ 2 Þf (x) Î[0, 2] = [4] + [- 4.5] + [5] + [- 5.5] + K + [- 49.5] + [50]
= 4 - 5 + 5 - 6 + K - 50 + 50 = 4
2. (b) f (m + n) = f (m) + f (n), m, n ÎN
| [x] | - 2 | [x] | - 2
\ f (3 + 3) = f (3) + f (3) 5. (c) f (x) = ³0
| [x] | - 3 | [x] | - 3
Þ f (6) = 2f (3) = 18 [Q f (6) = 18]
Also f (3) = f (2 + 1) = f (2) + f (1) = f (1 + 1) + f (1) Let | [x] | = t
f (3) = f (1) + f (1) + f (1) + – +
Þ 9 = 3f(1) Þ f(1) = 3 2 3
t Î (–¥, 2] È (3, ¥)
\ f (2) = f (1 + 1) = f (1) + f (1) = 3 + 3 = 6
Hence, f (2) ×f (3) = 6×9 = 54
1+ x ö 1+ x 3
3. (d) f (x) = cosec -1 æç ÷ ³1
è x ø x 2
Clearly , x ¹ 0
| 1+ x| 2 ³ | x| 2 –3 –2 1 2 3
1 + x 2 + 2x ³ x 2
2x + 1 ³ 0
6 Differential Calculus

| [x] | = 3 Þ x Î[- 3, - 2) È [3, 4) x -2


9. (c) Given, f (x) =
Domain of x = [-¥, - 3) È [- 2, 3) È [4, ¥) x-3
g(x) = 2x - 3
a= -3
x -2
b= -2 Let y = f (x) =
x-3
c =3
\ a + b + c = - 3 + (- 2) + 3 = - 2 Þ xy - 3y = x - 2
cosec x -1 Þ xy - x = 3y - 2
6. (b) Given, f (x) =
x - [x] Þ x (y - 1) = 3y - 2
cosec-1x 3y - 2
Þ f (x) = Þ x=
{x} y -1
For f (x) to be defined, -1 3y - 2
Þ f (y) =
ì | x| ³ 1 ìx £ - 1 or x ³ 1 y -1
í Þí
î { x } > 0 î x ¹ 1 integers -1 3x - 2
Þ f (x) =
i.e. x Î(-¥, - 1] È [1, ¥) - {integers} x -1
i.e. all non-integers except the interval [-1, 1] x+3
Similarly, g-1 (x) =
(here, -1 and 1 are included in except case, because of -1 and 1 2
are integers). -1 -1 13
Given, f (x) + g (x) =
2
7. (2) Given, a + a = 1
3x - 2 x + 3 13
b+ b =2 Þ + =
x-1 2 2
1 b
Q a×f (x) + a×f æç ö÷ = bx + … (i) Þ x 2 + 8x - 7 = 13 (x - 1)
èxø x 2
Þ x - 5x + 6 = 0
1
Replace x by , Þ (x - 2) (x - 3) = 0
x
æ 1ö b Þ x = 2,3
af ç ÷ + af (x) = + bx … (ii)
èxø x \ Sum = 2 + 3 = 5
Adding Eqs. (i) and (ii), we get 10. (c) y = 5logx
é 1 ù 1 Taking log on both sides,
(a + a) êf (x) + f æç ö÷ ú = æç x + ö÷ (b + b)
ë è x øû è xø log y = log x ×log5
1 log x 1
1 Þ = Þ = logy x
f (x) + f æç ö÷ log5 log y log5
èxø b + b 2
Þ = = =2 1
1 a+ a 1
x+ x=y log5
x
x 2 -`x - 2 11. (a) Given function f : R ® R, satisfies f (x + y) = f (x) + f (y)
8. (c) Given, g(x) = , f (x) = sin x
-1
2x 2 - ` x - 6 So, at x = y = 1
f (g(x)) = sin (g(x))
-1 f (2) = f (1) + f (1) = 2f (1) = 2 ´ 2 = 4
æ x2 - x - 2 ö Q f(1) = 2
fog(x) = sin-1 çç 2 ÷÷ At x = 2 and y = 1
è 2x - x - 6 ø
f (3) = f (2) + f (1) = 4 + 2 = 6
For the domain of fog(x), Similarly, f(4) = 8 and so on.
| g(x)| £ 1 [QDomain of f (x) is [-1, 1] n-1
Now, as g(n) = S f (k) = f (1) + f (2) + f (3) + ....+ f (n - 1)
½ x 2 - x - 2½ k =1
Þ ½ 2 ½£ 1
½2x - x - 6½ = 2 + 4 + 6 + ....+ 2 (n - 1)
(n - 1) (n)
½ (x + 1) (x - 2) ½ =2´ = n2 - n
Þ ½ ½£ 1 2
½(2x + `3) (x - 2)½
So, if g(n) = 20 Þ n2 - n = 20
½ x + 1½ x+1
Þ ½ ½£ 1 Þ -1 £ £1 Þ n (n - 1) = 20 Þ n = 5
½2x + 3½ 2x + 3
Hence, option (a) is correct.
2
æ x+1 ö
Þ ç ÷ £ 1 Þ (x + 1) 2 £ (2x + 3) 2
è 2x + `3 ø 12. (d) The given function f : (1, 3) ® R, defined by
2
Þ 3x + 10x + 8 ³ 0 Þ (3x + y)(x + 2) ³ 0 x (1)
, x Î(1, 2)
This implies, x [x] 1 + x2
f (x) = =
é 4 1 + x2 x (2)
x Î(-¥, - 2] È ê - , ¥ö÷ , x Î[2, 3)
ë 3 ø 1 + x2
JEE Main & Advanced Solutions (2021-18) 7

x 15. (d) Given, function f (x) = a x , a > 0 is written as sum of an even and
2
, x Î(1, 2)
1+ x odd functions f1 (x) and f2 (x) respectively.
Q f (x) =
2x ax + a- x ax - a- x
, x Î[2, 3) Clearly, f1 (x) = and f2 (x) =
2
1+ x 2 2
So, f1 (x + y) + f1 (x - y)
is a decreasing function, so 1 1
2(3) 3 = [ax + y + a- (x + y ) ] + [ax - y + a- (x - y ) ]
lim f (3 - h) ¾® = 2 2
h ®0 1+ 9 5
1é 1 ax ay ù
4 = êax ay + x y + y + x ú
f(2) = 2ë aa a a û
5
2 1 1é 1 1 1 ù
Similarly, f (x ®2- ) tends to and f (x ®1+ ) tends to . = êax æçay + y ö÷ + x æç y + ay ö÷ ú
5 2 2ë è a ø a èa øû
2 1 3 4
So range of the given function ‘f’ is æç , ö÷ È æç , ùú 1 1 1
è5 2 ø è5 5 û = æçax + x ö÷ æçay + y ö÷
2è a ø è a ø
Hence, option (d) is correct.
æ ax + a- x ö æ ay + a- y ö
8 2x - 8 -2x = 2 çç ÷÷ çç ÷÷ = 2f1 (x) ×f1 (y)
13. (c) Given function, f (x) =
2x -2x
, x Î (-1, 1)
2 2
è øè ø
848x -+1 8
= = y (let) 1
84 x + 1 16. (a) We have, fk (x) = (sink x + cos k x), k = 1, 2, 3, …
k
1
On applying componendo and dividendo law, we get \ f4 (x) = (sin4 x + cos 4 x)
1+ y 4
84x = 1
1- y = ((sin2 x + cos 2 x) 2 - 2sin2 x cos 2 x)
4
On applying logarithm having base ‘8’ both sides, we get
1 1 1 1
æ1 + y ö = æç1 - (sin2x) 2 ö÷ = - sin2 2x
4x = log8 ç ÷ 4è 2 ø 4 8
è1 - y ø
1
æ1 + y ö 1 æ1 + y ö and f6 (x) = (sin6 x + cos6 x)
1 6
Þ x = log8 ç ÷ = (log8 e) loge ç ÷
4 è1 - y ø 4 è1 - y ø 1
= {(sin2 x + cos 2 x) 3 - 3sin2 x cos 2 x
{by base change property of logarithm loga b = loga e×loge b} 6
1 3
By interchanging the variables x and y, we get the inverse (sin2 x + cos 2 x)} = ìí1 - (2sin x cos x) 2 üý
1 æ1 + x ö 6î 4 þ
function of f (x) and it is f -1 (x) = (log8 e) loge ç ÷. 1 1 2
4 è1 - x ø = - sin 2x
6 8
Hence, option (c) is correct. 1 1 3-2 1
Now, f4 (x) - f6 (x) = - = =
4 6 12 12
14. (c) The given function f : R ® R is
f (x) = | x| (x - sin x) … (i) 17. (119) Given, X has exactly 5 elements and Y has exactly 7 elements.
Q The function ‘f’ is a odd and continuous function \ n(X) = 5 and n(Y) = 7
and as lim f (x) = ¥ and lim f (x) = - ¥, so range is R, therefore Now, number of one-one functions from X to Y is
x®¥ x ® -¥ a = 7P5 = 7C5 ´ 5!
‘f’ is a onto function. Number of onto functions from Y to X is b
é x (x - sin x), x ³ 0
Q f (x) = ê a1 b1
ë - x (x - sin x), x < 0
a2 b2
é 2x - sin x - x cos x, x > 0
\ f ¢ (x) = ê
ë - 2x + sin x + x cos x, x < 0
é (x - sin x) + x (1 - cos x), x > 0
ê (- x + sin x) - x (1 - cos x), x < 0 a7 b5
ë
Q for x > 0, x - sin x > 0 and x (1 - cos x) > 0
\ f ¢ (x) > 0" x Î(0, ¥) 1, 1, 1, 1, 3 or 1, 1, 1, 2, 2
Þf is strictly increasing function, " x Î(0, ¥). 7! 7!
\ b= ´ 5! + ´ 5!
Similarly, for x < 0, - x + sin x > 0 3! 4! (2!) 3 3!
and (- x) (1 - cos x) > 0, therefore, f ¢ (x) > 0" x Î(- ¥, 0) = (7 C 3 + 3 7C 3 ) 5! = 4 ´ 7C 3 ´ 5!
Þf is strictly increasing function, " x Î(0, ¥) b - a (4 ´ 7C3 - 7C5 ) 5!
\ =
Therefore ‘f’ is a strictly increasing function for x ÎR and it implies 5! 5!
that f is one-one function. = 4 ´ 35 - 21 = 140 - 21 = 119
8 Differential Calculus

Limits
1. (b) Using L-Hopital’s rule 1é 1 1 1 1 1 1 ù
= ê æç - ö÷ + æç - ö÷ + ....+ æç - ö÷ ú
p 2 ë è2 3 ø è3 4 ø è 10 11 ø û
×2sec x ×sec x × tan x ×f (sec2 x) - 0
lim 4 [using Leibnitz 1 1 1 1 æ 11 - 2 ö 9
p 2x = æç - ö÷ = ´ ç ÷=
x® 2 2 11 2 è 2 ´ 11 ø 44
è ø
4
theorem]
5. (d) f (2) = 4, f ¢ (2) = 1
p
×2( 2) 2 × (1) f (2) x 2f (2) - 4f (x)
= 4 = 2f (2) Now, lim
p x®2 x -2

4 0
Applying L-Hospital Rule as form on putting x = 2
2. (c) Q a, b are distinct roots of x + bx + c = 0 2 0
2x f (2) - 4f ¢ (x)
Þ x 2 + bx + c = (x - a) (x - b) = 0 So, lim
2
x®2 1
e2 (x + bx + c)
- 1 - 2(x 2 + bx + c) = 2 × 2 × f (2) - 4f ¢ (2)
Now, lim
x ®b (x - b)2 = 4 × 4 - 4 × 1 = 12
e2 (x - a) (x - b) - 1 - 2(x - a) (x - b) a x e x - b log e (1 + x) + gx 2e - x
= lim 6. (3) lim = 10
x ®b (x - b)2
x ®0 x sin2 x

e2 (b - a + h)h - 1 - 2(b - a + h)h a x ex - b loge (1 + x) + gx 2e- x


= lim Now, lim
h ®0 h2 x ®0 x sin2 x
2
2(b - a + h)h a x (1 + x + x 2 /2 + ..) - b
1 + 2( b - a + h)h +
2! æ x2 x3 ö
+ ... - 1 - 2h(b - a + h) çç x - + ...÷÷ - gx 2 (1 - x + x 2 /2 ...)
= lim è 2 3 ø
h ®0 h2 = lim
2 2 x ®0 x sin2 x
2(b - a + h ) h + ...
= lim x
h ®0 h2 a b
x (a + b) + x 2 (a + b /2 + g) + x 3 æç - - g ö÷ ...
= 2(b - a) = 2(b2 - 4c)
2 è2 3 ø
= lim
x ®0 x sin2 x
3. (b) Given, lim ( x 2 - x + 1 - ax) = b
x®¥ For limit to exist, the numerator must have degree greater than
( x 2 - x + 1) + ax or equal to denominator.
Þ lim ( x 2 - x + 1 - ax) =b
x®¥
( x 2 - x + 1 + ax) Degree of denominator = 3
x 2 - x + 1 - a2 x 2 \ For limit to exist, a - b =0 ...(i)
Þ lim =b b
x®¥
x 2 - x + 1 + ax and a+ + g =0 ...(ii)
2
Limit exists only if a2 = 1 Also, for terms greater than degree ‘3’, gives 0 as x ®0
\ a= ± 1 a b
\ - - g = 10 ...(iii)
-x + 1 2 3
Þ lim =b
x®¥
x 2 - x + 1 + ax From Eq. (i), b = a
a 3a
1 From Eq. (ii), g = æç + aö÷ = -
-1 + è2 ø 2
x -1
Þ lim =b Þ =b Putting these in Eq. (iii),
x®¥ 1 1 1 +a
1- + 2 + a a a 3a 3a - 2a + 9a
x x - + = 10 Þ = 10
But a¹ -1 2 3 2 6
10a
Þ a=1 Þ = 10 Þ a = 6
6
1
\ b= - Now, a = b Þb = 6
2 - 3a
1 Again, g= Þg = - 9
(a, b) = æç1, - ö÷ 2
è 2ø
\ a + b + g = 6 + 6 - 9 = 12 - 9 = 3
9 x
4. (a) We have, S = lim å 7. (a) As, we know that,
x ® 2 n= 1 n(n + 1) x 2 + 2 (2n + 1) x + 4
9 r £ [r] < r + 1
2 1 9 (n + 2) - (n + 1)
= å = å 2r £ [2r] < 2r + 1
n = 1 4(n2
+ 3n + 2) 2 n = 1 (n + 1) (n + 2)
3r £ [3r] < 3r + 1
1 9 é 1 1 ù
= åê - M M M
2 n = 1 ë n + 1 n + 2 úû
nr £ [nr] < nr + 1
JEE Main & Advanced Solutions (2021-18) 9

Adding (r + 2r + 3r + 4r + ¼ + nr) £ [r] + [2r] + [3r] a - 4e4 x


L = lim 4x
+ [4r] + ... [nr] < (r + 1) + (2r + 1) + (3r + 1) + (4r + 1) + ¼ + (nr + 1) x ®0 a(e - 1) + ax (4e4 x )
Þr (1 + 2 + 3 + 4 + ¼ + n) £ [r] + [2r] + [3r] + ¼ + [nr] [Limit exist everywhere except a = 4]
< (r + 2r + 3r + ¼ + nr) + (1 + 1 + 1 + ¼ + 1) Again, apply L-Hospital rule,
1442443
n- times -16e4 x
n(n + 1) r × (n(n + 1)) L = lim
Þ r× £ [r] + [2r] + [3r] + ¼ + [nr] < +n x ®0 a(4e4 x ) + a(4e4 x ) + ax (16e4 x )
2 2
-16 -2 -2 -1
n(n + 1) ö n(n + 1) ö = = = = (use a = 4)
r × æç ÷ r × æç ÷+n 4a + 4a a 4 2
è 2 ø [r] + [2r] + [3r] + ¼ + [nr] è 2 ø
Þ £ <
n2 n2 n2 Given, L = b
1 -2 -1
n×næç1 + ö÷ ×r Þ = =b
n(n + 1) ×r è nø (1 + 0) ×r r a 2
Now, lim = lim = = …(i) -1
n ® ¥ 2×n2 n®¥ 2n 2
2 2 Then, a - 2b = 4 - 2æç ö÷ = 4 + 1 = 5
n(n + 1) è2ø
×r + n
2 n(n + 1) ×r + 2n 10. (d) Given, limit form is 1¥ .
and lim = lim
n®¥ n2 n®¥ 2n2 æ 1 1 1ö
ç1+ + + ¼ + ÷
ì 1 2ü 2 3 n÷
n2 íæç1 + ö÷r + ý
lim ç
n ® ¥ç n ÷
è n ø n þ (1 + 0) ×r + 0 r è æ1 1 ö æ÷ø1 1 1 1
ç
= lim î = = …(ii) L = e S = 1 + ç + ÷ + ç + + + ö÷ + ¼
n®¥ 2n2 2 2 è2 3 ø è4 5 6 7 ø
From Eqs. (i) and (ii), by Sandwich theorem, we conclude that, 1 1 1 1 1 1 1 1
Clearly, S < 1 + æç + ö÷ + æç + + + ö÷ + ¼ + æç n + ¼ + n ö÷
[r] + [2r] + [3r] + ¼ + [nr] r è2 2 ø è4 4 4 4 ø è2 23ø
lim = 144244
n®¥ n2 2 2 n times

Sandwich Theorem S <1+ 1+ 1+ 1+ ¼+ 1


Þ Let g(x) £ f (x) £ h(x) S <n+ 1
æ n+ 1 ö
and lim g(x) = lim h(x) = l lim ç ÷
x ®a x ®a n ® ¥ çè 2 n + 1 - 1 ÷ø
\ L=e Þ L = e0
\ lim f (x) = l
x ®a \ L =1
ae x - b cos x + ce - x sin2 (p cos 4 x)
8. (4) We have, lim =2 11. (c) lim
x ®0 x sin x x ®0 x4

æ x2 ö æ x2 x4 ö æ x2 ö sin2 [p (1 - cos 4 x)] p 2 (1 - cos 4 x) 2


açç1 + x + ¼÷÷ - bçç1 - + ¼÷÷ + cçç1 - x + ¼÷ = lim .
è 2! ø è 2 4! ø è 2! ÷ø x ®0 [p (1 - cos 4 x)] 2 x4
Þ lim
x ®0 æ x3 ö sin4 x (1 + cos 2 x) 2
x çç x - + ¼÷÷ = lim p 2 = lim p 2 (1 + cos 2 x) 2 = 4p 2
è 3! ø x ®0 x4 x ®0
tan3 x - tan x
æ a b cö 2 12. (d) a = lim
(a - b + c) + (a - c) x + ç + + ÷ x + ¼ x ® cos æ x + p ö
p
è2 2 2 ø ç ÷
Þ lim 4
è 4ø
x ®0
2 x4 tan x (tan x + 1) (tan x - 1)
x - +¼ = lim
6 p p

4 cos æç x + ö÷
Here, in numerator, all the coefficients of xk , where k < 2 has to è 4ø
be zero, then only limit will exist.
sin x æ sin x - cos x ö æ sin x + cos x ö
a-b+ c =0 ×ç ÷ç ÷
cos x è cos x øè cos x ø
a-c =0 = lim
p 1
Þ a=c x®
4
(cos x - sin x)
2
Þ b = 2a 1
- 2´ ´ 2
After solving limit, - 2 sin x (sin x + cos x) 2
a+ b+ c = lim = = -4
=2 x®
p
cos 3 x 1
2 4
2 2
So, a + 2a + a = 4 lim
cos x - 1
cot x x ® 0 tan x
Þ a=1 and b = lim (cos x) =e
x ®0
\ a = 1, b = 2 and c = 1 lim -
sin x
2
a+ b+ c =1+ 2+ 1=4 =e
x ®0
sec x
= e0 = 1
ax - (e 4x - 1) é 0 formù Equation whose roots are a and b, is
9. (5) lim = L (say)
x ®0 ax (e 4x - 1) êë 0 úû x 2 + 3x - 4 = 0
Apply L - Hospital rule, \ a =1,b = 3
10 Differential Calculus

æ x ö
é æ 1 9x 2 æ 1 x2
13. (c) lim çç ÷÷ ö öù
x ®0
è 8 1 - sin x - 8 1 + sin x ø x 3 ê çç + + ¼ ÷÷ + çç - +¼÷÷ ú
êë è 6 120 ø è 3 5 ø ûú
Þ L = lim
Rationalise denominator three times, x ®0 3x 3
æ x ö æ 8 1 - sin x + 8 1 + sin x ö 1 1 1+ 2
lim çç ÷ç ÷ +
1
x ® 0 è 1 - sin x - 1 + sin x ÷ Þ L= 6 3 = 6 =
ç ÷
8 8
ø è 8 1 - sin x + 8 1 + sin x ø
3 3 6
æ 4 1 - sin x + 4 1 + sin x ö æ 1 - sin x + 1 + sin x ö 1
ç ÷ ç ÷ \ 6L + 1 = 6 ´ + 1 = 2
ç 4 1 - sin x + 4 1 + sin x ÷ ç 1 - sin x + 1 + sin x ÷
è ø è ø 6
cos - 1 (x - [x] 2) × sin- 1 (x - [x] 2)
é x ù 8 16. (d) lim+
= lim ê ú ( 1 - sin x + 1 + sin x )
8
x ®0 x - x3
x ® 0 (1 - sin x) - (1 + sin x)
ë û
(4 1 - sin x + 4 1 + sin x ) ( 1 - sin x + 1 + sin x ) x ®0 + h
é x ù8 cos - 1 (h - 0) ×sin- 1 (h - 0)
= lim ê ú ( 1 - sin x + 1 + sin x )
8 = lim
x ® 0 - 2sin x
ë û
h ®0 h - h3
(4 1 - sin x + 4 1 + sin x ) ( 1 - sin x + 1 + sin x ) cos h×sin- 1 h
-1
= lim
1
= æç - ö÷ (2) (2) (2) éQ lim sin x = 1ù h ® 0 h (1 - h) (1 + h)

è 2ø êë x ® 0 x úû
æ sin- 1 h ö é cos - 1 h ù p
= lim çç ÷ê ú = 1×
= -4 x+ 2 h ® 0è h ÷ø êë (1 - h) (1 + h) úû 2
14. (3) lim (2 - cos x cos 2x ) x2 = 1¥ p
x ®0
x+2
RHL =
2
Þ lim (1 + 1 - cos x cos 2x ) x2
x ®0 17. (a) Method (I)
æx + 2ö
lim (1 - cos x cos 2 x ) ç æ tan(p cos 2 q) ö
x ®0 è x2 ø
÷
Let L = lim çç ÷
Þ e q ® 0 è sin(2p sin2 q) ÷
ø
x2 x4
Þ cos x = 1 - + - æ tan[p (1 - sin2 q)] ö
2! 4! = lim çç ÷
q ® 0 è sin(2p sin2 q) ÷ ø
(2x) 2 (2x) 4
Þ cos 2x = 1 - + -
2! 4! æ tan(p - p sin2 q) ö
1
= lim çç ÷
q ® 0 è sin(2p sin2 q) ÷
æ x2 x4 ö 2 ø
= çç1 - + - K÷÷ æç1 - 2x 2 + x 4 K÷ ö2
è 2 24 øè 3 ø æ - tan(p sin2 q) ö
= lim çç ÷
q ® 0 è sin(2p sin2 q) ÷
We have to extract till the coefficient of x 2 as denominator is x 2 . ø
1
æ x2 ö æ x2 ö é - tan(p sin2 q) ù
So, çç1 - ÷÷ (1 - 2x 2 ) 2 = çç1 - ÷÷ (1 - x 2 ) ê ´ (p sin2 q) ú
2ø 2ø
ú= -1
2
= lim ê (p sin 2q)
è è
æ x2 x4 ö æ 3 2 ö q ® 0 ê sin(2p sin q)
2 ú 2
= çç1 - - x2 + ÷ = ç1 - x ÷ ê 2
´ (2p sin q) ú
è 2 2 ÷ø è 2 ø ë (2p sin q) û
æx + 2ö
ç ÷
lim (1 - cos x cos 2 x ) è x 2 ø
Method (II)
So, ex ® 0
é æ 3x 2 öù é tan(p cos 2 q) ù æ 0ö
lim ê 1 - ç ÷ú æ x + 2 ö
÷ ç ÷ Let L = lim ê ú çForm ÷ [Using L-Hospital Rule]
ç
ë è 2 ø úû è x 2 ø
x ®0ê q ® 0 ê sin(2p sin2 q) ú è 0ø
=e ë û
æ 3x 2 ö
÷ æx + 2ö sec2 (p cos 2 q) (- 2p cos q×sin q)
ç
lim ç 2 ÷ ç ÷ L = lim
= ex ® 0
è ø è x2 ø
= e3 q ®0 cos(2p sin2 q) × (4p sin q×cos q)
\ a 3
e = e Þa = 3 - 1 (- 1) 2 - 1
-1 -1
= ´ =
sin x - tan x 2 1 2 2
15. (d) Given, L = lim x
x ®0 3x 3 ò0 (sin t )dt
18. (a) Given, lim
x ®0 x3
æ 12 × x 3 12 ×32 × x5 ö æ x 3 x5 ö
çç x + + + ¼ ÷÷ - çç x - + ¼÷ 0
è 3! 5! ø è 3 5 ÷ø Q It is of the form .
Þ L = lim 0
x ®0 3x 3 By differentiating numerator and denominator,
(using expansion of sin x and tan-1 x) -1
sin x 2 ×2x sin x ×2x
æ x 3 9x5 ö æ x 3 x5 ö lim = lim
çç + + ¼ ÷÷ - çç - + ¼÷ x ®0 3x 2 x ® 0 3x 2
è 3! 5! ø è 3 5 ÷ø 2 sin x 2 2
Þ L = lim = lim = (1) =
x ®0 3x 3 3 x ®0 x 3 3
JEE Main & Advanced Solutions (2021-18) 11

ì n ö üï
æ 1
19. (1) Given, lim tanïí å tan- 1 ç ÷
2 ý (a + 2x) 1 / 3 - (3 x) 1 / 3 é 0 formù
n® ¥
îï r = 1 è 1 + r + r ø þï 23. (d) lim , (a ¹ 0)
n
x ® a (3a + x) 1 / 3 - (4x) 1 / 3 ëê 0 ûú
æ ö
= tanç lim å [tan- 1 (r + 1) - tan- 1 r] ÷
çn ® ¥
è r =1
÷
ø Put x = a + h
(a + 2a + 2h) 1/ 3 - (3a + 3h) 1/ 3
p ö So, lim
æ h ®0 (3a + a + h) 1 / 3 - (4a + 4h) 1 / 3
= tanç lim æç tan- 1 (n + 1) - ö÷ ÷
èn ® ¥è 4 øø
é 1/ 3 1/ 3 ù
2h 3h
æ p pö
= tanç - ÷ = tan = 1
p (3a) 1/ 3 ê æç1 + ö÷ - æç1 + ö÷ ú
è2 4ø 4 êë è 3a ø è 3h ø ú
û
= lim
h ®0 1/ 3 1/ 3
Hence, the required value is 1. 1/ 3 æ
é hö æ 4h ö ù
(4a) ê ç1 + ÷ - ç1 + ÷ ú
20. (a) êë è 4a ø è 4a ø ú
û
ì æp ö æp öü é 1 + 2h - 1 - 3h + higher ù
ïï 3 sinçè 6 +h÷ø - cos çè 6 +h÷ø ïï ê ú
lim 2 í 9a 9a
ý 1/ 3 ê ú
h ®0
ï 3h ( 3 cos h - sinh) ï æ 3ö ê degree terms ú
= lim ç ÷
ïî ïþ h ®0 è 4 ø ê h 4h ú
ê 1 + 12a - 1 - 12a + higher ú
ì æ 3 æp ö 1 æ p öö ü ê ú
ï2 ç sinç +h÷ - cos ç +h÷ ÷ ï êë degree terms úû
ï è 2 è 6 ø 2 è 6 øø ï
= lim 2 í ý æ 2-3 ö æ -1 ö
h ®0
ï 2 ´ 3h æç 3 cos h - 1 sinhö÷ ï 3
1/ 3 ç ÷ 3 1/ 3 ç ÷
ï è 2 2 ø ïþ = æç ö÷ ç 9 9 ÷ = æç ö÷ ç 9 ÷
î è 4 ø ç 1 - 4 ÷ è 4 ø ç -3 ÷
è 12 12 ø è 12 ø
ì æpö æp ö æpö æp ö ü 1
ïïcos çè 6 ÷ø sinçè 6 +h÷ø -sinçè 6 ÷ø cos çè 6 +h÷ø ïï 3
1/3
4 4
1-
3 42 / 3 24 / 3 2 2
1/3
= lim 2 í ý = æç ö÷ = = = = æç ö÷
h ®0
ï p
3h æçcos cos h - sin sinhö÷
p ï è4 ø 2-
1
35 / 3
(3) 2 35 / 3 3 è9 ø
ïî è 6 6 ø ïþ 3 3
Hence, option (d) is correct.
ì æp pö ü
ïï sinçè 6 + h - 6 ÷ø ïï 1 + x 2 + x 4 – 1) / x ( 1 + x 2 + x 4 – 1) / x
= lim 2 í ý x (e ( –1) Lim e –1
24. (b) Lim = =1
h ®0
ï 3hcos æçh + p ö÷ ï x ®0
2
1 + x + x –1 4
x ®0
2
( 1 + x + x –1) / x 4
ïî è 6 ø ïþ
2 ì sinh ü é Lim (1 + x 2 + x 4 ) 1/ 2 –1 ey –1 ù
= lim í ý ê Q x ®0 = 0 and Lim
y ®0 = 1ú
h ®0 3 îhcos(h + p / 6) þ ë x y û
2 sinh 1
= × lim × lim æ (x - 1) 2 ö
3 h ® 0 h h ® 0 cos(h + p / 6) çò t cos (t 2) dt ÷
25. (*) L = lim ç 0 ÷
2 1 2 2 4 x®1
ç (x - 1) sin(x - 1) ÷
= × (1) × = × 1× = è ø
3 cos(p / 6) 3 3 3
Let x - 1 = h, so as x ®1 Þh ®0
x + x2 + x3 + K + xn - n æh 2 ö
21. (40) Given, lim = 820, (n ÎN) ç t cos(t 2 )dt ÷
x®1 x -1 ç ò ÷
ìï x - 1 x 2 - 1 x 3 - 1 xn -1 üï \ L = lim ç 0 ÷
Þ lim í + + + ... + ý = 820 h ®0 ç hsinh ÷
x ® 1 ï x -1 x -1 x -1 x -1 þï
î ç ÷
ç ÷
Þ 1 + 2 + 3 + ... + n = 820 è ø
n (n + 1) On applying L’Hospital rule, we get
Þ = 820
2 æ h2 (cos(h4 )) ×2h ö
L = lim çç ÷
Þ n(n + 1) = 1640 = 40 ´ 41 Þ n = 40 h ®0 è hcos h + sinh ÷ ø
22. (b) For some l ÎR - {0, 1} 2h2 cos(h4 ) 0 ´ 1
= lim = =0
1 - x + | x| h ®0 sinh 1 + 1
lim =L cos h +
x ®0 l - x + [x] h
(*) None option is correct.
1-x - x
Þ L = lim
x ® 0- l -1 3 x + 3 3 - x - 12 æ 0 formö
26. (36) lim ç ÷
1- x + x
x®2 3- x /2 - 31 - x è0 ø
= lim
x ® 0+ l -0 Put x = 2 + h as x ®2 Þ h ®0
1 1 1
Þ L= = Q |l - 1| = | l| Þ = 32 + h + 31 - h - 12 9×3h + 3×3-h - 12
|l - 1| | l| 2 = lim = lim
h ®0 h h ®0 1 -h / 2
\ L=2
-1 - (3 - 3-h )
3 2 - 3 -1 - h 3
12 Differential Calculus

9(3(3h - 1) + (3-h - 1)) æ x2 ö x2


= lim 2sin2 çç ÷÷ .2sin2
h ®0 3-h (3h / 2 - 1) è4ø 8
Þ lim = 2-k
x ®0 x8
é æ 3h - 1 ö æ 3-h - 1 ö ù ìé 2 8 ü
ê 3çç ÷÷h + çç ÷÷ (-h) ú ïê sin x
2
x2 ù x
è h ø è -h ø sin ú 2
ï
= lim 9×3h êê ú
Þ lim 4 íê 2 4 ´
ï 8 ú (32) ï -k
h ®0 æ h /2 - 1 ö h ú 2 8 ý =2
ê ç h /2 ÷ ú
x ®0
ï ê x / 4 x ú x ï
êë è 3 ø2 úû ïêë 8 úû ï
î þ
4 -k -8
é æ 3h - 1 ö æ 3-h - 1 ö ù Þ 10 = 2 Þ2 = 2 Þk = 8
-k

ê 3çç ÷÷ - çç ÷÷ ú 2
h ê è h ø è -h ø ú Hence, answer is 8.00.
= lim 9×3 ê ú
h ®0
ê 1 æ 3h / 2 - 1 ö ú
30. (c) Given equation p(x) = x 2 - x - 2 = (x - 2)(x + 1) having a positive
çç ÷÷
êë 2 è h /2 ø úû root a, so a = 2.
é ù 1 - cos(p(x))
Now, lim
ê 3loge 3 - loge 3 ú x ®a+ x + a-4
= 9 ´1 ê ú
1 p(x) ö½
ê loge 3 ú ½ 2 sinæç
½
ë 2 û ÷½
½ è 2 ø½
= lim
æ 3 - 1ö ah - 1 x -2
= 9ç ÷ = 36 [Q lim 3h = 30 = 1 and lim = loge a] x ® 2+
è 1 /2 ø h ®0 h ®0 h
é æ (x - 2)(x + 1) ö ù
æ 3x 2 + 2 ö
1/x 2
ê sinçè 2
÷ú
ø
27. (c) Given limit lim ç 2 ÷ = P (let), having 1 form,
¥
= lim ê 2 ú [Qsin q > 0, if q ® 0+ ]
x ®0è 7 x + 2 ø x®2 ê+
x -2 ú
êë úû
1 æç 3 x 2 + 2 ö÷
lim -1
x ® 0 x 2 çè 7 x 2 + 2 ÷ é æ (x - 2)(x + 1) ö ù
P =e ê sinçè
ø
\ ÷
2 ø æ x + 1 öú æ3 ö 3
= lim ê 2 ´ç ÷ú = 2 ç ÷ =
{Q If lim (f (x))g (x) have indeterminant form 1¥ , then x ® 2+ ê (x - 2)(x + 1) è 2 øú è2 ø 2
x ®a lim g (x )(f (x ) - 1)
lim (f (x))g (x) = ex ® a }
êë 2 úû
x ®a
(1 - x) 1 / x - e -1
lim
1 æç 3 x 2 + 2 - 7 x 2 - 2 ö÷ 31. (1.00) The right hand limit lim+
x ® 0 x 2 çè 7x2 + 2
÷ x ®0 xa
Þ P =e ø
ì1 ü
1 æç 4 x 2 ö÷ -4 í log e (1 - x )ý
lim -
x ® 0 x 2 çè 7 x 2 + 2 ÷ø
lim
x ®0 7 x 2 + 2 eî x þ
- e -1
=e =e = lim a
x ®0 +
x
On applying limit, we get 1 æç
-x -
x2 x3
-
ö
- K÷
1 x çè 2 3 ÷
P = e -4 / 2 = e -2 = e ø
- e -1
e2 = lim
x ®0 + xa
Hence, option (c) is correct. æ x x2 x3 ö
1/x ç- - - -...... ÷
p ç 2 3 ÷
28. (d) Given lim æç tanæç + x ö÷ ö÷ [Having 1¥ form] e -1 . e è 4 ø
- e -1
x ®0 èè ø 4 ø = lim a
x ®0 +
x
limì1 é æx x2 x3
p ùü ö
= e í ê tanæç + x ö÷ - 1ú ý
x ®0 -ç +
ç
+ + ... ÷
÷
è2 3 4
îx ë è4 ø û þ e ø
-1
= e-1 lim a
lim
ì 1 1 + tan x ö ü x ®0 + x
= ex ®0 í æç - 1÷ ý
î x è 1 - tan x ø þ The above limit will be non-zero, if a =1. And at a = 1, the value of
tan A + tanB the limit is
As tan(A + B) =
1 - tan A tanB 1
= e -1 æç - ö÷ = -
1
lim æ
1 2 tan x ö è 2ø 2e
= ex ® 0 ç ´ ÷
è x 1 - tan x ø 32. (8) The limit
éQ lim tan x = 1ù 4 2 (sin3x + sin x)
= e2 êë x ®0 x úû lim

p æ2sin2x sin 3x + cos 5x ö - æ 2 + 2cos 2x + cos 3x ö
2 ç ÷ ç ÷
Hence, option (d) is correct. è 2 2ø è 2ø
29. (8.00) Given, 4 2 (2sin2x cos x)
= lim
ìï 1 æ x2 x2 x2 x 2 ö üï x ® 2sin2x sin 3x 5x 3x
p
lim í ç1 - cos -cos + cos cos ÷÷ ý = 2-k 2 + çcos - cos ö÷ - 2 (1 + cos 2x)
æ
x ®0 ïx8 ç 2 4 2 4 ø þï 2 è 2 2ø
î è
1 æ x2 ö æ x 2 ö -k 8 2 sin2x cos x
Þ lim 8 çç1 - cos ÷÷ çç1 - cos ÷÷ = 2 = lim
x ® 2sin2x sin 3x - 2sin2x sin x - 2 (2cos 2 x)
p
x ®0 x è 2 øè 4ø 2
2 2
JEE Main & Advanced Solutions (2021-18) 13

4 2 sin2x cos x (x - 1) (x - 3)
= lim = lim

p æ 3x x
- sin ö÷ - 2 cos 2 x x®2 (x - 4)
2 sin2x çsin
è 2 2ø
(2 - 1) (2 - 3) 1 ´ (- 1) 1
4 2 sin2x cos x = = =
= lim (2 - 4) (- 2) 2
x ® 2sin2x cos x sin x - 2 cos 2 x
p
2
2
1 + 1 + y4 - 2
4 2 sin2x 36. (a) Clearly, lim
= lim y ®0 y4
x ® 2sin2x sin x - 2 cos x
p
2
2
8 2 sin x 1+ 1+ y4 - 2 1 + 1+ y4 + 2
= lim = lim ´
x ® 4sin x sin x - 2
p y ®0 y4 1 + 1 + y4 + 2
2
2
8 2 16 [rationalising the numerator]
= = = 8.
4
- 2 4-2 (1 + 1 + y ) - 2 4 [Q (a + b) (a - b) = a2 - b2 ]
2 = lim
y ®0
x4 - 1 x 3 - k3 y 4 ( 1 + 1 + y 4 + 2)
33. (d) Given, lim = lim 2
x®1 x -1 x ® k x - k2
1 + y4 - 1 1 + y4 + 1
(x - 1) (x + 1) (x 2 + 1) = lim ´
Þ lim y ®0
y 4 ( 1 + 1 + y 4 + 2) 1 + y4 + 1
x®1 x -1
(x - k) (x 2 + k 2 + xk) [again, rationalising the numerator]
= lim
x ®k (x - k) (x + k) y4
= lim
3k 2 8 y ®0
y 4 ( 1 + 1 + y 4 + 2) ( 1 + y 4 + 1)
Þ 2´2= Þ k=
2k 3
x 2 - ax + b 1
34. (c) It is given that lim =5 …(i) = (by cancelling y 4 and then by direct substitution).
x®1 x -1 2 2 ´2
Since, limit exist and equal to 5 and denominator is zero at x = 1, 1
so numerator =
4 2
x 2 - ax + b should be zero at x = 1,
sin2 x é 0 formù
So 1 - a + b = 0 Þ a = 1 + b …(ii) 37. (a) Given limit is lim
x ®0 2 - 1 + cos x êë 0 úû
On putting the value of ‘a’ from Eq. (ii) in Eq. (i), we get
x 2 - (1 + b) x + b sin2 x éQ1 + cos x = 2cos 2 x ù
lim =5 = lim
x®1 x -1 x ®0 x êë 2 úû
2 - 2 cos
2
sin2 x sin2 x
(x 2 - x) - b(x - 1) = lim = lim
Þ lim =5 x ®0 x x ®0 x
x®1 x -1 2 æç1 - cos ö÷ 2 ´ 2sin2 æç ö÷
è 2ø è4ø
(x - 1) (x - b) éQ1 - cos x = 2sin2 x ù
Þ lim =5
x®1 x -1 êë 2 4 úû
2
Þ lim (x - b) = 5 x 16
x®1 = lim 2
= =4 2 [lim sin x = lim x]
x ®0
æ x ö 2 2 x ®0 x ®0
Þ 1-b=5 Þ b= -4 …(iii) 2 2ç ÷
è4ø
On putting value of ‘b’ from Eq. (iii) to
Eq. (ii), we get x + 2 sin x é 0 formù
38. (b) Let P = lim êë 0 úû
x ®0 2 2
a= -3 x + 2 sin x + 1 - sin x - x + 1
So, a+ b= - 7 On rationalization, we get
(x + 2 sin x)
35. (a) Given functions are f (x) = 5 - | x - 2 | P = lim 2
x ® 0 x + 2 sin x + 1 - sin2 x + x - 1
and g(x) = | x + 1 |, where x ÎR.
Clearly, maximum of f (x) occurred at x =2, so a = 2. ( x 2 + 2sin x + 1 + sin2 x - x + 1)
and minimum of g(x) occurred at x = - 1, so b = - 1. = lim ( x 2 + 2sin x + 1 + sin2 x - x + 1)
x ®0
Þ ab = - 2
x + 2 sin x
(x - 1) (x 2 - 5x + 6) ´ lim
Now, lim x ®0 x - sin2 x + 2 sin x + x
2
x ® - ab x 2 - 6x + 8 x + 2 sin x
= 2 ´ lim é 0 formù
(x - 1) (x - 3) (x - 2) x ®0 x 2 - sin2 x + 2 sin x + x êë 0 úû
= lim [Q ab = - 2]
x®2 (x - 4) (x -2) Now applying the L¢ Hopital’s rule, we get
14 Differential Calculus

1 + 2 cos x tan(p sin2 x) + x 2


P = 2 ´ lim = lim
2x - sin 2x +2 cos x + 1
x ®0
x ®0 +
x2
(1 + 2)
=2 [on applying limit] æ tan(p sin2 x) p sin2 x ö
0-0+ 2+ 1 = lim çç . + 1÷÷
+
x ®0 è p sin2 x x2 ø
3
=2´ =2
3 tan (p sin2 x) sin2 x
= p lim . lim +1
x + 2 sin x x ® 0+ p sin2 x x ® 0+ x2
Þ lim =2
x ®0
x + 2 sin x + 1 - sin2 x - x + 1
2 éQ lim tan x = 1 ù
ê x ®0 x ú
x ([x] + | x | ) sin [x] x ([x] - x) sin [x] =p+1 ê ú
39. (c) lim- = lim (Q | x | = - x,
| x| x ® 0- -x êand lim sin x = 1 ú
if x x<®0)0 êë x ®0 x úû
and LHL
x (- 1 - x) sin(- 1) tan(p sin2 x) + (| x | - sin(x [x]) 2
= lim (Q lim [x] = - 1) = lim
x ® 0- -x x ® 0-
x ®0 -
x2
- x (x + 1) sin(- 1)
= lim = lim (x + 1) sin(- 1) tan (p sin x) + (- x - sin(x (- 1)) 2
2
x ® 0- -x x ® 0- = lim
x ® 0- x2
= (0 + 1) sin (- 1) (by direct substitution)
é Q| x | = - x for x < 0 ù
= - sin1 [Q sin(- q) = - sin q] êand [x] = - 1 for - 1 < x < 0ú
ë û
40. (a) Given,
tan(p sin2 x) + (x + sin(- x)) 2
p = lim
(1 - | x | + sin|1 - x | ) sin æç [1 - x] ö÷ x ®0 -
x2
è2 ø
lim
x ® 1+ |1 - x| [1 - x] tan(p sin x) + (x - sin x) 2
2
= lim [Qsin(- q) = - sin q]
x ®0 -
x2
Put x = 1 + h, then
æ tan(p sin2 x) + x 2 + sin2 x - 2x sin x ö
x ® 1+ Þh ® 0+ = lim çç ÷÷
p
-
x ®0 è x2 ø
(1 - | x | + sin|1 - x| ) sin æç [1 - x] ö÷
è2 ø æ tan(p sin2 x) sin2 x 2x sin x ö
\ lim = lim çç + 1+ - ÷
x ® 1+ |1 - x| [1 - x] -
x ®0 è x 2
x2 x 2 ÷ø
p
(1 - | h + 1| + sin| - h| ) sin æç [- h] ö÷ æ tan (p sin2 x) p sin2 x sin2 x sin x ö
è2 ø = lim çç . + 1+ -2 ÷
= lim -
x ®0 è 2
p sin x x 2
x 2
x ÷ø
h ® 0+ | - h| [- h]
tan(p sin2 x) p sin2 x
p = lim . lim +
(1 - (h + 1) + sinh) sin æç [- h] ö÷ x ®0 - 2
p sin x x ®0 -
x2
è2 ø
= lim sin2 x sin x
h ® 0+ h [- h] 1 + lim - 2 lim
x ®0 -
x2 x ® 0- x
(Q | - h| = h and | h + 1 | = h + 1 as h > 0)
= p + 1+ 1-2= p
p
( - h + sinh) sin ç (- 1) ö÷
æ
Q RHL ¹ LHL
è2 ø
= lim \ Limit does not exist.
h ® 0+ h (- 1)
x cot4x x 1 tan2 2x
(Q [x] = - 1 for - 1 < x < 0 and h ® 0+ 42. (b) lim =lim . 2
x ®0 sin 2 x × cot2 2x x ®0 tan4x sin x 1
Þ -h ® 0- )
(- h + sinh) æ -pö 1 4x x 2 tan2 2x
= lim sin ç ÷ = lim .
h ® 0+ -h è 2 ø x ®0 4 (tan4x) sin2 x x2
(- h + sinh) sinh - h 1 4x æ x ö æ tan2x ö 4
2 2
= lim (-1) = lim = lim
h ®0 +
-h h ® 0+ h ç ÷ .ç ÷ .
x ®0 4 (tan4x) è sin x ø è 2x ø 1
sinh ö æh ö sinh ù 1 4 x tan x ù
= lim æç ÷ - lim ç ÷ = 1 - 1 = 0
é
lim
êQh ® = 1ú = . 1 .11. éêQ lim = 1 = lim =1
h ® 0 + è h ø h ® 0 + èh ø 0+ h 4 1 ë x ®0 sin x x ®0 x ú û
ë û
cot3 x - tan x
41. (d) Key Idea 43. (c) Given, limit = Lt
x ® p/ 4 p
cos æç x + ö÷
lim f (x) exist iff lim f (x) = lim f (x) è 4ø
x ®a x ® a+ x ® a- 1 - tan4 x 1 éQcot x = 1 ù
= Lt ´
x ® p/ 4 1 tan 3
x êë tan x úû
At x = 0, (cos x - sin x)
tan(p sin2 x) + (| x | - sin(x [x])) 2 2
RHL = lim (1 - tan2 x) 2 (1 + tan2 x)
x ®0 +
x2 = Lt ´

p cos x - sin x
tan3 x
tan(p sin x) + (x - sin(x ×0)) 2
2
é Q| x | = x for x > 0 ù 4
= lim êand [x] = 0 for 0 < x < 1ú
x ®0 +
x2 ë û
JEE Main & Advanced Solutions (2021-18) 15

cos 2 x - sin2 x 2 (sec2 x) 45. (b, c) It is given, that f : R ® R and


= Lt ´ [Q1 + tan2 x = sec2 x]

p cos x - sin x cos 2 x tan3 x f (h) - f (0)
4 Property 1 : lim exists and finite, and
h ®0 | h|
(cos x - sin x) (cos x + sin x) 2 sec4 x
= Lt ´ f (h) - f (0)

p (cos x - sin x) tan3 x Property 2 : lim exists and finite.
4 h ®0 h2
[Q (a2 - b2 ) = (a - b) (a + b)]
4
Option a,
2sec x sinh - sin0 1 sinh ö
= Lt (cos x + sin x) P2 : lim = lim æç
p
tan3 x ÷ = doesn’t exist.

4
h ®0 h2 h ®0 h è h ø

2 ( 2) 4 æ 1 1 ö Option b,
= ç + ÷ [on applying limit]
(1) 3 è 2 2ø h2 / 3 - 0
P1 : lim = lim h2 / 3 - 1/ 2 = lim h1/6 = 0
2 h ®0 | h| h ®0 h ®0
= 4 2 æç ö÷ = 8
è 2ø exists and finite.
lim p - 2 sin-1 x Option c,
44. (b) Let L = , then
x ® 1- 1- x | h| - 0
P1 : lim = lim | h| = 0, exists and finite.
h ®0 | h| h ® 0
lim p - 2sin-1 x p + 2sin-1 x
L= ´ [on rationalization] Option d,
x ® 1- 1- x p + 2sin-1 x h| h| - 0 | h| ì 1, if h ® 0+
P2 : lim = lim =í
lim p - 2sin-1 x 1 h ® 0 h2 h ®0 h
î-1, if h ® 0
-
= ´
x ® 1- 1- x p + 2sin-1 x f (h) - f (0)
So lim does not exist.
p
p - 2æç - cos -1 x ö÷
h ®0 h2
lim è2 ø 1
= ´ Hence, options (b) and (c) are correct.
x ® 1- 1- x p + 2sin-1 x 46. (c) Key idea Use property of greatest integer function [x] = x - { x }.
éQsin-1 x + cos -1 x = p ù æ 1 2 15 ö
êë 2 úû We have, lim x ç éê ùú + éê ùú + ¼+ éê ùú ÷
x ®0 + è ë x û ëxû ë x ûø
lim 2cos -1 x lim 1
= ´ We know, [x] = x - { x }
x ® 1- 1- x x ® 1- p + 2sin-1 x 1 ù 1 ì1 ü
\ é
1 lim 2cos -1 x é lim pù êë x úû = x - íî x ýþ
-1
= êQ x ® 1- sin x = ú é n ù = n - ìn ü
2 p x ®1
-
1- x ë 2û Similarly, êë x úû x íî x ýþ
Put x = cos q, then as x ® 1- , therefore q ® 0+
1 lim 2q æ1 1 2 2 15 15 ö
Now, L = \ Given limit = lim x ç - ìí üý + - ìí üý + … - ìí üý÷
2 p q ® 0 +
1 - cos q x ®0 + è x îx þ x îx þ x î x þø
1 lim 2q æ 1 2 15 ö
= éQ1 - cos q = 2sin2 q ù = lim (1 + 2 + 3+ ...+15) - x ç ìí üý + ìí üý + ... + ìí üý÷
2 p q ® 0 2 sinæ q ö
+ êë 2 úû x ®0 + èî x þ î x þ î x þø
ç ÷
è2 ø é ìn ü ù
q êQ0 £ í x ý < 1, therefore ú
2× æç ö÷ = 120 - 0 = 120 ê î þ ú
1 è2 ø nü nü ú
= × 2 lim ê ì ì
2 p q ®0 + q ê0 £ x í x ý < x Þ x lim x í ý = 0ú
sinæç ö÷ ë î þ ®0 + î x þ û
è2 ø
1 2 é lim q ù
= 2 2 = êQ x ® 0+ = 1ú
2 p p ë sin q û

Continuity and Differentiability


ì (x - 3)(x + 1)e (3x - 2) 2 ; x >3
ïï
(3x - 2) 2 æ1 + x ö
1. (c) f (x) = í - (x - 3)(x + 1)e ; -1£ x £3
ç ÷
ï (x - 3)(x + 1 ) e (3x - 2) 2 ; x < -1 lnç a ÷
ïî x 1 x 1 x
ç1 - ÷ lim æç ö÷ lnæç1 + ö÷ lim æç - ö÷ lnæç1 - ö÷
At x = - 1, let LHD be a, then its clear that RHD be - a. è b ø x ® 0 - èa ø è a ø x ® 0 - è b ø è b ø
lim = -
Similarly, at x = 3, if LHD is b, then RHD at x = 3 will be - b. x ® 0- x æ1 öx æ- 1 öx
ç ÷ ç ÷
So, f (x) is not differentiable at x = - 1, x = 3 èa ø è bø
æ 1 1ö
At, all other points f (x) will be differentiable. =ç + ÷
èa b ø
2. (a) f (x) is continuous at x = 0
f (0) = k
LHL at x = 0 = f(0) = RHL at x = 0
16 Differential Calculus

cos 2 x - sin2 x - 1 So, the graph of f (x) will be


lim
x ®0 + 2
x + 1 -1
-2sin2 x æ 2sin2 x ö
lim = lim - çç ÷÷ ( x 2 + 1 + 1 ) = -4 1/2
2
x ® 0+ 2 è x
+
x + 1 -1 x ® 0 ø
1 1 0 1/2 1 3/2 2 5/2 3
Þ + = -4=k
a b
æ 1 + 1 ö + æ4 ö = - 4 - 1 = - 5
ç ÷ ç ÷ f is continuous everywhere for 0 £ x £ 3.
èa b ø èk ø
1 3 5
But f is non-differentiable at x = , , and x = 1, 2
3. (14) For continuity 2 2 2
LHL at 0 = f(0) = RHL at 0 So, if set A denotes the points of discontinuity, then n(A) = 0.
p p And if set B denotes the points of non-differentiable, then
LHL = lim a sin (x - 1) = -a sin = - a …(i)
x ®0 - 2 2 n(B) = 5
tan2x - sin2x \ n(A) + n(B) = 0 + 5 = 5
RHL = lim
x ®0 + bx 3
p p
sin2x (1 - cos 2x) 6. (c) f : æç - , ö÷ ® R
= lim è 4 4ø
x ®0 + bx 3 .cos 2x
3a
sin2x ö (2sin2 x) 1 4 ì
= lim 2æç ÷ 2
. = ï(1 + | sin x | ) |sin x | , - p < x < 0
x ®0 + è 2x ø x b cos 2x b ïï 4
…(i) f (x) = í b x =0
ï cot 4 x
From Eqs. (i) and (ii), we get p
4
ï ecot 2 x , 0< x <
-a = ïî 4
b
Given f (x) is continuous at x = 0
Þ ab = -4
LHL at x = 0
Þ 10 - ab = 14 Put x = 0 - h
4. (a) We have, f (x) = x - [x] = { x } 3a
we get lim (1 - sin h) sinh
and g(x) = 1 - x + [x] = 1 - { x } h ®0
lim (1 - sinh - 1)
Y' 3a
1 eh ® 0 × = e3a
f(x) g(x) 3a - sinh
lim 3a
lim (1 + | sin x | ) | sin x | = ex ® 0 | sin x | |sin x |
= e3a
-
x ®0

X' X RHL at x = 0
–2 –1 D 1 2 cot 4 x
Y
lim ecot 2 x
x ® 0+
Again, h(x) = min[f (x), g(x) ], so graph of h (x) will be
Points of non-differentiability Put x = 0 + h
cot4h
we get lim cot2h
h(x) h ®0 e
1/2 cos 4h sin2h
lim ´
h ®0 ecos 2h sin4h
–2 –3/2 –1 –1/2 1/2 1 3/2 2
sin 2h
´ 2h
cos4h
´ 2h
From graph, it is clear that h(x) is continuous in [- 2, 2] but not cos2h sin 4h ´ 4h
-3 -1 1 3 lim e 4h = e1 / 2
differentiable at x = , - 1, , 0, , 1, in (- 2, 2). h ®0
2 2 2 2
As, f (x) is continuous at x = 0.
5. (5) f (x) = min {x - [x], 1 + [x] - x}
So, LHL = f(0) = RHL
f (x) = min ({ x }, 1 - { x }) 1
e3a = b = e 2
1
\ a = ,b = e
1 f(x)=x–[x] 6
1
\ 6a + b2 = 6 æç ö÷ + ( e) 2
f(x)=[x]–x+1 è6 ø
1 2 3 =1+ e
JEE Main & Advanced Solutions (2021-18) 17
tan(x - 2)
7. (b) Graph of max (sint : 0 £ t £ x) in x Î[0, p] x -2
lim f (x) = lim e =e
x ® 2+ x ® 2+
f(2) = m
1 As, f (x) is continuous.
So, LHL = f(2) = RHL
l
- =m =e
p/2 p m
and graph of 2 + cos x for x Î[p, ¥] l = -e2
m =e
3 l + m = e( -e + 1)
10. (a) f : [0, ¥) ® [0, ¥)
x
1 f (x) = ò [y] dy
0
p 2p 3p
Let x = I + f, 0 < f < 1
So, graph of 1 2 3 I I+f
ìmax [sint :0 £ t £ x], 0 £ x £ p f (x) = ò [y]dy + ò [y]dy + ò [y]dy + .... ò [y]dy + ò [y]dy
f (x) = í
î 2 + cos x , x>p 0 1 2 I-1 I

f (x) = 0 + 1 + 2 + K (I - 1) + I×f
Y
(I - 1) (I - 1 + 1) I(I -1)
= + I.f = + I×f
2 2
[x]([x] - 1)
f (x) = + [x]{ x }
2
[x]([x] - 1)
f (x) = + [x](x - [x])
X 2
p/2 p 2p I(I -1)
f ()I =
So, f (x) is differentiable everywhere in (0, ¥). 2
I(I - 1)
lim f (x) = lim + I(I + h - I)
x
x ®I - h ®0 2
8. (c) We have, f (x) = ò g (t) dt I(I - 1)
a =
2
So, f ¢ (x) = g(x) and f ¢¢(x) = g¢ (x) I(I - 1) (I - 2)
ìQf (x) = h (x) F (t) ×dt Þ f ¢ (x) ü lim f (x) = lim + (I -1) (I + h - I + 1)
ï òg (x) ï x ®I - h ®0 2
í ý (I - 1)(I - 2)
ïî= F [h(x)] ×h¢ (x) - F [g(x)] × g¢ (x) ïþ = + (I - 1)
2 (I - 1)I
Now, g¢ (x) g(x) = 0 =
2
Þ f ¢¢(x) f ¢ (x) = 0
\ f (x) is continuous and differentiable except at integer points.
If f (x) has five roots, then f ¢ (x) has atleast 4 roots and f ¢¢(x) has x
ì
atleast 3 roots. 11. (c) f (x) = ïí ò0(5 + | 1 - t | dt , x > 2
So, f ¢¢(x) ×f ¢ (x) = 0 has atleast 7 roots. ïî 5x + 1 ,x £ 2
x
Hence, the minimum number of roots of the equation
g¢ (x) g(x) = 0 is 7.
ò0 5 + |1 - t| dt
1 x
9. (a) = ò 5 + (1 - t)dt + ò 5 + (t - 1)dt
0 1
ì l| x 2 - 5x + 6| 1 x
ï , x <2 = ò (6 - t)dt + ò (4 + t)dt
2 0 1
ï m (5x - x - 6) 1 x
ï tan(x - 2) é t2 ù é t2 ù x2
ï = ê6t - ú + ê4t + ú = 1 + 4x +
We have f (x) = í e x - [x] , x >2 êë 2 úû0 êë 2 úû 1 2
ï m , x =2 2
ï ì x
Þ f (x) = í1 + 4x + 2 , x > 2
ï
ï
ï ïî 5x + 1, x £ 2
î
f (x) is continuous at x = 2.
At x = 2
\ lim f (x) = lim f (x) = f (2) LHL = lim (5x + 1) = 11
x ® 2- x ® 2+
l | (x - 3)(x - 2)| x ® 2-
\ lim f (x) = lim x
2
x ® 2- x ® 2 - m - (x - 3)(x - 2) RHL = lim æç1 + 4x + ö÷ = 1 + 8 + 2 = 11
l (x - 3)(x - 2) l x ® 2+ è 2ø
= lim =-
x ® 2 - m - (x - 3)(x - 2) m \ f(2) = 11
So, f (x) is continuous at x = 2.
18 Differential Calculus

ì4 + x, x > 2ü ì 0 , x < -2
f ¢(x) = í ý ï3 (1 + x) , -2 £ x < 0
î 5, x £ 2 þ ï
d f (x) = í2
Now, LHD at x = 2 is (5x + 1)| x = 2 = 5 3
ï (1 - x) , 0 £ x < 2
dx ï2
RHD at x = 2 is 4 + 2 = 6 î 0 , x >2
Here, LHD ¹ RHD ì 0 , x < -4
ï 3 (3 + x) , -4 £ x < -2
So, function is not differentiable at x = 2. ï
f (x + 2) = í 2
P ( x) 3
ì , x ¹2 ï (-1 - x) , -2 £ x < 0
12. (39) f (x) = ïí sin(x - 2) ï2
ïî 7, x =2 î 0 , x >4

Given, that P ¢ ¢ (x) is always a constant. ì 0 , x <0


ï 3 (x -1) , 0 £ x < 2
ÞP (x) is a 2 degree polynomial. ï
f (x - 2) = í 2
f (x) is continuous at x = 2 3
ï (-1 - x), 2 £ x < 4
P (x) ï2
lim =7
x ® 2 + sin(x - 2) î 0 , x >4
(x - 2) (ax + b) g(x) = f (x + 2) + f (x - 2)
Þ lim =7 [Q let P (x) = (x - 2) (ax + b)]
x ® 2+ sin(x - 2) ì3x + 6, -4 £ x £ 2
ï2
Þ 2a + b = 7 ...(i) ï 3x
ï- , -2 < x < 2
Now, P (x) = (x - 2) (ax + b) =í 2
P(3) = 9 (given) ï 3x
ï 2 - 6, 2 £ x £ 4
Þ 3a + b = 9 ...(ii) ï 0 ,
î | x| > 4
Subtracting Eq. (ii) from Eq. (i),
a=2 So, n = 0 and m = 4
From Eq. (i), b = 3 \ m+ n=4
Hence, P (x) = (x - 2) (2x + 3) 15. (b)
So, P(5) = (5 - 2) (2 ´ 5 + 3) = 3 ´ 13 = 39 ìsin x - ex ,x < 0
ï
ì x 3 -2 x
é 1 + 2xe ù f (x) = í a + [- x] ,0 < x < 1
loge ê , x ¹0
13. (a) f (x) = ïí (1 - cos 2x) 2 -x 2 ú
êë (1 - xe ) úû For continuity, ï 2x - b ,x ³ 1
î
ï a , x =0
î f (x) is continuous.
x3 So, lim f (0- ) = 0 - e0 = - 1
lim [loge (1 + 2xe-2 x )
x ®0 4sin4 x lim f (0+ ) = a - 1
- loge (1 - xe- x ) 2 ] = a (by expansion) …(i) Þ a- 1 = - 1 Þ a = 0
(2xe-2 x ) 2 lim f (1- ) = lim a + [- 1 - h] = a - 1
Q log(1 + 2xe-2 x ) = 2xe-2 x - +K h ®0
2 lim f (1+ ) = 2 (1 + h) - b = 2 - b
(xe- x ) 2
and log (1 - xe- x ) = - xe- x - -K \ 2-b=a-1 Þ b=2+ 1=3
2 \ a+ b=3
On putting the values in Eq. (i), we get
16. (1) Let f (x) = x 3 - 6 x 2 + 9x - 3
1 x æ x3 ö
lim æç × ö÷ çç 4 ÷÷ [2xe-2 x - 2(- xe- x )] f ¢ (x) = 3x 2 - 12x + 9
x ® 0 è 4 x ø è sin x ø
f ¢ (x) = 0 gives
[Q log (1 - xe- x ) 2 = 2 log (1 - xe- x )] 3x 2 - 12x + 9 = 0
4
1 x ö Þ 3(x - 1) (x - 3) = 0
= lim æç ö÷ æç ÷ (2xe
-2 x
+ 2xe- x )
x ®0 è 4x ø è sin x ø \ x = 1 or x = 3
4 Now, f(1) = 1 and f(3) = - 3
1 æ x ö
= lim æç ö÷ × ç ÷ × 2x × (e-2 x + e- x ) ì f (x), 0 £ x £ 1
x ® 0 è 4x ø è sin x ø
ï
g(x) = í 1, 1£ x £3
1
= æç ö÷ × (1) × (2) × (2) Þ a = 1 ï4 - x, 3 < x £ 4
î
è4 ø
ì æ 1 -| x | ö g(x) is continuous.
3 if | x| £ 2
14. (4) f (x) = ïí çè 2 ÷ø ì3(x - 1) (x - 3), 0 £ x £ 1
0 if | x| > 2 ï
îï g¢ (x) = í 0, 1£ x <3
g(x) = f (x + 2) - f (x - 2) ï - 1, 3 < x £4
î
g(x) is non-differentiable at x = 3.
So, the number of points in (0, 4) where g(x) is not differentiable is 1.
JEE Main & Advanced Solutions (2021-18) 19

17. (d) æ x + bx 3 - x ö
Again, lim f (x) = lim ç ÷
ì 1 , | x| ³ 1 x ®0 +
x ®0 + ç
bx5 /2 ÷
ï è ø
Given, f (x) = í | x |
ïax 2 + b , | x | < 1 ( x + bx 3 - x ) ( x + bx 3 + x)
î = lim
x ®0 + 5 /2 3
ì 1 , bx ( x + bx + x)
ï x £ - 1 or x ³ 1
Þ f (x) = í | x | (x + bx 3 - x)
ïax 2 + b , = lim
-1 < x < 1
î x ® 0+ bx5 / 2 ( x + bx 3 + x)
ì -1 , x £ -1
ï x x
= lim
ï 2 x ® 0+ x ( 1 + bx 2 + 1)
Þ f (x) = íax + b, -1 < x < 1
ï 1 1
ï x, x ³1 Þ lim f (x) = …(iv)
î x ® 0+ 2
Given, f (x) is differentiable at every point of domain. From Eq. (i), (ii), (iii) and (iv)
ì 1 , 1 a+ 1
x < -1 =b= +1
ï x2 2 2
ï
\ f ¢ (x) = í2ax , -1 < x < 1 1
Þ b = ,a = - 2
ï -1 2
ï x2 , x >1
î -3
\ a+ b=
Q f (x) is differentiatble at x = 1 2
\ (LHD at x = 1) = (RHD at x = 1) 19. (3) Method 1
Þ f ¢ (1 -) = f ¢(1 + ) Given, f (x + y) = f (x) × f (y) " x, y ÎR
1
Þ 2a = - 1 Þ a = - \ f (x) = ax
2
As, we know that, a function is differentiable at x = a, if it is Þ f ¢ (x) = ax ×l og(a)
continuous at x = a. Now, f ¢ (0) = l og(a)
Hence, f (x) is also continuous at x = 1. Þ 3 = l og(a)
i.e., (LHL at x = 1) = (RHL at x = 1) =f(1)
Þ a = e3
Þ a+ b=1
æ 1ö \ f (x) = (e3 ) x = e3 x
Þ ç- ÷ + b = 1 \ f (h) = e3h
è 2ø
3 f (h) - 1 ö æ e3h - 1 ö
Þ b= Now, lim æç ÷ = lim çç ÷
2 h ® 0è h ø h ® 0 è h ÷ø
1 3 æ e3h - 1 ö
Hence, a = - , b =
2 2 = lim çç ´ 3÷÷
h ® 0 è 3h ø
Note You can also (or apply) continuity and differentiability at
x = - 1. =3´1=3
18. (d) Method (2)
1 0
ìsin(a + 1) x + sin2x Let L = lim (f (h) - 1) æç formö÷
, x <0 h ®0h è0 ø
ï
ïï 2x
f (x + y) = f (x) + f (y)
Given, f (x) = í b, x =0
ï Put x = y =0
3
ï x + bx - x , x > 0 \ f (0) = f (0) × f (0)
ïî
bx5 / 2 Þ [f (0)] 2 = f (0)
Q f (x) is continuous at x = 0. Þ [f (0)] 2 - f (0) = 0
\ lim f (x) = lim f (x) = f (0) …(i) Þ f (0) [f (0) - 1] = 0
x ®0 - x ®0 +
Q f (0) = b …(ii) Þ f(0) = 0, f(0) = 1
sin(a + 1) x + sin2x ö Rejected because f (x) ¹ 0, " x ÎR
Now, lim f (x) = lim æç ÷
x ® 0- x ® 0- è 2x ø \ f(0) = 1
sin(a + 1) x sin 2x Using L-Hospital Rule,
Þ lim f (x) = lim æç + ö
÷
x ® 0- x ® 0- è 2x 2x ø f ¢ (h) - 0
L = lim = f ¢ (0) = 3
æ sin(a + 1) x æ a + 1 ö sin2x ö h ®0 1
= lim ç ´ç ÷+ ÷ cos(sin x) - cos x
x ® 0 - è (a + 1) x è 2 ø 2x ø 20. (6) f (x) =
a+ 1 x4
= +1 … (iii) As, f (x) is continuous everywhere, so f (0) = lim f (x)
2 cos(sin x) - cos x x ®0
= lim 4
x ®0 x
20 Differential Calculus

On expanding the numerator and only identifying the coefficient ì x + 2, x < 0


22. (b) f (x) = í 2
of x 4 will give us the required limit. î x , x ³0
3
æ sin2 x sin4 x ö ïì x , x <1
cos(sin x) = çç1 - + ÷ g(x) = í
è 2 24 ÷ø ïî3x - 2, x ³ 1
2 ìg(x) + 2, g(x) < 0
1æ x3 ö 1 f [g(x)] = í 2
=1- çç x - ÷÷ + (x) 4 î g (x), g(x) ³ 0
2è 6ø 24
Y –2
3x
1æ x4 ö x4 x2 x4 x4 =
= 1 - çç x 2 - ÷÷ + =1- + + g(
x)
2è 3 ø 24 2 6 24

x2 x4 X¢ X
cos x = 1 - + O
x=1
2 24

)= 3
x
cos(sin x) - cos x

g(x
\
x4 Y¢
æ x2 x4 x4 ö æ x2 x4 ö 3
çç1 - + + ÷ - ç1 - + ÷ When g(x) < 0 Þg(x) = x , x < 0
è 2 6 24 ÷ø çè 2 24 ÷ø 1
= = ìï x 3 , 0 £ x < 1
x4 6 When g(x) ³ 0 Þg(x) = í
îï3x - 2, x ³ 1
1 1
\ f (0) =
= ì x 3 + 2, x <0
6 k ï
f [g(x)] = í x6 , 0£ x <1
Hence, k = 6.
ï(3x - 2) 2 , x ³1
21. (b) Method (I) î
ìé æ 1 öù As, polynomial function is continuous everywhere in its domain.
ï 2 - sin ç ÷ ú | x | , x ¹ 0 So, f [g(x)] will be continuous everywhere at x < 0, 0 < x < 1 and
Given, f (x) = íêë è x øû
ïî 0, x =0 x > 1. We will check the behaviour of fog(x) only at boundary
points which is x = 0 and x = 1.
ì
ï- [2 - sin(1 / x)] x , x < 0 At x = 0, lim (x 3 + 2) = 2
x ® 0-
ï
Þf (x) = í 0 , x =0 lim x6 = 0
+
x ®0
ï é æ 1 öù Clearly, LHL ¹ RHL at x = 0
ï ê2 - sinçè ÷ø ú x , x > 0 So, fog(x) is discontinuous at x = 0.
î ë x û
At x = 1, lim x6 = 1
ì æ 1ö é 1 æ 1 öù x ® 1-
ïï - çè2 - sin x ÷ø - x ê - cos x çè - 2 ÷ø ú, x < 0 lim (3x - 2) 2 = 1
Þf ¢ (x) = í ë x û x ® 1+

ï æç2 - sin 1 ö÷ + x é - cos 1 æç - 1 ö÷ ù, x > 0


xø ê x è x 2 ø úû Also f(1) = 1
îï è ë
fog(x) is continuous at x = 1
ì- 2 + sin 1 - 1 cos 1 , x < 0
ï x x x Derivative test at x = 1,
Þf ¢ (x) = í f (1) - f (1 - h)
1 1 1 LHD = lim
ï 2 - sin + cos , x >0
î x x x h ®0 h
Here, f ¢ (x) is an oscillating function which is non-monotonic in 1 - (1 - h)6
= lim
(- ¥, 0) È (0, ¥). h ®0 h
Method (II) = lim 6(1 - h)5 = 6
h ®0
ì æ 1ö f (1 + h) - f (1)
RHD = lim
ï- çè2 - sin x ÷ø x , x < 0 h ®0 h
ï
Q f (x) = í 0 , x =0 [3(1 + h) - 2] 2 - 1
ïæ 1ö = lim
ï çè2 - sin ÷ x , x >0 h ®0 h
î xø
= lim 2 [3(1 + h) - 2] ×3 = 6
h ®0
From above we observe that, f (x) is continuous and \ fog(x) is continuous and differentiable at x = 1.
1 2 2
f æç ö÷ = f æç ö÷ = \ fog(x) is discontinuous and non-differentiable at x = 0.
èpø èpø p
So, number of points of non-differentiability of fog(x) is 1.
So, f (x) is non-monotonic in (0, ¥).
23. (1)
Further, lim (f) ® ¥
x ® -¥ ì x + a, x < 0
and lim f (x) ® ¥ f (x) = í
x®¥ î | x - 1|, x ³ 0
and f(0) = 0 ì x + 1, x <0
g(x) = í 2
Hence, f (x) is non-monotonic on (- ¥, 0) È (0, ¥). î(x - 1) + b, x ³ 0
JEE Main & Advanced Solutions (2021-18) 21

ì f (x) + 1, f (x) < 0 Applying L-Hospital Rule,


g[f (x)] = í 2
î[f (x) - 1] + b, f (x) ³ 0 p (- 1) (- 2x)
= lim
2 x ® 0 1 - (1 - x 2 ) 2
f (x) < 0
Case I x + a < 0 and x < 0 Þ x < - a p x
= ×2 lim
Case II | x - 1 | < 0 and x ³ 0 Þ Not possible 2 x ® 0 2x 2 - x 4
f (x) ³ 0 1
= p lim
Case I x + a ³ 0 and x < 0 Þ x Î[- a, 0) x®0
2 - x2
Case II | x - 1 | ³ 0 and x ³ 0 Þ x ³ 0 p
ì x + a + 1, x < 0 and f (x) <0 =
ï | x - 1 | + 1, 2
ï x ³ 0 and f (x) <0 p p
g[f (x)] = í 2 LHL = and RHL =
ï (x + a - 1) + b, x < 0 and f (x) ³0 4 2
ïî ( | x - 1 | - 1) 2 + b, x ³ 0 and f (x) ³0 Hence, LHL ¹ RHL
ì x + a + 1, x < -a So, the function will be discontinuous for every value of a ÎR.
ï
g[f (x)] = í (x + a - 1) 2 + b, -a£ x <0 \ No such a exist.
ï( | x - 1 | - 1) 2 + b, x ³0
î 25. (a) We have, f : S ® S, S = (0, ¥)
This is continuous function. f (x + 1) = x ×f (x)
Since, g[f (x)] is continuous for all x ÎR g :S ® R
So, g(f (x)) will be continuous at x = -a and x = 0 g(x) = loge f (x)
Now, at x = -a To find | g¢¢(5) - g¢¢(1) |
LHL=RHL = value of function Þ g(x + 1) = loge f (x + 1)
Þ 1 = 1 + b = 1 + b Þb = 0 Þ g(x + 1) = log[x ×f (x)]
At x = 0 Þ g(x + 1) = log x + logf (x)
LHL =RHL = value of function Þ g(x + 1) = log x + g(x)
Þ (a - 1) 2 + b = b Þ g(x + 1) - g(x) = log x
Þ (a- 1) 2 = 0 Þ g¢ (x + 1) - g¢ (x) = 1 / x
Þ a=1 -1
Þ g¢¢(x + 1) - g¢¢(x) = 2
Hence, a + b = 1 x
24. (c) x = 1, g¢¢ (2) - g¢¢ (1) = - 1 … (i)
ìcos - 1 (1 - { x } 2 ) sin- 1 (1 - { x }) x = 2, g¢¢ (3) - g¢¢ (2) = - 1 /4 … (ii)
ï , x ¹0
Given, f (x) = í {x} - {x}3 x = 3, g¢¢ (4) - g¢¢ (3) = - 1 /9 … (iii)
ï , x =0
î a x = 4, g¢¢ (5) - g¢¢ (4) = - 1 /16 … (iv)
{ x } = x - [x] Adding Eqs. (i), (ii), (iii) and (iv),
So, when x ® 0+ 1 1 1
g¢¢ (5) - g¢¢ (1) = - 1 - - -
Þ {x} = x - 0 = x 4 9 16
144 + 36 + 16 + 9 ö - 205
And, when x ® 0- = - æç ÷=
Þ {x} = x + 1
è 144 ø 144
205
LHL = lim f (x) So, | g¢¢ (5) - g¢¢ (1) | =
x ® 0- 144

cos - 1 [1 - (1 + x) 2 ] sin- 1 [1 - (1 + x)] 26. (b) Given, f (x) is continuous on R.


= lim
x®0 (1 + x) - (1 + x) 3 If f (x) is continuous, then
cos - 1 (- x 2 - 2x) sin- 1 (- x) f is continuous at x =`1
= lim Þ lim f (x) = f (1) = lim f (x)
x®0 (1 + x) (1 + 1 + x) (1 - 1 - x) x ® 1- x ® 1+
cos - 1 (- x 2 - 2x) Þ | a + 1 + b| = sin p = 0
= lim
x ® 0 (1 + x) (x + 2) Þ a + b = -1 …(i)
cos - 1 (0) p Also, f is continuous at x = -1
= = Þ lim f (x) = f (-1) = lim f (x)
12
. 4 x ® -1 - x ® -1 +
RHL = lim f (x)
x ® 0+ -p
cos - 1 (1 - x 2 ) sin- 1 (1 - x) Þ2sinæç (-1) ö÷ =| a - 1 + b|
= lim è 2 ø
x®0 x (1 - x) (1 + x)
Þ 2 =| a + b - 1| …(ii)
p cos - 1 (1 - x 2 ) Eq. (ii) is satisfied.
= lim
2 x®0 x \ a + b = -1
22 Differential Calculus

27. (d) Given, | f (x) - f (y) | £ | x - y | 2 ì 2x + 2 ; x < -2


ï -1
| f (x) - f (y)| ïï- 2x - 6 ; -2< x <
Þ £|x - y| 2
| x - y| Now, f ¢ (x) = í
-1
ï- 2x - 2 ; < x <1
Now, taking the limit, ï 2
f (x) - f (y) ïî 2x ; x >1
lim £ lim | x - y |
x®y x-y x®y Now, f ¢ (x) at 1, - 2 and - 1 /2.
Þ| f ¢ (y) | £ 0 [using the definition of f ¢ (y)] For x = 1,
Þf ¢ (y) = 0 [since, modulus value can never be less than 0] f ¢ (x) = 2x = 2 ´ 1 = 2
On integrating it, we get and - 2x - 2 = - (2 ´ 1) - 2 = - 4
both are not equal.
f (y) = c (constant)
\ Non-differentiable at x = 1
Given, f(0) = 1 gives c = 1
Similarly, for x = - 2, f ¢ (x) = 2x + 2 = 2 ´ (- 2) + 2 = - 2
\ f (y) = 1" y ÎR
and - 2x - 6 = - 2 ´ (- 2) - 6 = - 2 both are equal.
From given options, f (x) > 0 " x ÎR is satisfied only.
\ Differentiable at x = - 2
Hence, answer will be option (d).
and for x = - 1 /2, f ¢ (x) = - 2x - 6
xf (a) - af (x) -1 -1
28. (b) lim = - 2 ´ æç ö÷ - 6 = - 5 and - 2x - 2 = - 2 ´ æç ö÷ - 2 = - 1 both
x®a x -a è2ø è2ø
are not equal.
xf (a) - af (x) + af (a) - af (a)
= lim \ Non-differentiable at x = - 1 /2
x ®a x -a
\ The number of points at which f (x) is non-differentiable is 2.
(x - a)f (a) - a[f (x) - f (a)]
= lim 2x - 1 ö
x ®a x -a 31. (d) Given, f (x) = [x - 1]cos æç ÷ p where []
× is greatest integer
function and f : R ® R è 2 ø
(x - a)f (a) f (x) - f (a)
= lim - a lim Q It is a greatest integer function then we need to check its
x ®a x -a x ®a x -a continuity at x ÎI except these it is continuous.
= f (a) - af ¢ (a) Let x = n where n ÎI
2x - 1 ö
= 4 - a(2) [Given, f (a) = 4, f ¢(a) = 2] Then, LHL = lim [x - 1]cos æç ÷p
x ® n- è 2 ø
= 4 - 2a
2n - 1 ö
29. (5) For this particular problem, try to draw graph in the region (-3,3) , = (n - 2) cos æç ÷ p =0
è 2 ø
it will be as follows,
2x - 1 ö æ 2n - 1 ö p = 0
RHL = lim [x - 1]cos æç ÷ p = (n - 2) cos ç ÷
x ® n+ è 2 ø è 2 ø
2
and f (n) = 0.
1 Here, lim f (x) = lim f (x) = f (n)
x ®n - x ®n +
–Ö2 Ö2
\ It is continuous at every integers.
–3 –2 –1 1 2 3
Therefore, the given function is continuous for all real x.
32. (d) Given function
ìaex + be- x , - 1 £ x < 1
Thus, points of discontinuity will be at -2, 2 because the curve ï
f (x) = í cx 2 , 1£ x £3
breaks at these points and at -1, 0, 1 because curve has sharp
ï ax 2 + 2cx, 3 < x £ 4
points. î
\ Point of discontinuity are -2, - 1, 0, 1, 2 i.e. 5 points. Since, given function is continuous for some a,b,c ÎR, So
30. (2) Given, lim f (x) = lim f (x)
x ® 1- x ® 1+
f (x) = | 2x + 1 | - 3 | x + 2 | + | x 2 + x - 2 |
Þ ae + be = c -1
… (i)
= | 2x + 1 | - 3 | x + 2 | + | x + 2 | ´ | x - 1 |
-1 and lim f (x) = lim f (x)
Here, critical points are x = , - 2, 1 x ® 3- x ® 3+
2 Þ 2
c(3) = 9a + 6c
ì x 2 + 2x + 3 ; x < -2
ï 2 Þ 3c = 9a Þc = 3a … (ii)
-1
ï - x - 6x - 5 ; - 2 < x < Now, f ¢ (0) + f ¢ (2) = a - b + 4c = e [given] … (iii)
ï 2
\ f (x) = í From Eqs. (i), (ii) and (iii), we get
-1
ï- x 2 - 2x - 3 ; < x <1 a - (3a - ae)e + 12a = e
ï 2
ïî x 2 - 7 ; x >1 Þ (13 - 3e + e2 )a = e
e
Þ a= 2
e - 3e + 13
JEE Main & Advanced Solutions (2021-18) 23
2
33. (10) Given functional identity Lim 2tf(x) - x 2 2f (t)f ¢ (t)
t®x =0
f (x + y) = f (x) + f (y) + xy 2 + x 2 y 1
Differentiate w.r.t. x, we get Þ 2xf 2 (x) - 2x 2f (x)f ¢ (x) = 0
f ¢ (x + y) = f ¢ (x) + y 2 + 2xy [taking y as a constant] Since, x and f (x) Î(0, ¥), so
d (f (x)) dx
Now, put y = - x, then f (x) = xf ¢ (x) Þ ò =ò
f (x) x
f ¢ (0) = f ¢ (x) + x 2 - 2x 2 = f ¢ (x) - x 2 …(i)
f (x) Þ loge f (x) = loge (Cx)
As, lim = 1 Þf(0) = 0 and f¢ (0) = 1 …(ii)
x ®0 x Þ f (x) = Cx.
From Eqs. (i) and (ii), we get Q f (1) = e Þ C = e; \ f (x) = ex
f ¢ (x) = 1 + x 2 1
Now, as f (x) = 1 Þ ex = 1 Þ x = .
\ f¢ (3) = 1 + 9 = 10 e
Hence, answer is 10.00. 37. (8.00)
x
34. (a) For twice differentiable function on (1, 6), it is given that The given function, f (x) = x × éê ùú, for -10 < x < 10 may be
f ¢¢(x) ³ 4 ë2 û
x
x x discontinuous at the points, where is an integer. So, possible
Þ f ¢¢(x) dx ³ ò 4dx 2
ò points of discontinuity are
2 2

Þ f ¢ (x) - f ¢ (2) ³ 4(x - 2) x = 0, ±2, ± 4, ± 6, ± 8


Þ f ¢ (x) ³ 4x - 3 …(i) But at x =0
[Q f¢ (2) = 5] and f ¢ (x) ³ 1 [given] lim f (x) = 0 and lim f (x) = 0
x x x ®0 + x ®0 -
Þ ò f ¢ (x)dx ³ ò dx Þf (x) - f (2) ³ (x - 2) \ x = 0 is the point of continuity.
2 2 So, the given function f (x) is discontinuous at
Þ f (x) ³ x + 6 …(ii) [Q f(2) = 8] x = ± 2, ± 4, ± 6, ± 8
Q x Î(1, 6), so according to given option put x = 5, in inequalities (i) Therefore, the number of points of discontinuity of f is equal to
and (ii), we get eight.
\ f¢ (5) ³ 17 and f(5) ³ 11 38. (b) The given function,
Þ f (5) + f ¢ (5) ³ 28 2
ïìk (x - p) - 1, x £ p
Hence, option (a) is correct. f (x) = í 1
ïî k 2 cos x, x>p
35. (d)
ì p + tan–1 x, | x | £ 1| Is twice differentiable, so f (x) must be continuous at x = p, so
ï
Given function, f (x) = í 4 lim f (x) = f (p)
1 x ® p+
ï (| x |–1), | x | > 1|
î 2 Þ -k 2 = - 1 Þk 2 = 1 …(i)
ì p + tan–1 x, x Î[–1, 1] ì2k (x - p), x < p
ï4 Now, f ¢ (x) = í 1
ïï 1 î -k 2 sin x, x > p
Þ f (x) = í (x –1), x > 1 at x = 1 ì 2k 1, x<p
ï 2 and f ¢ ¢ (x) = í
ï 1 (–x –1), x < –1 -
î 2k cos x, x>p
ïî 2
\ lim f ¢ ¢ (x) = lim f ¢ ¢ (x) Þ k 2 = 2k 1
p + x ® p+ x ® p-
Q f(1) = , f(1 ) = 0
2 k2 1
Þ k1 =
= [from Eq. (i)]
Þf is discontinuous at x = 1 , so it is non-differentiable at x = 1 2 2
1
Now, at x = –1 \ The ordered pair (k 1, k 2 ) = æç , 1ö÷
p p è2 ø
f(–1) = – = 0
4 4 39. (c) Equation of the given curve is x 4e y + 2 y + 1 = 3
f(–1+ ) = 0 and f(–1– ) = 0 On differentiating w.r.t. ‘x’, we get
Þ f is continuous at x = –1 dy 1 dy
1 1 ey (4x 3 ) + x 4ey + =0
Q f' (–1+ ) = and f' (–1– ) = – dx y + 1 dx
2 2 \ At point P(1, 0),
\ f is non-differentiable at x = -1 dy 1 dy
e0 (4 ´ 1) + 1 .e0 + =0
\ f is continuous on R –{1} and differentiable on R –{–1, 1}. dx 0 + 1 dx
36. (b) dy
Þ = –2
2 2 dx P
f (x) – x 2f 2 (t)
Lim t
Since, t®x =0 \ Equation of tangent at point P(10, ) is
t –x
On applying L' Hospital Rule, we get y = –2(x –1) Þ2x + y = 2 …(i)
From the option point (–26
, ) contain by the tangent (i).
24 Differential Calculus

40. (a) The function f : R ® R is defined by f (x) = max{ x, x 2 } f (x)


Þ lim =2 …(i)
x ®0 x3
Y
y=x2 and it is given that f (x) is a polynomial of degree 3 and relation (i)
y=x will be true if lowest degree of polynomial f (x) is 3.
Now, let f (x) = ax5 + bx 4 + cx 3
(1, 1) f (x)
Q lim =2 Þ c =2
x ®0 x 3

X \ f (x) = ax5 + bx 4 + 2x 3
O
Þ f ¢ (x) = 5ax 4 + 4bx 3 + 6x 2
Q x ± 1 are critical points of f (x).
\ f¢ (1) = 0 and f¢ (-1) = 0
ìx 2 , x <0 ì2x, x <0
ï ï \ 5a + 4b + 6 = 0 …(ii)
= í x, 0 £ x < 1 Þ f ¢ (x) = í 1, 0 £ x < 1
ïx 2 , ï2x, 5a - 4b + 6 = 0 …(iii)
x ³1 î x ³1
î 6
From Eqs. (ii) and (iii), we get b = 0 and a = -
Q f¢ (0- ) = 0, but f¢ (0+ ) = 1, so f is not differentiable at x = 0. 5
Similarly, as f¢ (1- ) = 1, but f¢ (1+ ) = 2 Þf is not differentiable at x = 1. 6
\ f (x) = - x5 + 2x 3
So, the required set S = {0, 1}. 5
41 (b) Given function f: R ® R with f (0) = f (1) = f ¢ (0) = 0. Þ f ¢ (x) = - 6x 4 + 6x 2
So, by Rolle’s theorem, for some c Î(0, 1) f ¢ (c) = 0. and f ¢ ¢ (x) = - 24x 3 + 12x
And as f¢ (0) = 0 and function ‘f’ is twice differential. Q f¢ ¢ (1) = - 12 < 0 Þ x = 1 is point of maxima and
So, again for some x Î(0, 1). Q f¢ ¢ (-1) = 12 > 0 Þ x = -1 is point of minima.
f ¢¢(x) = 0 [By Rolle’s theorem] 45. (c) It is given that, the function
42. (3) Given function f (x) = |2 - | x - 3| | ì sin(a + 2) x + sin x
|2 + (x - 3)| , x < 3 | x - 1| , x < 3 ï x ; x <0
= = ï
| 2 - (x - 3)| . x ³ 3 |5 - x | , x ³ 3 f (x) = í b ; x =0
ï 2 1/ 3 1/ 3
1 - x, x <1 ï ( x + 3x ) - x ; x >0
î 4 / 3
x - 1, 1 £ x < 3 x
= is continuous at x = 0, then
5 - x, 3 £ x < 5
x - 5, x ³5 lim f (x) = f (0) = lim f (x)
x ®0 - x ®0 +
Q Function f (x) is not differentiable at x = 1, 3, 5
\ b = lim f (x)
\ S = {1, 3, 5} x ®0 +

Now åf (f (x)) = f (f (1)) + f (f (3)) + f (f (5)) ( x + 3x 2 ) 1 / 3 - x 1 / 3


x ÎS
= lim
= f (1 - 1) + f (5 - 3) + f (5 - 5) x ®0 +
x 4 /3
= f (0) + f (2) + f (0) (1 + 3x) 1/ 3 - 1
= lim
= (1 - 0) + (2 - 1) + (1 - 0) x ®0 +
x
= 1 + 1 + 1= 3 1
1 1 1 + (3x) + (higher degree terms in x) - 1
43. (5) The given function ‘f ’ defined on æç - , ö÷ by = lim 3 = lim
è 3 3ø x ®0 + x x ®0 +
ì1 æ 1 + 3x ö
ï log ç ÷, where x ¹ 0 [1 + (higher degree terms of x)]
f (x) = í x e è 1 - 2x ø = 1 (on applying limit) Þ b = 1
ï k, where x = 0
î Now, f (0) = lim f (x)
is continuous, then x ®0 -
1 æ 1 + 3x ö sin(a + 2) x + sin x [Qf (0) = b = 1]
k = lim loge ç ÷ Þ lim =1
x ®0 x è 1 - 2x ø x ®0 - x
a+ 3 ö ax
log (1 + 3x) loge (1 - 2x) ù
= lim éê e - 2sinæç x ÷ cos
úû è 2 ø 2 =1
x ®0 ë x x Þ lim
x ®0 - x
= 3 - (-2) = 5 Þ k = 5 é æa + 3 ö ù
ê sinçè 2 ÷ø x æ a + 3 ö ax ú
f ( x) ö Þ lim ê2 ´ ç ÷ ×cos ú = 1
44. (d) Given that lim æç2 + ÷ =4 x ®0 - ê æa + 3 ö x è 2 ø 2ú
x ®0è x3 ø êë ç
è 2 ø
÷ úû
f (x) Þ a+ 3=1 Þ a= -2
Þ 2 + lim =4
x ®0 x3 So, a + 2b = - 2 + 2(1) = 0
JEE Main & Advanced Solutions (2021-18) 25

46. (a) It is given that fo g(x) = x, then gof (x) = x f (x) g(x) f (x) g(x)
Þ lim = lim {Qf (1) = 0}
Þ g¢ (f (x))f ¢ (x) = 1 x ® 1- x -1 x ® 1+ x - 1

On putting x = b, we get f (1 - h)g(1 - h) f (1 + h)g(1 + h)


Þ lim = lim
1 h ®0 -h h ® 0 h
g¢ (f (b))f ¢ (b) = 1 Þ f ¢ (b) =
g¢ (f (b)) [(1 - h) 2 (-h) + (-h) sin(1 - h)]g(1 - h)
gof (x) = x Þ lim
Þ g f (b) = b (on putting x = b) h ®0 -h
Þ g(f (b)) = g(a) (Qg(a) = b) [(1 + h) 2 (h) + (h) sin(1 + h)] g(1 + h)
= lim
Þ f (b) = a h ®0 h
1 1 Þ lim [(1 - h) 2 + sin(1 - h)]g (1 - h) = lim
\ f ¢ (b) = = (given)
g¢ (a) 5 h ®0 h ®0
[(1 + h) 2 + sin(1 + h)]g (1 + h)
Hence, option (a) is correct. It does not mean that g(x) is continuous or differentiable at x = 1.
47. (a) As we know that x - 1 < [x] £ x, But if g is differentiable at x = 1, then it must be continuous at x = 1
and so fg is differentiable at x = 1.
where [x] denote the greatest integer £ x.
4 4 4 49. (4) The given functions f : (- 1, 1) ¾® R and g : (- 1, 1) ¾® (- 1, 1) be
So, - 1 < éê ùú £ defined by
x ëxû x
f (x) = | 2x - 1 | + | 2x + 1 | and g(x) = x - [x].
4 4 4
\ lim x æç - 1ö÷ < lim x éê ùú £ lim x As, we know the composite function (fog) (x) is discontinuous at
x ®0 è x ø x ®0 ë x û x ®0 x
the points, where g(x) is discontinuous for given domain. And,
4 since g(x) is discontinuous at x = 0lies in interval (- 1, 1), so value of
Þ lim (4 - x) < lim x éê ùú £ lim 4
x ®0 x ®0 ë x û x ®0 c = 1.
And, since (fog) (x) is not differentiable at the point where g(x) is
So, according to Squeeze theorem, we have not differentiable as well as at those points also where g(x)
4 attains the values so that f (g(x)) is non-differentiable.
lim x éê ùú = 4 = A (given)
x ®0 ë xû Since g(x) is not continuous at x = 0 Î(- 1, 1) so fog(x) is not
Now, the function f (x) = [x 2 ]sin(px) is continuous at every differentiable and as f (x) = | 2x - 1 | + | 2x + 1 | is not differentiable
integral value of x, so according to given options A + 1 = 5 Î / at x = - 1 /2 and 1/2, so (fog) (x) is not differentiable for those x, for
which g(x) = - 1 /2 or 1/2.
Integer.
1
So f (x) is discontinuous when x = 5 = A + 1. But g(x) ³ 0, so g(x) can be 1/2 only and for x = - 1 /2 and 1/2, g(x) = .
2
Hence, option (a) is correct. So, (fog) (x) is not differentiable at x = - 1 /2, 0, 1 /2 , therefore value
48. (a, c) Given functions f : R ® R be defined by of d = 3
f (x) = x 3 - x 2 + (x - 1) sin x and g : R ® R be an arbitrary function. \ c + d = 1 + 3 = 4.
Now, let g is continuous at x = 1, then
50. (a, b, d) The given function f : R ® R is satisfying as
(fg)(x) - (fg)(1) (fg)(1 - h) - (fg)(1)
lim = lim
x ® 1- x -1 h ® 0 1-h-1 f (x + y) = f (x) + f (y) + f (x) f (y).
(fg)(x) = f (x) . g(x) (given) f (x + h) - f (x)
Q So, f ¢ (x) = lim
f (1 - h) . g(1 - h) - f (1) . g(1) h ®0 h
= lim
h ®0 -h f (x) + f (h) + f (x) f (h) - f (x)
= lim
f (1 - h) . g(1) h ®0 h
= lim
h ®0 -h f (h)
= lim (1 + f (x))
h ®0 h
{Qf(1) = 0 and g is continuous at x = 1, so g(1 - h) = g(1)}
Q f (x) = xg(x)
(1 - h) 2 (- h) + (- h) sin(1 - h)
= g(1) lim = (1 + sin1)g(1) f (x)
h ®0 -h Þ g(x) =
x
Similarly, f (x)
(fg)(x) - (fg)(1) f (1 + h) .g(1) \ lim = lim g(x) = 1 (given)
lim = lim x ®0 x x ®0
x ® 1+ x -1 h ®0 h
\ f ¢ (x) = 1 + f (x)
(1 + h) 2 (h) + hsin(1 + h) f ¢ (x)
= g(1) lim Þ =1
h ®0 h 1 + f (x)
= (1 + sin1)g(1) Þ loge (1 + f (x)) = x + c
QRHD and LHD of function fg at x = 1 is finitely exists and equal, so Þ 1 + f (x) = ex + c
fg is differentiable at x = 1 Þ f (x) = ex + c - 1
Now, let (fg)(x) is differentiable at x = 1, so Q f(0) = 0
(fg)(x) - (fg)(1) (fg)(x) - (fg)(1)
lim = lim \ c =0
x ® 1- x -1 x ® 1+ x -1
Therefore, f (x) = ex - 1 is differentiable at every x ÎR.
f (x)g(x) - f (1)g(1) f (x)g(x) - f (1)g(1)
Þ lim = lim And f ¢ (x) = ex Þf¢ (0) = 1
x ® 1- x -1 x ® 1+ x -1
26 Differential Calculus

(x) ex - 1 53. (a) Given function is


Now, g(x) = f = and if g(0) = 1
x x ì 2 cos x - 1 p
,x ¹
LHD (at x = 0) of ïï
f (x) = í cot x - 1 4
g(0 - h) - g(0) p
g(x) = lim ï k , x=
h ®0 -h ïî 4
e- h - 1 p
Q Function f (x) is continuous, so it is continuous at x = .
-1 4
-h p
= lim \ f æç ö÷ = lim f (x)
h ®0 -h è4ø x®p
4
e- h - 1 + h 1 lim
= lim = 2 cos x - 1
h ®0 h2 2 Þ k=x®p
and, RHD (at x = 0) of 4 cot x - 1
g(0 + h) - g(0) Put
p
x = + h,
g(x) = lim
h
h ®0 4
p
eh - 1 - h 1 when x ® , then h ®0
= lim = 4
h ®0 h2 2
p
So, if g(0) = 1, then g is differentiable at every x ÎR. 2 cos æç + hö÷ - 1
lim è4 ø
k=
51. (c) Given function f : [-1, 3] ® R is defined as h ®0 p
cotæç + hö÷ - 1
ì| x | + [x], -1 £ x < 1 è4 ø
ï
f (x) = í x + | x | , 1 £ x < 2 1 1
2 éê cos h - sin hùú - 1
ï x + [x ], 2 £ x £ 3 lim ë 2 2 û
î =
h ®0 cot h - 1
ì - x - 1, - 1 £ x < 0 -1
ï x, cot h + 1
0£ x <1
ïï é
= í 2x, 1 £ x < 2 [Q if n £ x < n + 1, " n ÎInteger, [x] = n] êQ cos (x + y) = cos x cos y - sin x sin y and
ï x + 2, 2 £ x < 3 ë
ï cot x cot y - 1 ù
îï 6, x =3 cot (x + y) = ú
cot y + cot x û
Q lim f (x) = - 1 ¹ f (0) [Q f(0) = 0] lim cos h - sin h - 1
x ®0 - =
h ®0 -2
Q lim f (x) = 1 ¹ f (1) [Q f(1) = 2]
x ® 1- 1 + cot h
Q lim f (x) = 4 = f (2) = lim f (x) = 4 [Q f(2) = 4] lim é (1 - cos h) + sin h ù
x ® 2- x ® 2+ = ê (sin h + cos h) ú
h ®0 ë 2sin h û
and lim f (x) = 5 ¹ f (3) [Q f(3) = 6]
x ®3- é 2h h h ù
lim ê 2 sin 2 + 2 sin 2 cos 2 ú
\ Function f (x) is discontinuous at points 0, 1 and 3. = (sin h + cos h)
h ®0 ê 4 sin
h
cos
h ú
52. (d) ê ú
1 ë 2 2 û
æ 1 + f (3 + x) - f (3) ö x é h h ù
Let l = lim ç ÷ [1¥ form] lim ê sin 2 + cos 2 ú
x ®0 è 1 + f (2 - x) - f (2) ø = ê ´ (sinh + cos h) ú
h ® 0 ê 2cos h ú
1 æ 1 + f (3 + x ) - f (3) ö
lim ç1 -
x ®0 x è
÷
1 + f (2 - x ) - f (2) ø ë 2 û
Þ l =e 1
é 1 + f (2 - x ) - f (2) - 1 - f (3 + x ) + f (3) ù
Þk=
lim ê ú 2
x ® 0ë x (1 + f (2 - x ) - f (2))
=e û
54. (b) Given function is f (x) = 15 - | x - 10| , x ÎR
é f (2 - x ) - f (3 + x ) + f (3) - f (2) ù
lim ê ú and g(x) = f (f (x)) = f (15 -| x - 10 | )
x ® 0ë x (1 + f (2 - x ) - f (2))
=e û
= 15 - | 15 - | x - 10 | - 10 |
= 15 - | 5 - | x - 10 | |
On applying L’Hopital rule, we get
-f ¢ (2 - x ) -f ¢ (3 + x )
ì15 - | 5 - (x - 10) | , x ³ 10
é
lim ê
ù =í
î15 - | 5 + (x - 10) | , x < 10
ú
x ® 0ë 1 - xf ¢ (2 - x ) + f (2 - x ) - f (2) û
l =e
ì15 - | 15 - x | , x ³ 10
On applying limit, we get =í
æ -f ¢ (2) - f ¢ (3) ö î 15 - | x - 5 | , x < 10
ç ÷
l = eè 1 - 0 + f (2) - f (2) ø = e0 = 1 ì 15 + (x - 5) = 10 + x , x < 5
1
ï 15 - (x - 5) = 20 - x , 5 £ x < 10
ï
æ 1 + f (3 + x) =í
- f (3) ö x
So, lim ç ÷ =1 ï 15 + (x - 15) = x , 10 £ x < 15
x ®0 è 1 + f (2 - x) - f (2) ø
ïî15 - (x - 15) = 30 - x , x ³ 15
JEE Main & Advanced Solutions (2021-18) 27

From the above definition it is clear that g(x) is not differentiable 58. (d) Given function
at x = 5, 10, 15. ìsin(p + 1) x + sin x
x , x <0
55. (b) Given function f (x) = [x] - éê ùú, x ÎR ï
ë4 û ïï x
f (x) = í q , x =0
æ 4 + h ùö ï 2
Now, lim f (x) = lim ç [4 + h] - éê ÷ ï x+x - x , x >0
x ® 4+ h ®0 è ë 4 úû ø ïî x 3 /2
[Q put x = 4 + h, when x ® 4+ , then h ®0] is continuous at x = 0, then
= lim (4 - 1) = 3 f (0) = lim f (x) = lim f (x) …(i)
h ®0 x ®0 - x ®0 +
æ 4 - h ùö sin(p + 1) x + sin x
and lim f (x) = lim ç [4 - h] - éê ÷ lim f (x) = lim
x ® 4- h ®0 è ë 4 úû ø x ®0 - x ®0 - x
[Q put x = 4 - h, when x ® 4- then h ®0]
= p + 1 + 1 =p + 2 éQ lim sin(ax) = aù
= lim (3 - 0) = 3 êë x ®0 x úû
h ®0
4 x + x2 - x x [(1 + x) 1/ 2 - 1]
and f(4) = [4] - éê ùú = 4 - 1 = 3 and lim f (x) = lim 3 /2
= lim
ë4 û x ®0 +
x ®0 +
x x ®0 +
x x
Q lim f (x) = f (4) = lim f (x) = 3 æ 1 æ1 ö ö
x ® 4- x ® 4+ ç ç - 1÷ ÷
1
ç1 + x + 2 è 2 ø 2
x + .... - 1÷
So, function f (x) is continuous at x = 4. ç 2 2! ÷
ç ÷
56. (d) Given function = lim
è ø
[Q (1+ x)n
ìa| p - x | + 1, x £ 5 x ®0 + x
f (x) = í n(n - 1) 2 n(n - 1(n - 2)) 3
îb| x - p | + 3, x > 5 = 1 + nx + x + x + ...,| x| < 1]
1× 2 1×2×3
and it is also given that f (x) is continuous at x = 5.
æ 1 æ1 ö ö
Clearly, f (5) = a(5 - p) + 1 …(i) ç ç - 1÷ ÷
1 2 è 2 ø 1
lim f (x) = lim [a| p - (5 - h)| + 1] = lim ç + x + ...÷ =
x ®5 - h ®0 x ®0 + ç
2 2! ÷ 2
ç ÷
è ø
= a(5 - p) + 1 …(ii)
From Eq. (i), we get
and lim f (x) = lim [b| (5 + h) - p| + 3] 1 1 3
x ®5 + h ®0 f (0) = q = and lim f (x) = p + 2 = Þ p = -
= b(5 - p) + 3 …(iii) 2 x ®0 - 2 2
Q Function f (x) is continuous at x = 5. æ 3 1ö
So, (p, q) = ç - , ÷
è 2 2ø
\ f (5) = lim f (x) = lim f (x)
x ®5 + x ®5 - 59. (d) Key Idea A function is said to be continuous
Þ a(5 - p) + 1 = b(5 - p) + 3 if it is continuous at each point of the domain.
ì 5 if x £1
Þ (a - b)(5 - p) = 2 ïa + bx
ï if 1 < x < 3
2 We have, f (x) = í
Þ a-b= ïb + 5x if 3 £ x < 5
5-p ïî 30 if x ³5
57. (d) Given function, g(x) = | f (x)| Clearly, for f (x) to be continuous, it has to be continuous at x = 1,
where f :R ® R be differentiable at c ÎR and f (c) = 0, then for x = 3 and x = 5
function ‘g’ at x = c [Q In rest portion it is continuous everywhere]
g(c + h) - g(c) \ lim (a + bx) = a + b = 5 …(i)
g¢ (c) = lim [where h > 0] x ® 1+
h ®0 h [Q lim f (x) = lim f (x) = f (1)]
x ® 1- x ® 1+
| f (c + h)| - | f (c)|
= lim lim (b + 5x) = b + 25 = 30 …(ii)
h ®0 h x ®5-
[Q lim f (x) = lim f (x) = f (5)]
| f (c + h)| x ®5- x ®5+
= lim [as f (c) = 0(given)]
h ®0 h
On solving Eqs. (i) and (ii), we get b = 5 and a = 0
f (c + h) - f (c)
= lim [Qh > 0] Now, let us check the continuity of f (x) at x = 3.
h ®0 h
Here, lim (a + bx) = a + 3b = 15
f (c + h) - f (c) x ® 3-
= lim
h ®0 h and lim (b + 5x) = b + 15 = 20
x ® 3+
= f ¢ (c) [Qf is differentiable at x = c] Hence, for a = 0 and b = 5, f (x) is not continuous at x = 3
Now, if f ¢ (c) = 0, then g(x) is differentiable at x = c, otherwise LHD \ f (x) cannot be continuous for any values of a and b.
(at x = c) and RHD (at x = c) is different.
28 Differential Calculus

60. (a) Key Idea This type of problem can be solved graphically Y
ìïmax {| x | , x 2 }, | x| £ 2
We have, f (x) = í y=f(x)
ïî 8 - 2| x| , 2 < | x| £ 4 O 1
–1
Let us draw the graph of y = f (x) –1 , –1 , 1 , –1 ,
Ö2 Ö2 Ö2 Ö2
For | x | £ 2f (x) = max{| x| x 2 }
y=–|x|
Let us first draw the graph of y = | x |
and y = x 2 as shown in the following figure. Q graph of y = - | x | is
y=x2
y=|x| Y
2

X
1

–2 –1 0 1 2
–1
and graph of y = - 1 - x 2
–2
Y
Clearly, y = | x | and y = x 2 intersect at x = - 1, 0, 1
Now, the graph of y = max {| x| , x 2 } for | x | £ 2 is X
4

y=x2
[Q x 2 + y 2 = 1 represent a complete circle]
ì - 1- x2 , 1
1
ï -1 < x £ -
2
ï
ï 1 1
–2 –1 1 2 Þ f (x) = í -| x| , - <x£
ï 2 2
y =|x| ï 1
2
ï - 1- x , < x <1
î 2
For | x | Î(2, 4]
From the figure, it is clear that function have sharp edges, at
ì8 - 2x, x Î(2, 4] 1 1
f (x) = 8 - 2| x | = í x=- , 0,
î8 + 2 x , x Î [- 4, - 2) 2 2
é Q2 < | x | £ 4 ù \ Function is not differentiable at 3 points.
ê Þ | x | > 2 and | x | £ 4ú
ê ú 62. (a) Key Idea This type of problem can be solved graphically.
ê –4 –2 2 4 ú
êë úû
ì - 1, -2£ x <0
Hence, the graph of y = f (x) is We have, f (x) = í 2
îx - 1, 0 £ x £ 2
4 and g(x) = | f (x)| + f (| x| )
1, -2£ x <0
2x

y=

ì
ì 1, -2£ x <0 ï
8+

8–

1 Clearly, | f (x)| = í 2 = í- (x 2 - 1), 0 £ x < 1


y=

2x

î| x - 1| , 0 £ x £ 2 ï x 2 - 1, 1£ x £2
î
–4 –2 –1 1 2 4
and f (| x | ) = | x | 2 - 1, 0 £ | x | £ 2
[Q f (| x | ) = - 1 is not possible as | x | </ 0]
From the graph it is clear that at x = - 2, - 1, 0, 1, 2 the curve has = x 2 - 1, | x | £ 2 [Q | x | 2 = x 2 ]
2
sharp edges and hence at these points fis not differentiable. = x - 1, - 2 £ x £ 2
\ g(x) = | f (x)| + f (| x | )
61. (a) Key Idea This type of questions can be solved graphically.
ì 1 + x 2 - 1, - 2 £ x < 0 ì x 2, -2£ x <0
Given, f : (-1, 1) ¾® R, such that ï 2 2 ï
= í- (x - 1) + x - 1, 0 £ x < 1 = í 0, 0£ x <1
f (x) = max ìí-| x | , - 1 - x 2 üý ï x 2 - 1 + x 2 - 1, ï2 (x 2 - 1), 1 £ x £ 2
î þ 1£ x £2
î î
On drawing the graph, we get the follwong figure. Now, let us draw the graph of y = g(x), as shown in the figure.
JEE Main & Advanced Solutions (2021-18) 29

Y
(2,6) Clearly, lim f ¢ (x) = 0 = lim f ¢ (x)
x ®0 - x ®0 +
(–2,4) y=x 2
and lim f ¢ (x) = 0 = lim f ¢ (x)
y=0 y=2 (x2 –1)
x ® p- x ® p+
X¢ X \ f is differentiable at x = 0 and x = p
–2 –1 O 1 2

Hence, f is differentiable for all x.


2
Y¢ 66. (d) (i) Given, f1 : R ® R and f1 (x) = sin ( 1 - e - x )
1 \ f1 (x) is continuous at x = 0
[ Here, y = 2 (x 2 - 1) or x 2 = (y + 2) represent a parabola with
2 2 1 2
Now, f1 ¢ (x) = cos 1 - e- x . (2xe- x )
vertex (0, - 2) and it open upward] 2
2 1 - e- x
Note that there is a sharp edge at x = 1 only, so g(x) is not
differentiable at x = 1 only. At x = 0
63. (b) We have, f (x) = sin| x | - | x | + 2 (x - p) cos| x | f1 ¢ (x) does not exists.
ì- sin x + x + 2(x - p) cos x, if x < 0 \ f1 (x) is not differential at x = 0
f (x) = í
î sin x - x + 2(x - p) cos x, if x ³ 0 Hence, option (2) for P.
[Qsin(- q) = - sin q and cos(- q) = cos q] ì | sin x| , if x ¹ 0
\ f ¢ (x) = ï
(ii) Given, f2 (x) = í tan-1 x
ì- cos x + 1 + 2cos x - 2(x - p) sin x ; if x < 0 îï 1, if x = 0
í
î cos x - 1 + 2cos x - 2(x - p) sin x , if x > 0 ì - sin x
ï tan-1 x x <0
Clearly, f (x) is differentiable everywhere except possibly at x = 0
ï
[Qf ¢ (x) exist for x < 0 and x > 0] ï sin x
Þ f2 (x) = í -1
x >0
Here, Rf ¢ (0) = lim (3cos x - 1 - 2(x - p) sin x) = 3 - 1 - 0 = 2
+
ï tan x
x®0 ï
and Lf ¢ (0) = lim (cos x + 1 - 2(x - p) sin x) ïî 1 x =0
x ® 0-
=1+ 1-0=2
Clearly, f2 (x) is not continuous at x = 0.
Q Rf ¢ (0) = Lf ¢ (0)
So, f (x) is differentiable at all values of x. \ Option (1)for Q.
Þ K=f (iii) Given, f3 (x) = [sin (loge (x + 2))] , where [ ] is G.I.F.
64. (c) Let us draw the graph of y = f (x), as shown below and f3 : (-1,ep/ 2 - 2) ® R
Clearly, the function f (x) = min {sin x,cos x } is not differentiable at It is given
- 3p p - 1 < x < ep/ 2 - 2
x= and
4 4 Þ - 1 + 2 < x + 2 < ep/ 2 - 2 + 2
[these are point of intersection of graphs of sin x and cos x in Þ 1 < x + 2 < ep/ 2
- 3p p ü loge 1 < loge (x + 2) < loge ep / 2
(- p, p), on which function has sharp edges]. So, S = ìí , ý, Þ
î 4 4þ p
Þ 0 < loge (x + 2) <
which is a subset of 2
ì -3p , - p , 3p , p ü p
í ý Þ sin0 < sinloge (x + 2) < sin
î 4 4 4 4þ 2
Þ 0 < sinloge (x + 2) < 1
65. (a) We have, f (x) = | x - p|×(e | x | - 1) sin| x|
\ [sinloge (x + 2)] = 0
ì (x - p) (e- x - 1) sin x, x <0
ï \ f3 (x) = 0, f ¢3 (x) = f3 ¢ ¢ (x) = 0
f (x) = í- (x - p) (ex - 1) sin x, 0 £ x < p
It is differentiable and continuous at x = 0.
ï (x - p) (ex - 1) sin x, x³p
î \ Option (4) for R
We check the differentiability at x = 0 and p. ì 2 æ1ö
We have, ïx sinç ÷ , if x ¹ 0
(iv) Given, f4 (x) = í èxø
ì(x - p) (e- x - 1) cos x + (e- x - 1) sin x ïî 0, if x = 0
ï
+ (x - p) sin xe- x (-1), x < 0 1
ï Now, lim f4 (x) = lim x 2 sin æç ö÷ = 0
ï x x x ®0 x ®0 èxø
ï- [(x - p) (e - 1) cos x + (e - 1) sin x
f ¢ (x) = í 1 1
ï + (x - p) sin xex ],0 < x < p f4 ¢ (x) = 2x sinæç ö÷ - cos æç ö÷
èxø èxø
ï x x
ï(x - p) (e - 1) cos x + (e - 1) sin x For x = 0,f4 ¢ (x) = lim
f (0 + h) - f (0)
ï + (x - p) sin xex , x > p h ®0 h
î
30 Differential Calculus

1 1 1 ö
h2 sin æç ö÷ - 0 æ
Again, lim f ¢ (x) = lim ç2x sin æç ö÷ - cos æç ö÷ ÷
èh ø x ®0 x ®0 è èxø è x øø
Þ f4 ¢ (x) = lim
h ®0 h
does not exists.
1
Þ f4 ¢ (x) = lim h sin ç ö÷ = 0
æ
1
h ®0 èh ø Since, lim cos æç ö÷ does not exists.
x ®0 èxø
ì æ1ö æ1ö
ï2x sin ç ÷ - cos ç ÷, x ¹ 0 Hence, f ¢ (x) is not continuous at x = 0.
Thus, f4 ¢ (x) = í èxø èxø
îï 0, x =0 \ Option (3) for S.

dy/dx as a Rate Measurer and Tangents Normals


1. (b) f (0) = 0, f (1) = 1 and f (2) = 2 dy b2 x
= - 2 = m1 (Let)
Let h(x) = f (x) - x dx a y
Clearly h(x) is continuous and twice differentiable on (0, 2) and differentiating Eq. (ii) w.r.t. x,
Also, h(0) = h(1) = h(2) = 0 dy - x
= = m2 (Let)
\ h(x) satisfies all the condition of Rolle’s theorem. dx y
\ There exist C1 Î(0, 1) such that h¢ (c1) = 0 Let angle be q.
Þ f ¢ (c1) - 1 = 0 Þ f ¢ (c1) = 1 Then,
also there exist c2 Î(1, 2) such that h¢ (c2 ) = 0 -b2 x x
2
+
m1 - m2 y
Þ f ¢ (c2 ) = 1 tan q = = a y2 2
Now, using Rolle’s theorem on [c1, c2 ] for f ¢ (x) 1 + m1m2 b x
1+ 2 2
We have f ¢ ¢ (c) = 0,c Î(c1, c2 ) a y
Hence, f ¢ ¢ (x) = 0 for some x Î(0, 2). xy (a2 - b2 ) a3b3 (a2 - b2 ) a-b
3 2
= 2 2
= 2
× =
2. (a) Given, f (x) = x - ax + bx + 4, x Î[12
,] ab (a + b) a2b2 ab
Here, f (1) = f (2) æa - b ö
Þ q = tan-1 ç ÷
Þ 1 - a + b - 4 = 8 - 4a + 2b - 4 Þ 3a - b = 7 … (i) è ab ø
Also, f ¢ (x) = 3x 2 - 2ax + b
4 4. (b) Let (x, y) be any arbitrary point on curve x 2 = 2y and find the
According to the question, f¢ æç ö÷ = 0 tangent line equation at this point, such that tangent line at (x, y) is
è3 ø
parallel to line x - y = 1.
2
4 4 To find tangent equation, differentiate the following equation so
Þ 3 ´ æç ö÷ - 2aæç ö÷ + b = 0
è3 ø è3 ø that we can find slope,
Þ - 8a + 3b = - 16 … (ii) x 2 - 2y = 0 …(i)
From Eqs. (i) and (ii), dy dy
2x - 2 = 0 gives =x
a = 5, b = 8 dx dx
\ (a, b) = (5, 8) Slope (say m1) = x
3. (c) Given curves
Also, slope of line x - y = 1 or y = x - 1 is 1 (say m2 ). Since, x - y = 1
and tangent line is parallel.
x2 y2
+ =1 …(i) Therefore, their slope be equal.
2
a b2 Hence, m1 = m2 gives, x = 1
and x 2 + y 2 = ab, …(ii) 1
Put x = 1 in Eq. (i), we get y =
From Eqs. (ii) 2
1
y 2 = ab - x 2 Thus, (x, y) = æç1, ö÷
è 2ø
From Eq. (i),
1
Perpendicular distance between line x - y = 1 and point æç1, ö÷ is
b2 x 2 + a2 (ab - x 2 ) = a2b2 è 2ø
a2b given as,
(b2 - a2 ) x 2 = a2b(b - a) Þ x 2 = ,
a+ b ½(1) (1) + æ 1 ö (-1) - 1½
½ ç ÷ ½ ½ -1 ½
è2 ø
a2b ab2 P =½ ½ =½ ½
y 2 = ab -
(1) + (-1) ½ ½2 2½
= 2 2
a+ b a+ b ½
½ ½
æ a2b ab2 ö÷ 1
Point of intersection ç , [\ using perpendicular distance formula] =
ç a+ b a+ b ÷ 2 2
è ø
Now, differentiating Eq. (i) w.r.t. x, we have
JEE Main & Advanced Solutions (2021-18) 31

5. (4) If the curves cut at right angle, then product of slopes will be -1. dv da 36.
\ = 3a2 = 3a2 [from Eq. (i)]
First curve x = y 4 dt dt 12a
dv
Differentiate it, we get So, at a = 10 cm, = 09 . ´ 10 = 9 cm 3 /sec
dy dy 1 dt
1 = 4y 3 Þ = 3 Hence option (c) is correct.
dx dx 4y
1 8. (c) The average speed of the car, for time interval [t 1, t 2] is
Slope of first curve (m1) = 3 [at point (x 1, y 1)]
4y 1 f (t 2 ) - f (t 1) a (t 22 - t 21 ) + b (t 2 - t 1) d (f (t))
= =
Second curve xy = k t2 - t1 t2 - t1 dt
dy \ 2at + b = a (t 2 + t 1) + b
Differentiate it, 0 = x +y
dx t + t2
Þ t= 1
dy - y 2
Þ =
dx x \ The average speed of the car over the time interval [t 1, t 2 ] is
-y 1 t + t2
Slope of second curve (m2 ) = [at (x 1, y 1)] attained at the point 1 .
x1 2
Þ m1.m2 = - 1 9. (d) It is given that, a spherical iron ball of 10 cm radius is coated with
a layer of ice of uniform thickness, let the thickness is ‘x’ cm, then
1 æ -y 1 ö volume of the ball is
Þ ç ÷ = -1
4y 31 è x 1 ø 4
V = p(10 + x) 3
-1 3
Þ = -1
4y 21 x 1 On differentiating w.r.t. ‘t’, we get
dV dx
-1 = 4p(10 + x) 2 , … (i)
Þ 6
= -1 [using x 1 = y 41 ] dt dt
4(y 1) where t is time in min.
1 dV
Þ y61 = It is given, the = - 50cm3 /min,
4 dt
Also, x 1y 1 = k, using x 1 = y 41 , we get k = y51 or k6 = (y 1) 30 Now when x is 5 cm, then
5 dx
1 æ1 ö - 50 = 4p(10 + 5) 2 [from Eq. (i)]
Q y61 = , then y 30
1 =ç ÷ dt
4 è4 ø dx 50 1
Þ =- =- cm / min
\ (4k)6 = 46 .k6 = 46 (y 1) 30 dt 4p(225) 18p
5
1 Negative sign indicates the thickness of ice layer decreases with
= 46 æç ö÷ = 4 time. Hence, option (d) is correct.
è4 ø
\ (4k)6 = 4 10. (d)
6. (d) Given, curve Þ y = x 3 … (i) If function f : [-7, 0] ® R be continuous on [-7, 0] and
3 differentiable on (-7, 0), then according to LMVT, we have
P (t,t ) f (-1) - f (-7)
Equation of tangent at P (t, t 3 ) = f ¢ (x) £ 2 , " x Î(-7, - 1)
(-1) - (-7)
(y - t 3 ) = 3t 2 (x - t) … (ii) f(-1) - (-3)
From Eqs. (i) and (ii), Þ £2
6
x 3 - t 3 = 3t 2 (x - t) Þ f(-1) £ 9 …(i)
Þ (x - t) (x + t 2 + xt) = 3t 2 (x - t)
2
f (0) - f (-7)
Similarly, = f ¢ (x) £ 2 , " x Î(-7, 0)
Þ x 2 + xt - 2t 2 = 0 0 - (-7)
Þ (x - t) (x + 2t) = 0 f(0) - (-3)
Þ £2
Þ x = t or x = - 2t 7
This is not possible. Þ f(0) £ 11 …(ii)
Now, the coordinate of Q = (x, y) From Eqs. (i) and (ii), we get
= (- 2t, (- 2t) 3 ) [Q y = x 3 ] f (-1) + f (0) £ 20
\ Q = (- 2t, - 8t 3 ) \ f (-1) + f (0) lies in the interval (-¥, 20].
\ Ordinate of the point dividing PQ in the ratio 1 : 2 is 11. (d)
2t 3 + (- 8t 3 ) Given function f (x) = x 3 - 4x 2 + 8x + 11, when x Î[0, 1] is a
= - 2t 3
1+ 2 continuous function in interval x Î[0, 1] and differentiable in
7. (c) Since, surface area of cube, A = 6a 2 cm 2. intervalx Î(0, 1), so according to Lagrange’s mean value theorem
for x = c Î(0, 1)
dA
It is given, . cm 2 /sec
= 36 f (1) - f (0)
dt f ¢ (c) =
1-0
da
Þ 12a = 36 . cm 2 /sec …(i) (1 - 4 + 8 + 11) - 11
dt Þ (3x 2 - 8x + 8) x = c =
1-0
Now, as volume of cube, v = a3 cm 3
32 Differential Calculus

Þ 3c2 - 8c + 8 = 5 15. (a) Given curve is y = x + sin y


Þ 3c2 - 8c + 3 = 0 \ On differentiating both sides w.r.t. x, we get
8 - 64 - 36 4 - 7 dy dy
Þ c= = [Qc Î(0, 1)] = 1 + cos y
6 3 dx dx
2
æ x + aö dy 1
12. (c) The given function f (x) = loge ç ÷ holds the Rolle’s Þ = … (i)
è 7x ø dx 1 - cos y

theorem for the interval [3, 4], so Q tangent at point (a, b) at given curve is parallel to line joining
æ0, 3 ö and æ 1 , 2ö.
f (3) = f (4) ç ÷ ç ÷
è 2ø è2 ø
9 + aö æ 16 + a ö
Þ loge æç ÷ = log eç ÷ dy 2 - (3 /2)
è 21 ø è 28 ø So = =1
9 + a 16 + a dx (a, b) (1 /2) - 0
Þ = 1
3 4 Þ = 1 Þ1 = 1 - cosb Þcosb = 0
Þ 36 + 4a = 48 + 3a 1 - cosb
Þ a = 12 …(i) Þ sinb = ± 1
and f ¢ (c) = 0, for some c Î(3, 4) Now, as point (a, b) on the given curve,
é 7x 7 x (2x + 0) - (x 2 + a) 7 ù So b = a + sinb Þ b - a = sinb
Þê 2 ´ ú =0
êë x + a (7 x) 2 úû x = c Þ | b - a| = | sinb| Þ | b - a| = 1
c(2c) - (c2 + 12) 16. (d) As the point P (h, k) is the nearest point on the curve
Þ 2
=0 y = x 2 + 7 x + 2 , to the line y = 3 x - 3, so the tangent to the parabola
(c + 12)c y = x 2 + 7 x + 7 at point P (h, k) is parallel to the line y = 3 x - 3
Þ c2 - 12 = 0 Qc Î(3, 4) dy
\ = 2h + 7 = 3 Þh = - 2 … (i)
Þ c = 12 Qc Î(3, 4) dx p
\ f ¢ ¢ (c = 12) and the point p(h, k) on the curve, so
c(c2 + 12)(2c) - (c2 - 12)(3c2 + 12) k = h2 + 7h + 2 = (- 2) 2 + 7 (- 2) + 2
=
((c2 + 12)c) 2 Þ k = 4 - 14 + 2 Þk = - 8
(2 ´ 12 ´ 24) - (0 ´ 48) 1 \ Point P(- 2, - 8)
= =
(24) 2 (12) 12 Now, equation of normal to the parabola
Hence, option (c) is correct. y = x 2 + 7 x + 7 at point P(- 2, - 8) is
-1
13. (*) Since, the functions f : [0, 1] ® R which are continuous on [0, 1] and y +8= (x + 2)
differentiable on (0, 1). dy
If f is a constant function then options (b), (c) and (d) are dx p
incorrect. 1
Þ y + 8 = - (x + 2) Þ x + 3y + 26 = 0
According to LMVT, for c¢ Î(C, 1) 3
f (1) - f (c) 17. (c) Equation of the given curve is
f ¢ (c¢) =
1-c y = (1 + x) 2 y + cos 2 (sin-1 x)
but c¢ ¹ c, so option (a) is also incorrect. Þ y = (1 + x) 2 y + 1 - sin2 (sin-1 x)
14. (c) It is given that a function f is continuous on [a, b] and twice Þ y = (1 + x) 2 y + 1 - x 2 …(i)
differentiable on (a, b) , such that for all x Î(a, b), f ¢ (x) > 0 and f ¢¢(x) < 0. [as sin(sin-1 x) = x]
Now, by LMVT for c Î(a, b), there is a Î(a, c) , such that So, at x = 0, y = 2.
f (c) - f (a)
f ¢ (a) = … (i) Now, let a point P (0, 2) on the curve.
c -a
On differentiating the Eq.(i) both sides w.r.t. x, we get
and there is b Î(c, b), such that dy dy
f (b) - f (c) = 2y (1 + x) 2 y - 1 + 2(1 + x) 2 y (loge (1 + x) ) - 2x
f ¢ (b) = … (ii) dx dx
b-c dy
So, at point P (0, 2) =4
Qf ¢¢(x) < 0 "x Î(a, b), then f ¢ (x) is a decreasing function, so dx P
f ¢ (b) < f ¢ (a) [Q a < b] \ Equation of normal to the curve at point ‘P ’ is
From Eqs. (i) and (ii), on putting the values of f¢ (a) and f¢ (b), we 1
y -2= (x - 0)
get -4
f (b) - f (c) f (c) - f (a) Þ x + 4y = 8
<
b-c c -a Hence, option (c) is correct.
f (c) - f (a) c - a 18. (4) The equation of tangent to the curve, y = e x at a point (c, e c) is
Þ >
f (b) - f (c) b - c y - ec = ec (x - c) …(i)
[Qf ¢ (x) > 0, so f (x) is an increasing function "x Î(a, b) and a < c < b] and equation of normal to the curve, y 2 = 4x at the point (1, 2) is
Hence, option (c) is correct. y - 2 = - 1(x - 1) …(ii)
JEE Main & Advanced Solutions (2021-18) 33

Q The lines (i) and (ii) intersect at same point on the X-axis, so put Þ x 21 = 4
y = 0 in both the equation and equate, we get Þ x1 = ± 2
x = 3 = c - 1 Þc = 4 Now, slope of tangent to the curve at P (x 1, y 1) is
Hence, answer is 4.00. 1 dy 6x 1
m= - = =
19. (b) Equation of the given curve is mN dx (x 1 , y 1 ) 2y 1 + 1
x 4ey + 2 y + 1 = 3 ½ 6x 1 ½ ½ 6(±2) ½ 12
Þ | m| =½ ½=½ ½= = 4
On differentiating w.r.t. ‘x’, we get ½2y 1 + 1½ ½2(1) + 1½ 3
dy 1 dy Hence, answer is 4.
ey (4x 3 ) + x 4ey + =0
dx y + 1 dx 22. (b) Let the thickness of layer of ice is x cm, the volume of spherical
\ At point P(1, 0), ball (only ice layer) is
dy 1 dy 4
0
e (4 ´ 1) + 1 .e 0
+ =0 V = p[(10 + x) 3 - 103 ] …(i)
dx 0 + 1 dx 3
dy On differentiating Eq. (i) w.r.t. ‘t’, we get
Þ = –2
dx P dV 4 dx
= p(3(10 + x) 2 ) = - 50 [given]
\ Equation of tangent at point P(10 , ) is dt 3 dt
y = –2(x –1) Þ2x + y = 2 …(i) [- ve sign indicate that volume is
From the option point (–2, 6) contain by the tangent (i). decreasing as time passes].
20. (b) Equation of given curve, 2 dx
Þ 4p(10 + x) = - 50
y = f (x) = x loge x, (x > 0) dt
At x = 5 cm
dy dx
\ = f ¢ (c) = 1 + loge c [4p (10 + 5) 2 ] = - 50
dx x = c dt
dx 50 1 1
Q The tangent to the given curve y = f (x) at point x = c is parallel Þ =- =- =- cm/min
dt 225(4p) 9(2p) 18p
to line segment joining points (1, 0) and (e, e).
e 1
So, 1 + loge c = So, the thickness of the ice decreases at the rate of cm/min.
e-1 18p
e 1 23 (b) Given curve is y = x 3 + ax - b …(i)
Þ loge c = -1= passes through point P(1, - 5) .
e-1 e-1
\ - 5 = 1 + a - b Þb - a = 6 …(ii)
æ 1 ö
ç ÷
èe - 1 ø and slope of tangent at point P(1, - 5) to the curve (i), is
Þ c =e is positive.
dy
m1 = = [3x 2 + a] (1, -5) = a + 3
21. (4) Let a point P (x 1, y 1) on the curve y 2 - 3 x 2 + y + 10 = 0, then slope dx (1, -5)
of
1 Q The tangent having slope m1 = a + 3 at point P(1, - 5) is
normal to the curve at point P is mN = - perpendicular to line - x + y + 4 = 0 , whose slope is m2 = 1.
æ dy ö
ç ÷
è dx ø (x 1 , y 1 ) \ a+ 3 = -1 Þa[Qm= -41m2 = -1]
Now, on substituting a = -4 in Eq. (ii), we get b = 2
On differentiating the curve w.r.t. x, we get
On putting a = -2 and b = 2 in Eq. (i), we get
dy dy
2y - 6x + =0 y = x 3 - 4x - 2
dx dx
dy 6x Now, from option (2, - 2) is the required point which lie on it.
Þ =
dx 2y + 1 24. (c) Given curve is y = f (x) = x 3 - x 2 - 2x ...(i)
2y + 1 So, f(1) = 1 - 1 - 2 = -2
\ mN = - 1
6x 1 and f(-1) = -1 - 1 + 2 = 0
3 Since, slope of a line passing through (x 1, y 1) and (x 2 , y 2 ) is given
Now, the normal to the curve at point P intersect the Y-axis æç0, ö÷, by
è 2ø
3 y -y
- y1 m = tanq = 2 1
x2 - x1
so slope of normal is mN = 2
0 - x1 \ Slope of line joining points (1, f(1)) and (-1, f (-1)) is
On equating the value of mN , we get f (1) - f (-1) -2 - 0
m= = = -1
3 - 2y 1 2y 1 + 1 1 - (-1) 1+ 1
=
x1 3x 1 dy
Now, = 3x 2 - 2x - 2
dx
Þ 9 - 6y 1 = 2y 1 + 1 (Q x 1 ¹ 0)
[differentiating Eq. (i), w.r.t. ‘x’]
Þ 8y 1 = 8 Þ y 1 = 1
According to the question,
Q Point P (x 1, y 1) on the curve, so
dy
(1) 2 - 3x 21 + 1 + 10 = 0 = m Þ 3x 2 - 2x - 2 = -1
dx
Þ 3x 21 = 12
34 Differential Calculus

Þ 3x 2 - 2x - 1 = 0 Þ a4 - 9 a2 = 0 Þ a = 0, - 3, 3
1 Þ a = 3 or - 3, [Q a ¹ 0]
Þ (x - 1) (3x + 1) = 0 Þ x = 1 ,-
3 Now, from Eq. (i),
1
Therefore, set S = ìí- , 1üý. a
î 3 þ b= 2
a -3
25. (b) Key Idea Use formula : 3 -3 1 1
1 Þ b= or = or -
Volume of cone = pr 2h, where r = radius and h = height of the cone. 9-3 9-3 2 2
3
According to the options, | 6 a + 2b | = 19 at
Given, semi-vertical angle of right circular cone
1 æ 1ö
= tan-1 æç ö÷ (a , b ) = ç ± 3, ± ÷
è2 ø è 2ø
1 1
Let a = tan-1 æç ö÷ Þ tana = 27. (b) Key Idea Angle between two curves is the angle between the
è2 ø 2 tangents to the curves at the point of intersection.
r 1 r Given equation of curves are
Þ = [from fig. tana = ]
h 2 h y = 10 - x 2 …(i)
1 and y = 2 + x2 …(ii)
Þ r= h …(i)
2 For point of intersection, consider
10 - x 2 = 2 + x 2
r
Þ 2x 2 = 8
Þ x2 = 4
Þ x=±2
l h Clearly, when x = 2 , then y = 6 (using Eq. (i))
a and when x = - 2, then y = 6
Thus, the point of intersection are (2, 6) and (-2, 6).
Let m1 be the slope of tangent to the curve (i) and m2 be the slope
1
Q Volume of cone is (V ) = pr 2h of tangent to the curve (ii).
3 dy dy
2 For curve (i) = -2x and for curve (ii) = 2x
1 1 1 dx dx
\ V = p æç hö÷ (h) = ph3 [from Eq. (i)]
3 2 è ø 12 \ At (2, 6), slopes m1 = - 4 and m2 = 4, and in that case
On differentiating both sides w.r.t. ‘t’, we get m - m1
| tan q| = 2
dV 1 dh 1 + m1m2
= p(3h2 )
dt 12 dt 4+ 4 8
dh 4 dV = =
Þ = 2 1 - 16 15
dt ph dt
dh 4 dV At (-2, 6), slopes m1 = 4 and m2 = - 4 and in that case
Þ = ´5 [Q given = 5 m 3 /min]
dt ph2 dt m - m1 -4 - 4 8
| tan q| = 2 = =
Now, at h = 10 m, the rate at which height of water level is rising 1 + m1m2 1 - 16 15
dh
= 28. (a) Given equation of curve is
dt h = 10 2
y = xex …(i)
4 1
= ´5= m/min Note that (1, e) lie on the curve, so the point of contact is (1, e).
p(10) 2 5p
Now, slope of tangent, at point (1,e), to the curve (i) is
26. (a) Equation of given curve is
x dy 2 2
y= , x ÎR, (x ¹ ± 3) …(i) = (x (2x) ex + ex ) (1, e)
x2 - 3 dx (1,e)

On differentiating Eq. (i) w.r.t. x, we get = 2e + e = 3e


Now, equation of tangent is given by
dy (x 2 - 3) - x (2x) (- x 2 - 3)
= = 2 ( y - y 1) = m ( x - x 1)
dx (x 2 - 3) 2 (x - 3) 2
y - e = 3e(x - 1) Þ y = 3ex - 2e
It is given that tangent at a point (a, b) ¹ (0, 0) on it is parallel to 4
the line On checking all the options, the option æç , 2eö÷ satisfy the
è3 ø
2x + 6y - 11 = 0.
equation of tangent.
2 dy
\ Slope of this line = - = 29. (d) The given curve is y = x 2 - 5 x + 5 …(i)
6 dx (a, b )
Now, slope of tangent at any point (x, y) on the curve is
a2 + 3 1 dy
Þ - =- = 2x - 5 …(ii)
(a2 - 3) 2 3 dx
Þ 3a2 + 9 = a4 - 6 a2 + 9 [on differentiating Eq. (i) w.r.t. x]
JEE Main & Advanced Solutions (2021-18) 35

Q It is given that tangent is parallel to line 32. (a, b, d) We have,


2y = 4x + 1 (f (0)) 2 + (f ¢ (0)) 2 = 85
dy and f :R ® [- 2, 2]
So, =2 [Q slope of line 2y = 4x + 1 is 2]
dx
(a) Since, f is twice differentiable function, so f is continuous
7
Þ 2x - 5 = 2 Þ 2x = 7 Þ x = function.
2
7 \ This is true for every continuous function.
On putting x = in Eq. (i), we get
2 Hence, we can always find x Î(r,s), where f (x) is one-one.
49 35 69 35 1 \ This statement is true.
y = - +5= - = -
4 2 4 2 4 (b) By L.M.V.T
7 1
Now, equation of tangent to the curve (i) at point æç ,- ö÷ and f ¢ (c) =
f (b) - f (a)
è 2 4ø b-a
having slope 2, is
1 7 f (b) - f (a)
y + = 2 æç x - ö÷ Þ | f ¢ (c)| =
4 è 2ø b-a
1 f (0) - f (- 4) f (0) - f (- 4)
Þ y + = 2x - 7 Þ | f ¢ (x0 )| = =
4 0+ 4 4
29
Þ y = 2x - …(iii) Range of f is [- 2, 2]
4
\ - 4 £ f (0) - f (-4) £ 4
1
On checking all the options, we get the point æç , - 7 ö÷ satisfy the f (0) - f (- 4)
è8 ø Þ 0£ £1
4
line (iii).
Hence, | f ¢ (x0 ) | £ 1.
30. (b) Given parabola is x 2 = 8y …(i)
Hence, statement is true.
Now, slope of tangent at any point (x, y) on the parabola (i) is æ 85 x ö
dy x (c) As no function is given, then we assume f (x) = 2sin ç ÷
= = tanq è 2 ø
dx 4
[Q tangent is making an angle q with the positive direction of æ 85 x ö
\ f ¢ (x) = 85 cos ç ÷
X-axis] è 2 ø
So, x = 4tanq Þ 8y = (4 tan q) 2 [on putting x = 4 tanq in Eq. (i)]
Now, (f (0)) 2 + (f ¢ (0)) 2 = (2sin0) 2 + ( 85 cos 0) 2
Þ y = 2 tan2 q
Now, equation of required tangent is (f (0)) 2 + (f ¢ (0)) 2 = 85
y - 2 tan2 q = tan q (x - 4 tan q) and lim f (x) does not exists.
x®¥
Þ y = x tan q - 2 tan2 q Hence, statement is false.
Þ x = y cot q + 2 tan q (d) From option b, | f ¢ (x0 )| £ 1
dy dy 3
31. (d) We have, y 2 = 6 x Þ 2y =6 Þ = and x0 Î(- 4, 0)
dx dx y
3 (f ¢ (x0 )) 2 £ 1
Slope of tangent at (x 1, y 1) is m1 = \
y1 Hence, g(x0 ) = (f (x0 )) 2 + (f ¢ (x0 ) 2 £ 4 + 1 [Qf (x0 ) Î[-2, 2]]
Also, 9x 2 + by 2 = 16 Þ g(x0 ) £ 5
dy Now, let p Î(-4, 0) for which g(p) = 5
Þ 18x + 2by = 0
dx Similarly, let q be smallest positive number q Î(0, 4)
dy -9x
Þ = such that g(q) = 5
dx by
Hence, by Rolle’s theorem is (p, q)
-9x 1
Slope of tangent at (x 1, y 1) is m2 = g¢ (c) = 0 for a Î(-4, 4) and since g(x) is greater than 5 as we
by 1 move form x = p to x = q
Since, these are intersection at right angle. and f (x)) 2 £ 4
27 x 1 2
Þ (f ¢ (x)) ³ 1 in (p, q)
\ m1m2 = - 1 Þ =1
by 21 Thus, g¢ (c) = 0
27 x 1 f ¢f + f ¢f ¢ ¢ = 0
Þ =1 [Q y 21 = 6x 1] Þ
6bx 1 So, f (a) + f ¢ ¢ (a) = 0 and f¢ (a) ¹ 0
9 Hence, statement is true.
Þ b=
2
36 Differential Calculus

Monotonicity, Maxima and Minima


1. (a) f (x) = x 3 - 6 x 2 + ax + b Þ a£ -4
Q f(2) = 0 Þ 2a + b = 16 Þ S = -4
and f(4) = 0 Þ 4a + b = 32 | R - S| = | - 2 + 4 | = 2
On solving a = 8, b = 0 4. (d) f (x) = 3 sin4 x + 10sin3 x + 6 sin2 x - 3
\ f (x) = x 3 - 6x 2 + 8x = x (x - 2) (x - 4) Þ f ¢(x) = cos x (12sin3 x + 30sin2 x + 12sin x)
f ¢ (x) = 3x 2 - 12x + 8 Þf ¢(x) = 6sin x cos x (2sin2 x + 5sin x + 2)
2 Þf ¢(x) = 3sin2x (2sin x + 1)(sin x + 2)
f ¢ (x) = 0 Þ x = 2 ±
3 f ¢ (x) = 0
Q f ¢ (x 2 ) = 0 and x 2 Î(2, 4) sin2 x = 0 or 2sinx + 1 = 0 [\ sin x ¹ - 2]
2
Þ x2 = 2 + and f ¢ (x 1) = - 1 Þ x = 0 or x = np + (-1)n æç - ö÷
p
3 è 6ø
Þ 3x 21 - 12x 1 + 8 = 0 -p p
As, x Îéê , ùú
Þ x 1 = 1, 3 (S 1 is true) ë 6 2û
2 öæ 2 öæ 2 -p
Now, 2(3x - 12x + 8) = 3 æç2 +
2
÷ç ÷ç - 2ö÷ x = 0,
è 3ø è 3ø è 3 ø 6
8 4 + – +
Þ x = , (S 2 is true)
3 3 –p 0 p/2
2. (c) f (x) = e 4x + 2e 3x - e x - 6 6
p
f ¢ (x) = 4e4 x + 6e3 x - ex So, f (x) is increasing in the interval x Îæç0, ö÷
è 2ø
= ex (4e3 x + 6e2 x - 1 )
p
Let g(x) = ex And f (x) is decreasing in the interval x Îæç - , 0ö÷
è 6 ø
h(x) = 4e3 x + 6e2 x - 1
g(x) > 0, " x ÎR ì -4 x 3 + 2x 2 + 3 x , x > 0
ï
h¢(x) = 12e3 x + 12e2 x = 12e2 x (ex + 1 ) 5. (c) f ( x) = í 3
h¢(x) > 0, " x ÎR îï 3 xe x ,x £ 0
h(x) is an increasing function. ì-4x 2 + 4x + 3 , x > 0
f ¢(x) = í x
Minimum value of h(x) will be when x ® - ¥ at [h(x)]min = - 1 and î 3e (x + 1) ,x £ 0
[h(x)]max = ¥
ì- (2x + 1) (2x - 3) , x > 0
f ¢ (x) = g(x) ×h(x) f ¢(x) = í x
î 3e (x + 1) ,x £ 0
Now, h(x) is an increasing function and h(x) varies from - 1 to + ¥.
So, this implies that h(x) cuts the X-axis at one point and which f¢(x)<0
f¢(x)<0 f¢(x)>0 f¢(x)>0
further implies f (x) changes its sign only at one point. Let’s say at
x=a –1 1/2 0 3/2
f (x) = e4 x + 2e3 x - ex - 6 3
f ¢(x) > 0 Þ x Îæç -1, ö÷
When, x ® - ¥; f (x) ® - 6 è 2ø
x ® + ¥; f (x) ® + ¥ x -1 2
6. (a) Given, f (x) = 3 loge -
x+1 x -1

3 (x + 1) ×1 - (x - 1) ×1 2
f(x) Þ f ¢ (x) = × +
æ x - 1ö (x + 1) 2 (x - 1) 2
ç ÷
è x + 1ø
æ x + 1ö é 2 ù 2
Þ f ¢ (x) = 3ç ÷ê ú+
y=–6 è x - 1 ø êë (x + 1) 2 úû (x - 1) 2
æ 2 öæ 3 1 ö
Þ f ¢ (x) = ç ÷ç + ÷
So, f (x) cuts the X-axis at a single point. è x - 1ø è x + 1 x - 1ø
3. (2) f (x) = x 2 + ax + 1 æ 2 ö é 3x - 3 + x + 1 ù
Þ f ¢ (x) = ç ÷ê
f ¢ (x) = 2x + a ú
è x - 1 ø ë ( x - 1) (x + 1) û
According to the question, f ¢ (x) ³ 0 for x Î[1,2] 2×2× (2x - 1)
For the least value 2x + a ³ 0 Þ f ¢ (x) =
(x - 1) 2 (x + 1)
Þ a ³ -2x Þ a ³ -2 Þ R = - 2
4(2x - 1)
For the greatest value 2x + a £ 0 So, f ¢ (x) =
(x - 1) 2 (x + 1)
Þ a £ -2x {x Î[1, 2]}
JEE Main & Advanced Solutions (2021-18) 37

Now, + – +
–¥ +¥
+ – + + –2 3
–1 1/2 1 4
For f (x) to be an increasing function, f ¢ (x) > 0. x Î(4, ¥) … (ii)
1 From Eqs. (i) and (ii), f (x) is increasing in x Î(- 5, - 4) È (4, ¥).
And f ¢ (x) > 0 at x Î(- ¥, - 1] È éê , ¥ö÷ 4x 3 - 3 x 2
ë2 ø 9. (a) Given, f (x) = - 2 sin x + (2x - 1) cos x
6
But domain of f (x) is x Î(- ¥, - 1) È (1, ¥) 12x 2 - 6x
f ¢ (x) = - 2cos x + (2x - 1) (- sin x) + cos x (2)
1 6
So, f ¢ (x) > 0 at x Î(- ¥, - 1) È é , 1ö÷ È (1, ¥)
êë 2 ø 2
= (2x - x) - 2cos x - 2x sin x + sin x + 2cos x
æ 1 ö = 2x 2 - x - 2x sin x + sin x = 2x (x - sin x) - 1(x - sin x)
or x Î(- ¥, - 1) È ç éê , ¥ö÷ - {1} ÷
è ë2 ø ø f ¢ (x) = (2x - 1) (x - sin x)
7. (2) Let f (x) = 2x 5 + 5 x 4 + 10x 3 + 10x 2 + 10x + 10 for x > 0
Using hit and trial method, x - sin x > 0
f(-2) = - 34 < 0 and f(-1) = 3 > 0 x < 0, x - sin x < 0
1
Hence, f (x) has a root in (-2, - 1). for x Î(-¥,0] È é , ¥ö÷, f ¢ ³ 0
êë 2 ø
Again,
1
f ¢ (x) = 10x 4 + 20x 3 + 30x 2 + 20x + 10 for x Îéê0, ùú, f ¢ (x) £ 0
2 1 ë 2û
= 10x 2 æç x 2 + 2x + 3 + + 2 ö÷ 1
è x x ø Hence, f (x) increases in éê , ¥ö÷.
é 1 1 ù ë2 ø
= 10x 2 ê æç x 2 + 2 ö÷ + 2æç x + ö÷ + 3ú
ëè x ø è xø û 10. (22) Let f (x) = ax 3 + bx 2 + cx + d
é 1
2
1 ù f ¢ (x) = 3ax 2 + 2bx + c
= 10x 2 ê æç x + ö÷ + 1 + 2æç x + ö÷ ú f ¢ ¢ (x) = 6ax + 2b
êë è xø è x ø úû
f ¢ (x) has local minima at x = -1, so

é 1 Q f¢ ¢ (-1) = 0 Þ -6a + 2b = 0 Þ b = 3a …(i)
= 10x 2 ê æç x + + 1ö÷ ú > 0, "x
êë è x ø úû f (x) has local minima at x = 1
f¢ (1) = 0 Þ 3a + 6a + c = 0
Þf (x) is strictly increasing function, since degree of f (x) is odd.
Þ c = -9a …(ii)
\ It has exactly on real root.
f(1) = - 10
Therefore, f (x) has atleast one root in (-2, - 1) = (a, a + 1)
Þ -5a + d = - 10 …(iii)
Þ | a| = | -2| = 2
f(-1) = 6 Þ 11a + d = 6…(iv)
8. (d) Solving Eqs. (iii) and (iv)
ì - 55x , x < -5 a = 1,d = -5
ï
Given, f (x) = í 2x 3 - 3x 2 - 120x , -5£ x <4 From Eqs. (i) and (ii),
ï2x 3 - 3x 2 - 36x + 10 , x ³4 b = 3,c = -9
î
ì - 55 , x < -5 \ f (x) = x 3 + 3x 2 - 9x - 5
\
ï
f ¢ (x) = í6(x 2 - x - 20) , - 5 £ x < 4 So, f(3) = 27 + 27 - 27 - 5 = 22
ï 6(x 2 - x - 6) , x ³4 11. (d) Let two pieces of wire one of length x and other of the length
î 20 - x.
ì - 55 , x < -5 Wire of length x is made into a square.
ï
f ¢ (x) = í6(x - 5) (x + 4) , - 5 £ x < 4 x/4
ï6(x - 3) (x + 2) , x ³4
î x/4 x/4
For f to be increasing, f ¢ (x) > 0.
Now, f ¢ (x) = - 55 is always less than zero. x/4
f ¢ (x) = 6(x - 5) (x + 4) < 0, - 5 £ x < 4 2
x
Critical points = 5, - 4 \ Area of square = æç ö÷ = A S (let)
è4ø
+ – + Wire of length (20 - x) is made into a regular hexagon.
–¥ +¥ a
–4 5 a a
–5 4
x Î(-5,-4) …(i)
a a
and f ¢ (x) = 6(x - 3) (x + 2) < 0, x ³ 4
a
Critical point, = 3, - 2
38 Differential Calculus

3 2 144
Area of hexagon = 6 ´ a (Let) Þ x=
4 p+4
2
3 3 æ 20 - x ö é 20 - x ù Circumference of circle = y
AH = ç ÷ Qa =
2 è 6 ø êë 6 úû = (36 - x)
144 36p
Sum of both area = 36 - =
x2 3 p+4 p+4
A = A S + AH = + (20 - x) 2
16 24 According to the question,
dA x 3 3x - 40 3 + 2 3x 36p
= - ( 20 - x) = k=
dx 8 12 24 p+4
4 4 36p
dA
= 0Þx =
40 3
=
40
= 40(2 - 3) Þ æç + 1ö÷k = æç + 1ö÷ = 36
è p ø è p 2ø p + 4
dx 3+ 2 3 3+2 x
2
d 2A 3 + 2 3 14. (c) f (x) = æç ö÷ ;x > 0
= >0 èxø
dx 2 24
Þ Area will be minimum, when \ log f (x) = x 2 (log2 - log x)
x = 40(2 - 3) f ¢(x) = f (x)[- x + (log2 - log x)2x]
20 - 40(2 - 3) 20 3 - 30 f ¢(x) = f (x) × x (2log2 - 2log x - 1)
\ Side of hexagon = =
6 3 For maxima or minima put f ¢ (x) = 0,
10(2 3 - 3) 10 we get
= =
3 2 3+3 4
2log2 - 2log x - 1 = logæç 2 ö÷ - 1 = 0
èx ø
2
12. (c) Length of box = a - 2x Þ x=
e
Breadth of box = b - 2x –
Sign of f¢(x) +
Height of box = x
Volume of box, V = (a - 2x) (b - 2x) x 2/Öe
Þ V = 4x 3 - 20x 2 - 2bx 2 + abx 2
\ At x = , f (x) has maximum value.
Differentiating V w.r.t. x, e
4
V(¢x) = 12x 2 - 4(a + b) x + ab 2 öe
2

Critical Point, V ¢(x) = 0 Maximum = æç ÷ = ee


è2 e ø
Þ 12x 2 - 4(a + b) x + ab = 0
15. (3) Let EG = x and DE = h
4(a + b) ± 16 (a + b) 2 - 4×12×ab
Þ x= Area of rectangle (A) = xh ...(i)
2(12)
A
(a + b) ± a2 + b2 - ab
x=
6
V ¢ ¢ (x) = 24x - 4(a + b) D F
(a + b) - a2 + b2 - ab
For x = , V ¢ ¢ (x) < 0 h
6
Hence, for maximum volume
B E G C
(a + b) - a2 + b2 - ab x
x= In DBDE,
6
h = BE tan60°
13. (36) Let x + y = 36
æ2 2 - x ö
where, x is perimeter of square and y is perimeter of circle. Þ h=ç ÷× 3 ...(ii)
x y è 2 ø
Then, side of square = and radius of circle =
4 2p From Eqs. (i) and (ii),
Now, Sum of areas of square and circle, 3
Area (A) = (2 2 - x) × x
x2 y2 2
A= +
16 4p 3
Þ A= (2 2x - x 2 )
x 2 (36 - x) 2 2
Þ A= + [Q y = 36 - x]
16 4p Differentiating w.r.t. ‘x’,
For minimum area, dA 3
= (2 2 - 2x)
dA dx 2
=0
dx dA
Substitute = 0. To determine the critical points.
dA 2x -2(36 - x) dx
Now, = + =0
dx 16 4p 3
Þ (2 2 - 2x) = 0 Þ x = 2
2
JEE Main & Advanced Solutions (2021-18) 39

d 2A 3 f (- 1) = a - b + c = 2 (given) …(i)
For maxima, = (-2) < 0
dx 2 2 f ¢ (- 1) = - 2a + b = 1 (given) … (ii)
From Eq. (i), f ¢¢(x) = 2a
æ 2ö 3 \ fmax
¢¢ (x) = 2a
h=ç ÷ 3 = 1
è 2 ø 2 Also, given maximum value of f ¢¢(x) =
2
3 1 1
So, area = 2 × = 3 i.e. 2a = Þa =
2 2 4
Squaring both sides, (area) 2 = 3 3
From Eq. (ii), b =
16. (a) f (x) = ax 2 + 6 x - 15 2
13
f ¢ (x) = 2ax + 6 From Eq. (i), c =
4
f ¢ (x) = 2(ax + 3)
3 x2 3 13
f (x) is increasing for æç - ¥, ö÷. \ f (x) = + x+
è 4ø 4 2 4
3 3 1 3 13
So, x=- = Here, f(- 1) = - + = 2
a 4 4 2 4
1 3 13
a= -4 and f(1) = + + = 5
4 2 4
g(x) = ax 2 - 6x + 15
Y
g¢ (x) = 2(ax - 3)
If g¢ (x) = 0 2
y=f(x)= x + 3 x+ 13
x=–3/4 5 4 2 4
13/4
2

g(x) X¢ X¢
–1 1

For x Î[- 1, 1]
3 3 f (x) Î[2, 5]
=- x=
a 4 \ Least value of a is 5.
3 x x
g(x) is maximum at x = - . 19. (b) Given, f (x) = (4a - 3) (x + loge 5) + 2(a - 7) cot ×sin2
2 2
4 x x
3 Þ f (x) = (4a - 3)(x + loge 5) + 2(a - 7) cos sin
17. (c) Given, f (x) = x 3 - 3 x 2 - f ¢¢(2) × x + f ¢¢(1) 2 2
3 2 Þ f (x) = (4a - 3)(x + loge 5) + (a - 7) sin x
Then, f ¢ (x) = 3x 2 - 6x - f ¢¢(2)
2 Þ f ¢ (x) = (4a - 3) (1 + 0) + (a - 7) cos x
f ¢¢(x) = 6x - 6 Þ f ¢ (x) = (4a - 3) + (a - 7) cos x
Put x = 1 and x = 2, When f ¢ (x) = 0, (4a - 3) + (a - 7) cos x = 0
f ¢¢(2) = 12 - 6 = 6 and f ¢¢(1) = 0 4a - 3
3 Þ cos x =
Therefore, f (x) = x 3 - 3x 2 - ´ 6x + 0 7 -a
2
As, - 1 £ cos x £ 1
Þ f (x) = x 3 - 3x 2 - 9x ...(i) 4a - 3 4a - 3
and f ¢ (x) = 3x 2 - 6x - 9 So, - 1 £ £1 Þ + 1³0
7 -a 7 -a
and f ¢¢(x) = 6x - 6 4a - 3 + 7 - a 3a + 4
Equate f ¢ (x) = 0 gives, Þ ³0 Þ ³0
7 -a 7 -a
Þ 3x 2 - 6x - 9 = 0
– + –
Þ 3(x 2 - 2x - 3) = 0
–4/3 7
3(x + 1) (x - 3) = 0 Þ x = - 1, x = 3
Now, f ¢¢(- 1) = 6(- 1) - 6 = - 12 < 0 a Î[- 4 /3, 7) …(i)
f ¢¢(3) = 6(3) - 6 = 12 > 0 4a - 3
Now, -1£0
\ (- 1) is maxima. 3 is minima. 7 -a
Local minimum value =f(3) 4a - 3 - 7 + a
Þ £0
f(3) = (3) 3 - 3(3) 2 - 9(3) [using Eq. (i)] 7 -a
= 27 - 27 - 27 5a - 10
Þ £0
f(3) = - 27 7 -a
18. (5) Given, f : [- 1, 1] ® R – + –
2 2 7
and f (x) = ax + bx + c
40 Differential Calculus

2
a Î(- ¥, 2] È [7, ¥) … (ii) 3
2
æ 3 ö
= æç r ö÷ + ç r ÷ [QBP = r cos q ]
From Eqs. (i) and (ii), è2 ø è 2 ø
4
a Îéê - , 2ùú 9 3
ë 3 û = r 2 + r 2 = 3r 2 Þ AB = 3r
4 4
20. (c) Let a DABC inscribed in a circle with centre O and radius r. Hence, the DABC is an equilateral triangle with side 3r.
A 21. (a) Given, curve is
1
y = x 4 - 5x 3 + 18x 2 - 19x …(i)
2
First, find the slope of given curve i.e. dy /dx,
O Differentiate Eq. (i),
dy 1
= (4x 3 ) - 5(3x 2 ) + 18(2x) - 19
q dx 2
B P C
= 2x 3 - 15x 2 + 36x - 19
Now, let f (x) = 2x - 15x 2 + 36x - 19
3

Let ÐOBC = q is slope of the curve and find its maximum value as follows,
1
Now, area of DABC = ´ Base × Height f ¢ (x) = 2(3x 2 ) - 15(2x) + 36
2
1 = 6x 2 - 30x + 36
A = ´ (BC) ´ (AP) …(i) Equate f ¢ (x) = 0 and solve for ‘x’,
2
6x 2 - 30x + 36 = 0
Now, BC = 2BP
Þ x 2 - 5x + 6 = 0
Consider DOBP, where OB = r 2
Þ x - 3x - 2x + 6 = 0
Then, BP = r cos q
Þ (x - 3) (x - 2) = 0
Hence, BC = 2r cos q
Þ x = 2 and 3
Again, AP = AO + OP
d
where, AO = r Now, f ¢¢(x) = (6x 2 - 30x + 36)
dx
Consider DOBP, where OB = r = 12x - 30
Then, OP = r sinq Then, f ¢¢(2) = 12(2) - 30 = 24 - 30
Þ AP = r + r sinq = -6<0
From Eq. (i), we get and f ¢¢(3) = 12(3) - 30 = 6 > 0
1
Area = ´ (2r cos q) ´ (r + r sin q) Qf ¢¢(2) < 0, this implies ‘2’ is point of maxim a.
2
\ At x = 2, slope will be maximum.
A = r 2 cos q(1 + sin q)
Since, at x = 2, slope will be maximum, then y-coordinate will be,
Now, 1
dA 2 y = (2) 4 - 5(2) 3 + 18(2) 2 - 19(2)
= r (- sin q)(1 + sin q) + r 2 cos 2 q 2
dq
= 8 - 40 + 72 - 38
= r 2 (cos 2 q - sin q - sin2 q)
= 72 - 70 = 2
= r 2 (1 - 2sin2 q - sin q)
\ Maximum slope occurs at point (2, 2).
= r 2 (1 + sin q)(1 - 2sin q)
dA 22. (144) f (x) = x 6 + ax 5 + bx 4 + cx 3 + dx 2 + ex + f
Equate =0 f (x)
dq As, lim =1 non-zero finite
Þ r 2 (1 + sin q)(1 - 2sin q) = 0
x ®0 x3
1 So, d = e = f = 0
Þ sinq =
2 and f (x) = x 3 (x 3 + ax 2 + bx + c)
p f (x)
Þ q= Hence, lim 3 = c = 1
6 x ®0 x
d 2A p Now, as f (x) = x6 + ax5 + bx 4 + x 3
Now, 2 < 0, when q =
dq 6 and f ¢ (x) = 0 at x = 1 and x = - 1
p
Þ A is maximum, when q = i.e. f ¢ (x) = 6x5 + 5ax 4 + 4bx 3 + 3x 2
6
Now, f¢ (1) = 0
p öæ p 3 3 2
\ Maximum area = r cos ç ÷ ç1 + sin ö÷ =
2 æ r Þ 6 + 5a + 4b + 3 = 0
è 6 øè 6ø 4
3 Þ 5a + 4b = - 9 … (i)
Height = AP = r
2 and f¢ (- 1) = 0
Consider DABP, Þ - 6 + 5a - 4b + 3 = 0
2 2
(AB) = (AP) + (BP) 2 Þ 5a - 4b = 3 … (ii)
JEE Main & Advanced Solutions (2021-18) 41

From Eqs. (i) and (ii), g(x)


\ f ¢ (x) = <0
a = - 3 /5 and b = - 3 /2 x2
3 3 Þf (x) is decreasing function for x Î(-1,0).
\ f (x) = x6 - x5 - x 4 + x 3
5 2 Similarly, for x Î(0, ¥), g¢ (x) < 0, so g(x) is decreasing function for
3 5 3 4 x Î(0, ¥).
\ 5f(2) = 5ê2 - (2) - (2) + (2) 3 ùú
é 6
ë 5 2 û So, g(x) < g(0)
é 3 ´ 32 3 ´ 16 ù Þ g(x) < 0," x Î(0, ¥)
= 5ê64 - - + 8ú g(x)
ë 5 2 û \ f ¢ (x) = 2 < 0
x
= 320 - 96 - 120 + 40 = 144
Þf (x) is decreasing function for x Î(0, ¥).
23. (a and b) Given,
\ The given function f (x) is decreasing function for (-1, ¥).
x 2 - 3x - 6 Hence, option (d) is correct.
f (x) = … (i)
x 2 + 2x + 4 25. (a) Since, the given function f (x) = (3 x - 7) x 2/ 3 is increasing for x ÎR
2 2
(x + 2x + 4) (2x - 3) - (x - 3x - 6) (2x + 2) \ f ¢ (x) ³ 0
Þ f ¢ (x) =
(x 2 + 2x + 4) 2 2 -1 / 3
Þ x (3x - 7) + x 2 / 3 (3) ³ 0, x ¹0
5x (x + 4) 3
Þ f ¢ (x) =
(x 2 + 2x + 4) 2 2(3x - 7) + 9x
Þ ³ 0, x ¹0
+ – +
x 1/ 3
–¥ +¥ 15x - 14 14
–4 0 Þ ³ 0 Þ x Î(- ¥, 0) È æç , ¥ö÷
x 1/ 3 è 15 ø
Sign scheme for f ¢ (x) 14 14
Q (- ¥, 0) È æç , ¥ö÷ Ì (- ¥, 0) È æç , ¥ö÷
Here, f is decreasing in the interval (- 2, - 1) and f is increasing in è 15 ø è 15 ø
the interval (1, 2).
Hence, option (a) is correct.
11 -3
Now, f(- 4) = ,f(0) = [from Eq. (i)] 26. (a) Given function
6 2
p p
and lim f (x) = 1 f (x) = x cos -1 (- sin| x | ), x Îéê - , ùú
x® ± ¥
- 3 11 ë 2 2û
\ Range = éê , ùú = x (p - cos -1 (sin| x | )) {Qcos -1 (- x) = p - cos -1 x}
ë 2 6û
é p ù ìQcos -1 x = p - sin-1 x ü
Hence, f (x) is into. = x ê p - æç - sin-1sin| x| ö÷ ú í ý
ë è2 øû î 2 þ
Y p
= x éê + | x| ùú {Qsin-1sin x = x}
11/6 ë2 û
p p
1 x æç - x ö÷, x Îæç - , 0ö÷
è2 ø è 2 ø
=
p p
–4 O
X x æç + x ö÷, x Îæç0, ö÷
è2 ø è 2ø
p p
- 2x, x Îæç - , 0ö÷
–3/2 2 è 2 ø
So, f ¢ (x) =
p p
+ 2x, x Îæç0, ö÷
f (x) has local maxima at x = - 4 2 è 2ø
and local minima at x = 0. p
-2, x Îæç - , 0ö÷
24. (d) è 2 ø
Þf ¢ ¢ (x) =
ìï 1 log (1 + x), x ¹ 0 p
Given function f (x) = í x e for x Î(-1, ¥) 2, x Îæç0, ö÷
è 2ø
ïî 1, x =0 p p
\ f¢ is decreasing in æç - , 0ö÷ and increasing in æç0, ö÷.
1 log (1 + x) è 2 ø è 2ø
Now, f ¢ (x) = - e 2 ,
x (1 + x) x Hence, option (a) is correct.
x - (1 + x) loge (1 + x) 27. (d) Since, p¢ (x) = 0 at x = 1 and x = 2 and p(x) is cubic polynomial, so
for x Î(-1, ¥) - {0} =
x 2 (1 + x) p¢ (x) = a(x - 1) (x - 2) = a (x 2 - 3x + 2)
Let another function
æ x3 3 ö
g(x) = x - (1 + x) loge (1 + x) \ p(x) = açç - x 2 + 2x ÷÷ + b
Q g¢ (x) = 1 - 1 - loge (1 + x) = - loge (1 + x) è3 2 ø

Since, for x Î(-1, 0), g¢ (x) > 0, So g(x) is increasing function for According to the question,
1 3
x Î(-1,0) but as p(1) = 8 Þaæç - + 2ö÷ + b = 8
è3 2 ø
g(x) < g(0)," x Î(-1,0)
æ 1 1ö
\ g(x) < 0, " x Î(-1,0) Þ aç + ÷ + b = 8 Þ 5a + 6b = 48 … (i)
è3 2 ø
42 Differential Calculus

8 f (x) = x 3 - 3x 2 - 9x + 5
and p(2) = 4 Þaæç - 6 + 4ö÷ + b = 4 \
è3 ø Q f ¢ (x) = 3x 2 - 6x - 9 = 0
Þ 2a + 3b = 12 … (ii) (for local maxima and minima)
From Eqs. (i) and (ii), we get Þ x 2 - 2x - 3 = 0 Þ x 2 - 3x + x - 3 = 0
a = 24, b = - 12 Þ (x + 1) (x - 3) = 0 Þ f ¢ (x) = 0
Q p(0) = b = - 12 Þ x = -1, 3
28. (d) It is given that x = 1 is a critical point of the function Q f ¢ ¢ (x) = 6x - 6
f (x) = (3 x 2 + ax –2 –a)e x Q f¢ ¢ (-1) = - 12 and f¢ ¢ (3) = 12
So, f ' (1) = ex (6x + a) + ex (3x 2 + ax –2 –a)| x = 1 = 0 \ x = 3 is point of local minima.
Þ 6 + a + 3 + a - 2 - a = 0 Þ a = –7. Hence, answer is 3.
\ f ' (x) = ex [3x 2 – x –2] = 0 31. (c) Given region is
2
Þ x = 1 or æç – ö÷ {(x, y) ÎR ´ R :0 £ x £
p
and 0 £ y £ 2sin(2x)
è 3ø 2
and f ¢ ¢ (x) = ex (6x –1 + 3x 2 – x –2) = ex (3x 2 + 5x –3) On drawing the diagram,
Q f ¢ ¢ (1) = 5e > 0 Let the side PS on the X-axis, such that P (x, 0), and Q (x, 2sin(2x)), so
Þ x = 1 is the point of local minima. p
length of the sides PS = QR = 2 æç - x ö÷ and PQ = RS = 2sin2x.
2 4 10 è4 ø
and f ¢¢ æç – ö÷ = e–2 / 3 æç – –3ö÷ = –5e–2 / 3 < 0
è 3ø è3 3 ø Y
2
Þ x = – is the point of local maxima.
3 2
29. (d) Given function,
Q
f (x) = (1 - cos 2 x) (l + sin x) R
2
= sin x (l + sin x)
\ f ¢ (x) = sin 2x (l + sin x) + sin2 x (cos x) O P p/4 X
S p/2
1
= sin 2x éê l + sin x + sin x ùú
ë 2 û \ Perimeter of the rectangle
3 p
= sin 2x éê l + sin x ùú y = 4 éê - x + sin2x ùú
ë 2 û ë4 û
For maxima and minima, as dy
For maximum, = 0
3 dx
f ¢ (x) = 0 Þsin 2x æç l + sin x ö÷ = 0
è 2 ø 1
Þ - 1 + 2cos 2x = 0 Þ cos2x =
p p 2
So, either sin2x = 0 Þ x = 0 as x Îæç - , ö÷ p p
è 2 2ø 2x = Þ x=
ì é p ùü
Þ íQ x Îê0, ú ý
3 3 6 î ë 2 ûþ
or l + sin x = 0 as there must exactly one maxima and exactly
2 d 2y
and = - 4sin2x| p <0
3 3 dx 2 x=
one minima, so l Îæç - , ö÷ - {0}.
p
x= 6
è 2 2ø 6
p
30. (3) Let a cubic polynomial \ At x = , the rectangle PQRS have maximum perimeter.
3 2
6
f (x) = ax + bx + cx + d
So length of sides
Q f(-1) = 10 p p p
Þ - a + b - c + d = 10 …(i) PS = QR = 2 æç - ö÷ =
è4 6 ø 6
Q f(1) = - 6 p
and PQ = RS = 2sinæç ö÷ = 3
Þ a + b + c + d = -6…(ii) è3 ø
Q f¢ (-1) = 0 p p
\ Required area = ´ 3 =
Þ 3a - 2b + c = 0 …(iii) 6 2 3
Q f¢ ¢ (1) = 0 32. (5.00) Given set S of polynomials with real coefficients
Þ 6a + 2b = 0
S = {(x 2 - 1) 2 (a0 + a1x + a2 x 2 + a3 x 3 ) :a0 , a1, a2 , a3 ÎR}
Þ 3a + b = 0 …(iv)
and for a polynomial f ÎS, Let
From Eqs. (i) and (ii), we get
f (x) = (x 2 - 1) 2 (a0 + a1x + a2 x 2 + a3 x 3 )
-2a - 2c = 16
it have - 1 and 1 as repeated roots twice, so graph of f (x) touches
Þ a + c = -8 …(v) the X-axis at x = - 1 and x = 1, so f ¢ (x) having at least three roots
From Eqs. (iii), (iv) and (v), we get x = - 1, 1 and a. Where a Î(- 1, 1) and f ¢ ¢ (x) having at least two roots
3a - 2(-3a) + (-a - 8) = 0 in interval (- 1, 1)
Þ 8a - 8 = 0 Þ a = 1 So, mf ¢ = 3 and mf ¢ ¢ = 2
So, b = -3, c = -9 and d = 5 \ Minimum possible value of (mf ¢ + mf ¢ ¢ ) = 5
JEE Main & Advanced Solutions (2021-18) 43

33. (0.50) The given function f : R ® R be defined by 1


Þ x £ and x (x - 1) £ 0
2
f( q ) = (sin q + cos q ) + (sin q - cos q ) 4 2
= 1 + sin2 q + (1 - sin2 q) 2 1 1
Þ x Îæç -¥, ùú and x Î[0, 1] So, x Îéê0, ùú
= 1 + sin2 q + 1 + sin2 2 q - 2sin2 q è 2û ë 2û
= sin2 2 q - sin2 q + 2 1
From, the above cases, x Îé0, ù È [1, ¥).
2 êë 2 úû
1 7
= æçsin2 q - ö÷ +
è 2ø 4 36. (b) Given function, f (x) = x kx - x 2 … (i)
The local minimum of function ‘ f ’ occurs when the function f (x) is defined if
1 kx - x 2 ³ 0 Þ x 2 - kx £ 0
sin2 q =
2 Þ x Î[0, k] … (ii)
p 5p 13p because it is given that f (x) is increasing in interval x Î[0, 3], so k
Þ 2q= , , ¼
6 6 6 should be positive.
p 5p 13p Now, on differentiating the function f (x) w.r.t. x, we get
Þ q= , , ,¼
12 12 12 x
f ¢ (x) = kx - x 2 + ´ (k - 2x)
but q Î{ l 1p, l 2 p, ¼ lr p }, where 0 < l 1 < ¼ < lr < 1. 2 kx - x 2
p 5p
\ q= , 2(kx - x 2 ) + kx - 2x 2 3kx - 4x 2
12 12 = =
2
So,
1
l 1 + ¼ + lr = + = 050
5
. 2 kx - x 2 kx - x 2
12 12 as f (x) is increasing in interval x Î[0, 3], so
34. (c) Given, f(x) = f (x) + f (2 - x) , " x Î (0, 2) f ¢ (x) ³ 0 " x Î(0, 3)
Þ f¢ (x) = f ¢ (x) - f ¢ (2 - x) …(i) Þ 3kx - 4x 2 ³ 0
Also, we have f ¢ ¢ (x) > 0 " x Î(0, 2) Þ 4x 2 - 3kx £ 0
3k
Þ f ¢ (x) is a strictly increasing function Þ 4x æç x - ö÷ £ 0
è 4ø
" x Î(0, 2).
é 3k ù
Now, for f(x) to be increasing, Þ x Îê0, ú (as k is positive)
ë 4û
f¢ (x) ³ 0 3k
[using Eq. (i)] So, 3£ Þ k ³4
Þ f ¢ (x) - f ¢ (2 - x) ³ 0 4
Þ f ¢ (x) ³ f ¢ (2 - x) Þ x > 2 - x Þ Minimum value of k = m = 4
[Qf¢ is a strictly increasing function] and the maximum value of f in [0, 3] is f(3).
Þ 2x > 2 Þ x > 1 Q f is increasing function in interval x Î[0, 3]
Thus, f(x) is increasing on (1, 2). QM = f (3) = 3 4 ´ 3 - 32 = 3 3
Similarly, for f(x) to be decreasing, Therefore, ordered pair (m, M) = (4, 3 3)
f¢ (x) £ 0 Hence, option (b) is correct.
Þ f ¢ (x) - f ¢ (2 - x) £ 0 [using Eq. (i)] x (d - x)
37. (a) We have, f (x) = -
Þ f ¢ (x) £ f ¢ (2 - x) (a 2 + x 2) 1 / 2 (b 2 + (d - x) 2) 1 / 2
Þ x <2- x [Qf¢ is a strictly increasing function] Differentiating above w.r.t. x, we get
2x < 2 Þ x < 1 1 2x
Þ (a2 + x 2 ) 1/ 2 - x
Thus, f(x) is decreasing on (0, 1). 2 (a2 + x 2 ) 1/ 2
f ¢ (x) =
35. (a) The given functions are (a2 + x 2 )
f (x) = ex - x and g(x) = x 2 - x, " x ÎR 2(d - x)(-1)
(b 2 + (d - x) 2) 1 / 2 (-1) - (d - x)
Then, h(x) = (fog)(x) = f (g(x)) 2(b 2 + (d - x) 2) 1 / 2
- [by using quotient
Now, h¢ (x) = f ¢ (g(x)) ×g¢ (x) (b 2 + (d - x) 2)
= (eg (x) - 1) × (2x - 1) rule of derivative]
= (e (x 2 - x )
- 1) (2x - 1) a2 + x 2 - x 2 b2 + (d - x) 2 - (d - x) 2
= +
x (x - 1) (a2 + x 2 ) 3 / 2 (b2 + (d - x) 2 ) 3 / 2
= (e - 1) (2x - 1)
Q It is given that h(x) is an increasing function, so h¢ (x) ³ 0 a2 b2
= 2 2 3 /2
+ > 0, " x ÎR
Þ (ex (x - 1) - 1)(2x - 1) ³ 0 (a + x ) (b + (d - x) 2 ) 3 / 2
2

Case I (2x - 1) ³ 0 and (ex (x - 1) - 1) ³ 0 Hence, f (x) is an increasing function of x.


1 38. (c) Given function is
Þ x ³ and x (x - 1) ³ 0
2 f (x) = 9x 4 + 12x 3 - 36x 2 + 25 = y (let)
Þ x Î[1 /2, ¥) and x Î(- ¥, 0] È [1, ¥), so x Î[1, ¥) dy
For maxima or minima put =0
Case II (2x - 1) £ 0 and [ex (x - 1) - 1] £ 0 dx
44 Differential Calculus

dy where, a is non-zero constant.


Þ = 36x 3 + 36x 2 - 72x = 0
dx f ¢ (x) = ax 3 - ax
Þ x 3 + x 2 - 2x = 0 a a
Þ f (x) = x 4 - x 2 + C [integrating both sides]
Þ x [x 2 + x - 2] = 0 4 2
2
Þ x [x + 2x - x - 2] = 0 where, C is constant of integration.
Þ x [x (x + 2) - 1(x + 2)] = 0 Now, since f (x) = f (0)
Þ x (x - 1) (x + 2) = 0 a 4 a 2 x4 x2
Þ x = -2, 0, 1 Þ x - x +C =C Þ =
4 2 4 2
By sign method, we have following
Þ x 2 (x 2 - 2) = 0 Þ x = - 2, 0, 2
– + – + Thus, f (x) = f (0) has one rational and two irrational roots.
–2 0 1 41. (b) Let h = height of the cone,
dy r = radius of circular base
Since, changes it’s sign from negative to positive at x = ‘-2 ’
dx
dy = (3) 2 - h2 [Ql 2 = h2 + r 2 ]
and ‘1’, so x = -2, 1 are points of local minima. Also, changes it’s
dx = 9 - h2 …(i)
sign from positive to negative at x = 0, so x = 0 is point of local
maxima.
\ S 1 = { -2, 1} and S 2 = {0}.
39 (b) Key Idea

3
h
Use formula of volume of cylinder, V = pr 2h where,

l=
(i)
r = radius and h = height
(ii) For maximum or minimum, put first derivative of r
V equal to zero
r
1
Now, volume (V ) of cone = pr 2h
3
1
Þ V (h) = p (9 - h2 )h [from Eq. (i)]
3
1
h = p[9h - h3 ] …(ii)
3 3
h/2 q
For maximum volume V ¢ (h) = 0 and V ¢¢(h) < 0.
r Here, V ¢ (h) = 0 Þ (9 - 3h2 ) = 0
Þ h= 3 [Qh </ 0]
Let a sphere of radius 3, which inscribed a right circular cylinder 1
having radius r and height is h, so and V ¢¢(h) = p (-6h) < 0 for h = 3
3
h
From the figure, = 3cos q Þh = 6cos q Thus, volume is maximum when h = 3
2
and r = 3sinq …(i) Now, maximum volume
Q Volume of cylinder V = pr 2h 1
V ( 3) = p (9 3 - 3 3) [from Eq. (ii)]
= p (3sin q) 2 (6cos q) 3
= 54p sin2 qcos q . = 2 3p
dV 42. (a)
For maxima or minima, =0
dq
We have, f (x) = 3x 3 - 18x 2 + 27 x - 40
Þ 54p [2sin qcos 2 q - sin3 q] = 0
Þ f ¢ (x) = 9x 2 - 36x + 27
Þ sin q[2cos 2 q - sin2 q] = 0
é p ù = 9(x 2 - 4x + 3) = 9 (x - 1) (x - 3) …(i)
Þ tan2 q = 2 êQ q Îæç0, ö÷ ú Þ tanq = 2 Also, we have
ë è 2 øû
S = { x ÎR : x 2 + 30 £ 11 x }
2 1
Þ sinq = and cos q = …(ii) Clearly, x 2 + 30 £ 11x
3 3 2
Þ x - 11x + 30 £ 0
From Eqs. (i) and (ii), we get
1 Þ (x - 5) (x - 6) £ 0 Þ x Î[5, 6]
h=6 =2 3
3 So, S = [5, 6]
40. (d) The non-zero four degree polynomial f (x) has extremum points at Note that f (x) is increasing in [5, 6] [Qf ¢ (x) > 0 for x Î[5, 6]
x = -1, 0,1, so we can assume \ f(6) is maximum, where
f ¢ (x) = a(x + 1)(x - 0) (x - 1) = ax (x 2 - 1) f(6) = 3(6) 3 - 18(6) 2 + 27 (6) - 40 = 122
JEE Main & Advanced Solutions (2021-18) 45

43. (b,c,d) Given function f : R ¾® R is From the graph for points of minima y 1, y 2 , y 3 ....., it is clear that
é x5 + 5x 4 + 10x 3 + 10x 2 + 3x + 1 , x <0 3 5 7 9
- y 1 > - x 1 > - y 2 > - x 2 ......
ê 2 2 2 2
ê x2 - x + 1 , 0£ x <1
f (x) = ê 2 3 8 | xn - yn | > 1, " n and x 1 > (y 1 + 1)
x - 4x 2 + 7 x - , 1£ x <3
3 3 5 9 13
ê And x 1 Îæç2, ö÷, x 2 Îæç4, ö÷, x 3 Îæç6, ö÷ ........
ê 10 , x ³3 è 2ø è 2ø è 2ø
ê (x - 2) loge (x - 2) - x +
ë 3 1
Þ xn Îæç2n, 2n + ö÷, " n.
é5x 4 + 20x 3 + 30x 2 + 20x + 3 , x <0 è 2ø
ê
2x - 1 , 0< x <1 Hence, options (a), (b) and (d) are correct.
So, f ¢ (x) = ê
ê 2x 2 - 8x + 7 , 1< x <3 Answer the following by appropriately matching the lists based on
ê
êë log ( x - 2) , x >3 the information given in the paragraph.
e

At x = 1, f¢¢ (1- ) = 2 > 0 and f¢¢ (1+ ) = 4 - 8 = -4 < 0 45. (a) For Z = { x : g(x) = 0}, x > 0
\ f ¢ (x) is not differentiable at x = 1 and Q g(x) = cos(2p sin x) = 0
p
f ¢ (x) has a local maximum at x = 1. Þ2p sin x = (2n + 1) , n ÎInteger
2
For x Î(-¥, 0)
2n + 1
f ¢ (x) = 5x 4 + 20x 3 + 30x 2 + 20x + 3 Þ sinx =
4
and since f¢ (-1) = 5 - 20 + 30 - 20 + 3 = -2 < 0 3 1 1 3
So, f (x) is not increasing on x Î(-¥, 0). Þ sin x = - , - , , [Qsin x Î[- 1, 1]]
4 4 4 4
Now, as the range of function f (x) is R, so f is onto function. 3 1 1 3
here values of sin x, - , - , , are in an A.P. but corresponding
Hence, options (b), (c) and (d) are correct. 4 4 4 4
sin(px) values of x are not in an AP so, (iii) ® R.
44. (a, b, d) Given, f (x) = ,x >0
x2 For W = { x :g¢ (x) = 0}, x > 0
x 2 p cos(px) - 2x sin(px) so, g¢ (x) = - 2p cos x sin(2p sin x) = 0
Þ f ¢ (x) =
x4 Þ either cos x = 0 or sin(2p sin x) = 0
x p
p
2x cos(px) éê - tan(px) ùú Þ either x = (2n + 1) or 2p sinx = np , n ÎIntegers.
2
= ë2 û
x4 Q 2p sinx = np
n 1 1
xp Þ sin x = = - 1, - , 0, , 1 {Qsin x Î[- 1, 1)}
2cos(px) éê - tan(px) ùú 2 2 2
= ë 2 û
p p
x3 \ x = np, (2n + 1) or np + (- 1)n æç ± ö÷
2 è 6ø
Since, for maxima and minima of f (x), f ¢ (x) = 0
px Þ (iv) ® P, R, S
Þcos(px) = 0 or tan(px) = , (as x > 0)
2 Hence, option (a) is correct.
px 46. (a) For, X = { x : f (x) = 0}, x > 0
Q cos(px) ¹ 0 Þ tan(px) =
2 Now, f (x) = 0
Y y=tan (px) Þsin(p cos x) = 0, x > 0
Þ p cos x = np, n ÎInteger.
y=px Þ cos x = n
2 Þ cos x = - 1, 0, 1 {Qcos x Î[- 1, 1]}
p
X Þ x = np or (2n + 1) , n is an integer. so, (i) ® (P), (Q)
1 1 3 5 7 2
O P1 2 P2 3 P3 4 9
2 2 2 2 For, Y = { x :f ¢ (x) = 0}, x > 0
2
Now, f ¢ (x) = 0
Þ - p sin x cos(p cos x) = 0
Þ either sinx = 0 Þ x = np, n is an integer, or cos(p cos x) = 0
p
3 Þ p cos x = (2n + 1) , n is an integer
Qf ¢ (P1- ) < 0 and f ¢ (P1+ ) > 0Þ x = P1 Îæç1, ö÷ is point of local minimum. 2
è 2ø 2n + 1
5 Þ cos x =
Q f ¢ (P2- ) > 0 and f ¢ (P2+ ) < 0 Þ x = P2 Îæç2, ö÷ is point of local 2
è 2ø 1
Þ cos x = ± , {Qcos x Î[- 1, 1]}
maximum. 2
From the graph, for points of maxima x 1, x 2 , x 3 …… it is clear that p 2p
Þ x = 2np ± or 2np ± , n is an integer.
5 9 13 17 3 3
- x 1 > - x 2 > - x 3 > - x 4 ......
2 2 2 2 So, (ii) ® (Q), (T)
Þ xn + 1 - xn > 2, " n. Hence, option (a) is correct.
46 Differential Calculus

47. (d) We have, 1ö 2


Þ h(x) = æç x -÷+
2 1 1 è xø x - 1
f (x) = x + 2 and g(x) = x -
x x x
f (x) 1 æ 1ö 2
Þ h(x) = x - > 0, ç x - ÷ + Î[2 2, ¥)
g(x) x è xø x - 1
2 x
æx - 1 ö + 2
1 1 1ö 2
x2 + ç 2
÷ æ
x - < 0, ç x - ÷ + Î(-¥, 2 2]
\ h(x) = x =
è xø x è xø x - 1
1 1
x- x- x
x x
\ Local minimum value is 2 2.

Potrebbero piacerti anche